Question Bank-Communication Systems-Excel Review

You might also like

Download as pdf or txt
Download as pdf or txt
You are on page 1of 130

CONTENTS

Preface
Dedication
Acknowledgement

Question Bank 1 Transmission Lines 1-1to 1-24


Question Bank 2 Radio Wave Propagation 2—1to2-14
Question Bank 3 Antenna 3-1to 3-24
Question Bank 4 Noise 4-1to 4-24
Question Bank 5 Amplitude Modulation (AM) 5-110 5-34
Question Bank 6 Frequency Modulation (FM) 6-—1to6—30
Question Bank 7 Television and Facsimile 7-1to7-—4
Question Bank 8 Microwave. Communications 6-110 8-6
Question Bank 9 Satellite Communications S-1to9-6
Question Bank 10 Multiplexing 10=1 to 10-6
Question Bank 11 Radar 11-1 to11-6
Question Bank 12 Waveguides 12-1to 12-8
Question Bank 13 Data and Digital Communications 13-10 13'-—16
Question Bank 14 Fiber Optics 14-1
to 14-18
Question Bank 15 Telephony 15-1to 15-6
Question Bank 16 Acoustics 16-—1to 16-12

Excel Review Center Awards


Excel Review Center Topnotchers
Question Bank 1— Transmission Lines

PROBLEM 1 PROBLEM 3
A parallel-wire line spaced al 1.27 cm 4, stripline Is formed using a multilayer
has a diameter of 0.21 cm. What is the board (dielectnc constant = 2). The
characteristic impedance? ceater track is 0.15 inch wide and 0.005
inch thick and the PC board firsitayer
‘i thickness is 0,05 inch, thick; with an
cae pci ovarall board thickness of twice the
single layer. What isiits characteristic
7, = chavacteristic impedance (€2) impedance?
0 = distance between-two condi tors Measured
rom center io center (em)
d= conductor diameter (cm| Hatutinas

Solution: 2, = charactenstic empedance (£2)


fy = relative permittivity
2D t= PCboard thickness (inch)
aoe 6 =center track width (inch)
Falta? c= center track thickness (inch)
Z, = 276l0g) (27) h=t+bie
2, = 296.81) Solution;
Z, = 300 (3

-, The characteristic impedance is 60 | | ———at |


300 ohms. 25> —=in,
= a o'}
=|
|
Wicd | 0.67 rb) 0.64
| \ hy |
ae) 18 | ea
Ina coaxial line if the-wmweer diameter is 60, | 4(0.05x2)
0:51 cm and the center conductor has a Z= el Sah
Ue ODE
diameter of 0.22 om, What is the | 0.67n(0,15}| 0.84 =
| \ 0.0525 | |
characteristic impedance?

Platintatins
a -, The characteristic impedance is
2, = characteristic impedance (£2) ie ROR:
D outer conductor diameter tem] @esee.
d= inner conductor diameter (cm) ge f PROBLEM 4
Solution: In an improperly loaded transmission
line, determine the power reflected fram
D the load if the incident power at the load
Se Leola is 500 W and the reflection coefficient Is
{0.51-)
=138lo0} ——
%0 I O27)
Z, = AO Hleputanes

oe . P..) = reflec
. The characteristic impedance fs a i
50:4 ohms. Prt = reflected power (W)
tT =reflecton coeticent

1-3
The Question Bank Series — Communications Engineering (Volume 2 - Problems) Question Bank 1 — Transmission Lines

Solution: Solution; Vilotartines


=mene
o
Pa = CP 7, =Charactenstic impedance (£2)
5) 20x10: )
VSWR = Re 12x07 |
di and d: = conductor diameter (mm)
P, = (0.71)"(500) £5
0 = condictor spacing (mm) G=1.23 mS/m
Py = 252.05 W Ri = (VSWRZ.)
Ri, = (4-2)(50) Solution: .. The conductance is 1.23 mS/m.
*. The power reflected from the load is R, = 210 ohms
252.05 W.
-, The value of purely resistive load
i. = BO
ie cosh N
ate) TR)
impedance is 210 ohms.
PROBLEM 5. Z, === cosh 1(13.98)
What is the charactenstic impedance of
an open-line with conductors 4 mm in
in wire communications system, diameter separated by 15 mm?
calculate the retum loss in dB, if the PCy tha eg Z, = 35.36 MQ
load impedance Is 660 ohms and the A microstrip line is formed using a 0.095

(2) (6)@)
characteristic impedance of the lina is Ploletranse
inch thick PC board (dielectric constant where:
300 ohms.
= 1.8), with a bottom ground plane and a
single 0.15 inch wide, 0.008 inch thick 2, = characteristic impedance (02)
track on the top. What is its 0 = distance between two conductors measured
Vlatiet inns
characteristic impedance? fromcenter to center {mm}

2, = load impedance (£2) net || 4307 (32) _ 3) d = conductor diameter (mm)


2] || ayo) | (40) 42 = relative permittivity
25. = characteristic impedance (02) Vlatatinpta:

N=13.98 Solution:
Solution: 2, = characteristic impedance (2)
= relalive permattivity “The Impedance of the balanced
Retum loss (4B)-20!0g| 3a4 h= PC board thickness {inch}
b= track width {inch}
2-wire is 35.36 Mio.

C= track thickness (inch), Tia o eee oa) 2(15)


Return loss (dB) = 201009 ee 300 | a8 if 4
600 —360| Salution:
For a parallel-wire line; determine the Z,=241,52.0
Return loss (dB) - 9.54 dB Conductance if the conductivity is 200 x
10° Sim and the conductor diameter is
“The characteristic impedance ofjan
. The return loss of the line ts 9.54 dB, 12 mm. The conductor spacing is 10
open-line Is 241.52 ohms.
mim,
__ 87 in| 5.98(0,.095) |
Frc " J1 854.41 | 0.8(0.15) 40.008 | Plitinedonss ats 1
The VSWR on a loss-free line of 50 ohm Z, =7249 The forward power ina transmission line
charactenstic impedance is 4.2. G = conductance (5) is 150 W and the reverse power is 20
Determine the value of the purely -, The characteristic impedance is 0 = Spacing between two conductors (mm) W. Calculate the SVR on the line.
resistive load impedance which is 72.4 ohms. d= conductor dameter (mm)
known to be largerthan 50 ohms:
= conducthnty (5/m)
Hates fie

PTotatioica!
yc) te Solution:
T = reflection coefficient
Determine the impedance ofa balanced
VSWR = voltage standing wave ratio 2-wire with Unequal diameters. d1= 12 C i Te
Prt == reflected power (VW)
mm and d?= 10mm, The spacing = incident power (W)
Ry = load resistance (©)
between wires is 30 mm. In= SW = standing wave ratio
#, = characteristic impedance (2)

1-4
The Question Bank Series — Communications Engineering (Volume ¢ - Problems) Ouestion Bank | - Transmission Lines

Solution: bedi) By Pseatim


Past ECE Board Problem
Galculate the velocity factor of a coaxial \, = propagation velocity (m/s)
{+ | 4=5175 m= 16974 ft Mi =welocity factor
cable used as a transmission line, with
SWR= YF ak I6.9RA Tt 4 = wavelength (m)
the characteristic impedance of 50 feng = = A
ohms, capacitance is 40 pF/m, and © = speed of light {m/s}
inductance equal to 50 wH/m _ Jangth = 4.24 ft. {= Fequency (Hz)

2. The actual length of the line is 4.24 ft. Solution


inal,
Pietyetinsia

Vp Pat
ov;
\i50 V_ = propagation velocity (m/s) PROBLEM 15
VW = velocity factor
SWR = 2.15 Past ECE Board Problem _ (ax10%)(0.68)
L= mductance (H/m)
C= capacitance (Fim) What is the actual length in feet of one- 30x10"
“The SWR of the line is 2.15, half wavelength of a coax with velocity
c= speed of light (m/s) _h Gam
factor of 0.63 at 28 MHz? Length = re

a8) 144 Solution: Length = 1.7 m= 5.576 ft


Hytatione
What is the characteristic impedance of
Vo = oy, “. The actual length of the line is
@ coaxial cable using a solid
Vy, _ 22.36x10" m/s Ve = propagation velocity (m/s) 5.576 ft.
polyethylene dielectric having a relative
¢. axioms ) = velocity factor
pemeability of 2.3, an inner conductor
of 1 mm diameter and-an outer Vi = 0.07445 A = wavelength (m)
conductor of diameter 5 mm? where
f= speed af light (m/s) PROBLEM 17
P= frequency (He) Past ECE Board Problem
1 1
Whatis the actual length in feat ofa
Plabatiin Solution: one-half wavelength of a coax with a
VL = 22.36x10" mis velocity factor of 0.61 at 27 MHz?
2, = characteristic impedance (£2)
Vo OM
0 = outer conductor diameter (mm) fiat
d= inner conductor diameter (mm) . The velocity factor of the coaxial 1"
Fine tna
cable is 0.0745. ; (3x10")(0.63)
Be = relative permittivity
210° \, = propagation velocity (m/s)
Solution: A =6.75'm Vi = velocity factor
bei)
33 03 Wa A= Wavelength {m)
d
Length= —
Past ECE Board Problem J 2 C= speed of light (m/s)
Zi ASB Whatis the actual length in feet of a
a d Length =3:3751m t= frequency (Hz)
one-quarter wavelength of a coax witha
Length =14.07-ft
velocity of 0.69 at40 MHz? Solution:
fe JARS) tog! .
| Ay
. The actual length of the line is
Z,=636 0 i
Flotirteam 11.07 Ft. = Ver Mi
f= ct
“The characteristic impedance of the Vp = propagalion velocity (m/s) (3x10° }(0.61)
coaxial cable is 63.6 ohms. Vj = velocity factor PROBLEM 16
37 xi08
4. = wavelength (m] Past ECE Board Problem h=6.78 m= 22.23 ft
¢= speed af light (m/s) What is the actual length in feet of one- A 22.23f
Langth= = = =e
Quarter wavelength of a coax with a sh, 2 2
Solution: Velocity factor of 0.68 at.30 MHz?
=p OM, The actual length is 11.12 ft.
f rh
The Question Bank Series — Communications Engineering (Volume 2 ~ Problems) Question Bank | -Transmussion Lines

PROBLEM 18 Solution:
Length = 4‘ 4
628 fR yor
Poa TL (8)
Par

Past ECE Board Problem


log 10
What is the actual length in feet ofa
one-half wavelength ofa coax with a ©, The actual length is 4.07 ft.
velocity factorof 0.59 al26 MH27 = —_——-
3W
= 754 mW
Foal 736 dB} 7
ath dh ed mao!
Plotatamts
Past ECE Board Problem
An amplifier with 20 dB gain Is + The output power of the attenuator is
Via = propagation velocity (m/s) 7.54 mW. Ford << 0,, Ds
Vy = welocity factor connedled to another with 10:dB gain by
means ofa transmission line with a joss
i= wavelength {m)
= spted of light (m/s)
of4 dB; ifa signal with a power level of -
PROBLEM 22 2, = 138
wey
log
14 dBm were applied to the system,
f= frequency {Hz}
calculate the power output Past ECE Board Problem
What would be ihe approximate seres
Solution:
impedance ofa quarter-wave matching
Hotati ity
line used to match a 600 ohms feed to
“=
Vp 6M 70 ohms antenna?
P, = output power (dBm) Z, =99.23.9
_— (8xi 0" }(0.59) PF, = input power (dBm)
Z, +1009
G; = total gain (8) HTertiatioms;
26x10" Ly = total loss (dB)
A=6.8 m=22.33 ft 2, = quarter-wave characteristic impedance (02) .. The impedance of the balanced 4-
4. 222331 wire IIne is 100 ohms.
Length= — =MiTG hh:
Solution 2, = characteristic impedance of the line ({2)
2 2 2 2, = load impedance (£2)
Feat} = Pyaten) + Bide) — La] PROBLEM 24
“The actual length is 17.16 ft. Solution:
Py(dein) = —14(apiny ¢ (20 410)—(4) Past ECE Board Problem
What is the characteristic impedance of
P, =12 dBm B= 258, a single wire with diameter d = 0.25 mm
PROBLEM19
Z,' = ,/(600)(70) = 204.94 0 placed at the center between grounded
“The power output of the system Is parallel planes separated by 1mm
Past ECE Board Problem
12 dBm. apart? The wire is held by a material
What is the actual length in feet ofa -. The approximate series impedance of
one-quarter wavelength of a coax with a with a velocity factor of 0.75,
the quarter-wave matching line is
velocity factor of 0.695 at 42 MHz?
bad ht)
1 At ea 204.594 ohms.
Notations:
Past ECE Board Problem
Fata (hions; An attenuator has a loss of 26 dB. Ifa PROBLEM 23 2, = characteristic impedance (£2)
power of 4 VV is applied to the
Vs = propagation velocity (m/s) attenuator, find the output power, Past ECE Board Problem h ='spacing between twa parallel planes (mm)
Vy = velocity factor What is impedance of a balance 4-wire d= conductor diameter (mm)
4. = wavelength (m} with a diameter of 0.25 cm and spaced f= relative permittivity
€ = speed of light (m/s) ] Letert juin 2.5 cm apart using an ingulatar with a
f= frequency (fz) dielectric constant of 2.567 Solution:
L = loss (dB)
Py, =Inpul. power (W)
Solution; Flotabinns:
Poy = ouput power (WW)
Ne: Mi
(ey Solution:
D= conductor spacing (cm)
_ (Sxi0"}(0.695) PB
d= conductor diameter (cm)
aaxio (dB)
L(dB) op
= 10tog—*t = relative permittivity
al
4=4.96 m=16.28 ft.
1-8
The Question Hank Series — Communications Engineering (Volume 2 - Problems) Question Bank | — Transmission Lines

d
bid 0] ed) Solution: Notations:
For of —< 0,75, Past ECE Board Problem
Determine the conductance of a two- T = rellection coefficient
138 ( 4h|
25 = e og ea wire open line with the following 2, = load impedance (C2)
parameters: 0 = 4 in d= 0.1 in. and
2, = Claracteristic impedance (2)
40). p= 2.6 x 10° O-m. Po = ‘ellected power (Wi)
Z, == 138(0.75) I log (0.25),
Pine = incident power (W)
Hintutiians
23 Tati 7S Soluton:
where G = conductance {Sim}
d=0.25 mm
D = spacing between two conductors (in) -. The input impedance of the line is
d = conductor diameter (in) 150 ohms.
h=7mm 242;
o = conductivity (5/m)
_ (80= j25)=75
SN, 4078 p= resistivity (Chim)
ea TS ea ~ (80=j2sy4-75
he
Solution) Past ECE Board Problem p= 35:95 2-135
. The characteristic impedance of the Determine the standing wave ratio of a 127-482-1731
single wire line is 75 ohms. 300-2 line whose load is 400+ {150 ©. t= 0284 -123.69—|7|.40

og Prat = (Ir) x 100%


PROBLEM 25 Plotations: Pas (|

Past ECE Board Problem 7 = reflection coefficient roel = (0.28)x 100%


me
What isthe phase coefficient ofa SWRA = standing wave ratio
transmission line at the frequency of 10 G=2,76 «107 Sim 2, = |oad impedance (£2) Fit= 7.84%
MHz ifthe velocity of propagation is 2.5 ve
x 10° mis? where 2, = charactenstic impedance (£2)
i { ”. Tha amount of power reflected back
Solution:
g=— SPP

p 26x10 to the source Is 7.84 percent of


Hat ucpatidgs
o=3,.85% 10° Sim incident power.
V; = propagation velocity (m/s)
“The conductance of the line is
f} = phase coefficient (rag/rn}
2.76 x 10° Sim. PROBLEM 30
f= frequency (Hz)
Past ECE Board Problem
Solution ae) 13 _~ (400+ 150) - 300 What is the attenuation of a. 50-02
=025244.22 coaxial cable if it has resistance’ of
(400+ j150)+300
Past ECE Board Problem O.121-Cuim?
eaiteesnee What is the input impedance of a {20 = 0.25.444.22
Va Ma transmission line if its characteristic
2n(10 x 10°} impedance is 300 (2 and the load is 600 SWR = oe = 167 Fetes:

eee 108 0? Assume a quarter wavelength


(t= allenuation constant:(Np/m)
_o.2s
fi=0:25 (2at section only “The standing wave ratio of the line is A= line resistance (Qim)
2, = characteristic impedance (()
Potaliona;
». The phase coefficient
is 0.25 rad/m. adit) Wd)
Z,, = input impedance of the line (02) Solution:
4 = load impedance (©) Past ECE Board Problem
R
2, = characteristic impedance (Q) What proportion of the incident power is a== ——
7 (Neperilength)
(Neper/length
¥ = propagation constant reflected back from the load fora 75-12
| = length ofthe tine line terminated with Z,= 50 - j25 9?

1-10
The. Question Bank Series — Communications Engineering (Volume 2 - Problems) (Question Bank | — Transmission Lines

d= conductor diameter (mm) Flefutineis Vv.


= = 121% 107 Np/m dX Pao
& = relative permittivity f
Vp = propagation velocity (m/s)
a=1.21x10° Mx a.6a6
ce Solution: £, = relative permittivity pi AEOKTO nd =0:374m
m Ne
i = velocity factor 530x101
a = 0.0105 dBim 75° | -0.07am
L=0.374] 2)
-, The attenuation constant of the line
Z,= 7Eloa| 2)
ae qd
Solution: 360 |
is 1.21 x 107 Np/m or 0.0105 dB/m. L=7.8em
fs =27 al 702) |
V, = He
1 yea The length of the transmission lina
ve
at) TA Z, =298 2 Z 1 is 7,8 cm.
Find the inductance of a 5 meter length » 2A
transmission line if the characteristic - The characteristic impedance of the
line is 258 ohms.
V, = 0.69 PROBLEM 36
impedance is 52 ohms and the
capacitance is: 75 pF/m. Vi=V.C Calculate the SWR ofthe line ifthe
V, = 0.69(3x10") characteristic impedance of the line is
PROBLEM 33 75 and is terminated by 50.0,
Flotations:
A coaxial cable with inner diameter of 3 V. = 207x10°m/s
mm and an outer diameter of 9 mm.
2, = characteristic impedance (2) Delermine the characteristic impedance
Flatotivns:
» The propagation velocity of the
L = inductance (Him) if the dielectric is polyethylene having signal is 207 x 10° mis. SWR = standing wave ratio
C= capacitance (F/m) 6 = 2.3;
2, = ‘characteristic impedance (2)
Solution:
PROBLEM 35 2. = load impedance (3)
FHejations:

Z,=,/—L
A 540 MHz signal is propagating along.a Solution:
2, = characteristic impedance (£2) RG 8/u coaxial cable with a velocity
Tee factor 0.66. Fa 75° phase shift is
& = relative permittivity
L=z,2¢ 0 = outer conductor diameter (mm)
needed, what is the length of the
transmission line?
L=(52)' (75x10) d= inner conductor diameter (mm)
L=203 nim Solution Potiativieys
¢. The standing wave ratio is 1.5,
L=203 nHim(5 meter)
b=4,015
pH 138 D Ve = propagation velocity (mis) PROBLEM 37
Po fs log) —
«The total inductance of the line is alt Ve = velocity factor
C= speed of light (ms)
A ‘transmitter sends. a 5:W of power to a
75 {1 line. Suppose the transmitter and
1.015 pH, f= frequency (Hz}
the line is matched but the load js not.
}, = wavelength (m) Determine the power absorbed by the
aeNS ee Z,=43.4.0 L= Jength (cm) load if the coefficient of reflection is
0:75,
An open-wirée transmission line with a 2 Solution:
-. The characteristic impedance of the
mm diameter each separated by 12 mm line is 43.4 ohms.
Calculate the characteristic impedance. Ve= Vc Fotatiziiea:

PROBLEM 34
V, =0.66(300x10°)- 198x10°mis tT =relleciion coeificient
Hlotistinws Pie = reflected power (W)
A coaxial transmission line uses Teflon P,. = incident power (W}
2, = characteristic impedance (2) as dielectric (6, = 2.1). What is the P| = power al the load (W)
0 = distance between two conductors measured propagation velocity of the signal?
from center to. center (mm)

1-12 1-13
The Question Bank Series - Communications Engineering (Volume 2 - Problems) Question Bank | — Transmission lines

Solution; Notations: Solution: f adB |

Pa ate 2, = load impedance (©)


ak = 59 som)
1Np |
P, =0,75"(5 W) 2, = characteristic impedance (€2) = 1,5 dB) ————_
Ze = inputimpedance (62) pers (ts
P, =2.6125 W
2. = wavelength (mi) aW=0173
P, = Pac — Frat Vp = propagation velocity (m/s)
Vp =V/G Pout = Rye
P,=5 W-2:8125W 1 = frequency (Hz)
Pog = 50e 20-173)
Pp =2.1875 W 0 = phase shitt (degrees) V,= 0.66(300x10") =198x10°m/s
Po, = 108.2W
« The power absorbed by the load
Solution:
ha f
Is 2.1825 'W. -, The power received in the antenna
= ed =0.0825m is 106.2 W.
eel 2400
PROBLEM 38 2 0.9(300x10" )
Since this is a shorted stub, parallel PROBLEM 42
A generator sends a 250 W of power 50x10" resonant, quarter wavelength will be used
down a line. Ifthe SWR of the line is TDR equipment receives the signal
im
2.5, determine the power absorbed by #=———| 360" )= 66.67" ft A 350 ns after the test signal had been
the load. 5.4aa ) 4 transmitted. If the discontinuity of the
Zee: 2, +12, tand line is found 35 meter from the test
p= one 0.020625 point, what is the velocity factor of the
Hotations:
Z, +2, tang
line?
n66.67 ] L=2.0625 cm
Pac = incident power (W') zZ,, = (50 i + j(75)(ta
P.= power at the load (W)
75+ j(120)(tan66.67) «The length of the shorted section Vetatines:

SWR = standing wave ratio is 2.0625 cm.


7. =34'4—j122
d=distance from the test point and the
Solution: discontinuity paint (rm)
”. The input impedance of the line dt}8 Ne Y, = propagation velocity (m/s)
is 34.4 = j12.2 ohms,
_4SWR_ The transmitter has an output power of L= time traveled {rom test point to discontinuity
150 W) Suppose the line is 50 meters point and back (sec)
(1+swry ™ long and perfectly matched with a loss ¢ = speed of ight (m/s)
eae) 3H eC)
4(2. of 3 dB/100m, how much power is
pee5) (sow) A parallel tuned circuit is to be received in the antenna? Soution:
{1+2.5) constructed from a shorted section of
P, = 204.1W parallel-wire line. Ifthe operating Vet Vyct
frequency is 2.4 GHz and dielectric used Hitations:

is polyethylene, what is the length of the Garon ae


.. The power absorbed at the load shorted section? (2 = attenuation constant (dB/m)
is 204.1 W. Vi = 2d

U=lenath of the line (m) ci


Pa = input power (WV) (2})(35m)
Flatations: Pas = output power (WW)
rah (300x10°m/s)(250x10-*s}
Vy = velocity factor
ATS ohm characteristic impedance Solution: Vi=0.939
transmission line is terminated in by a €, = relative permittnaly
load impedance of 120 ohms. A = wavelength (m)
. The velocity factor of the line
Determina the input impedance looking Vp = propagation velocity (m/s) Tx i G ,Antenna
into the line 1 meter long, if the line has Is 0.933.
{ = frequency (Hz) Pi, Pout
a Velocity factor of 0.9 and operates at a L = length of the line (cm)
frequency of 50 MHz.

1-15
The Question Bank Series —- Communications Engineering (Volume 2 - Problems) Question Bank | — Transmission Lines

PROBLEM 43 - The SWR of the line is 1.76. R ite) TU


Vio fea .
= V.

The line has 97% velocity factor. If the A radio frequency 50 ohms transmission
two minima are located 30 cm apart, ling is connected to a coll with internal
determine the frequency.
PROBLEM 45 Mg R475 4.75 resistance of 20 ohms.and inductive
A transmission line 90 meter long has a Via R R reactance of 5 ohms ata frequency of

Vlotitions:
characteristic impedance of 75 ohms
The Jine is connected through a 15'V de
18Ms
R Mio
410 MHz The line is to be matched to the
load by means of a quarter-wave line
source and 75 ohms source resistance and a stub that ana connected across
A= wavelength (m) R= 75 the load. Calculate the characteristic
at lime t= 0. What is the Vollage across
Vi = propagation velocity (m/s) the input immediately after t = 07 [¥-1] impedance of the quarter wave
{ = frequency (Hz) Ya
transformer,

Solution: Flatatienis:
R= 76 =150 Hota thoes:
Vi = inpul voltage (V)
4. =2(30 em)=60em Ve = source voltage (V} YU= Wad admittance (5)
74 = charactersstic impedance (Q)}
fe - The resistance that terminates the
=

Solution:
line is 160 ohms.
¥. = admittance after tuning out the reactance (5)
97(300x10") & = load impedance (2)
a

Vy,
0.60 y= 2 edit)
1H
7 =
dy =
impedance alter tuning dut the reactance (©)
quarter wave characteristic impedance (£2)
f=485 MHz
Mies iS 75V A parallel wire transmission ling has an Kya = stub reactance (€2)
2 inductance of 120 nHim and a Bee = stub susceptance (5)
+, The frequency of the signal Is
485 MHz. characteristic Impedance of 320 ohms.
.. The voltage across the input Calculate the capacitance of the ling, Solution:
immediataly aftert = 0 is 7.5 V.
a) | Flat tina!
The power incident inthe line is 200 W. hte
tH eT)
Assuming the reflected power is 15 W, 2, = charactenstc impedance (£2) ‘ie
1 :
-=(47-ji2)
mS
A transmission line 90 meter long has a
what is the SWR of the line?
characteristic impedance of 75 ohms L = inductance (Him) +2045] (7-92)
The line is connected through a 15-V de C= capacitance (Fim)
Flatutiena: source and 75 ohms source resistance wit stub connected
al time t= 0. The voltage at the input Solution:

= reflection coefficient end changes to 10 V, with the same ic ¥f=G_ =47 mS


polarity as before, at time t= 1.5 ps.
Z.=| eel

O|r]
Pa = reflected power (W)
What is the resistance that terminates 2.-==
Poe = incident power ('W)
the lines? S,
SWR = standing wave ratio aL i
Cc
Solution:
ze “arms
= = 713-6
Plotitinas 2
c_ 20x 10 2, = Ze
Vie = voltage across R (V) (320)
1+ Frat
Vs = source voltage (¥) G=1.17 pFim Z, =,/(50)(21-3) = 32.6.0
Pus
SWR =—__ R= resistance (C2)
thee .. The capacitance of the line ~, The characteristic impedance of the
Poe Solution: poe is 1.47 pFim. quarter wave transformer is 32.6 0.
maa
1+ [15°
swR-—)200.
4.75 a) Wee
a (1S
300
The Question Rank Series — Communications Engineering (Volume 2 - Problems) Question Bank 1 — Transmission Lines

aed Te) Flotation: Solution:


Ar-open-wire line has a 3 mm diameter so, 4
2, = characteristic impedance ({2)
separated 6mm from each other by. an Is f= 30" | ‘|
air dielectric. Calculate the charactenstic
10 VA) Wl 50>
L = inductance (Him) 360 |
impedance of the line C = capacitance (F/m)
4. = wavelength (nm) =a
y B= 6 Yen
f= frequency (Hz)
Vistitdsais

ae V,= propagation velocity (m/s) (ha ole = Pat /LC

2; = characteristic impedance (£2) \ Solution


=2ef/LC

ma
0 = distance between two conductors measured
vt,

Nae
10
from center to. center (mm)
d = conductor diameter (mm) "50+ 50 Teta

ec]
|, =100 mA

oy
Ey = relative permittivity 1

4.
Solution:
neyo
f
te
(253 x 10)(56 x 10°)
po Cf
Ine iter
“The source current or line current | 12(30 x 10°) | 46.8x 107°
which flows into the line is 100 mA,
276 f 3p) Vp = 2.65 x 10° L=165 ne
25> 09 se
al ty
aN ara
Mp
276. | | 2(6) “The inductance of the line is
Za 7 oa] a_ A certain instrument measures the input
2.65x 10°
=
165 nHim.
impedance of a line and gave a reading
Z, = 1662 of 1200 ohms when iis far-end. terminals ~ 265
x 10°
open circuited and 300 ohms when it is A=1'm athe
short circuited. Calculate the
“The characteristic Impedance of the The reflected current at the load ofa
charactenstic impedance of the line. Electrical length = 1.5m (_%,)
line is 166 ohms. \ im mismatched line is 3 mA. If the reflection
Electrical length =1.5 coefficient of the load is 0.7, what is the
Flatidinna: incident current?
ae) eT Electrical length = ; A
A radio frequency coaxial transmission #, = characteristic impedance (C2)
Fadatirnes
line has a characteristic impedance of Zoe = open circuit Input Impedance (©)
50 shims and is connected across the -. The electrical length of the line is.
4. = short circult Input impedance (©)
terminals of a signal generator, The 5h. T= rellection enefficient
sending end voltage is 10'¥. Assuming let = reflected current (A)
Solution:
that the generator, line and the load are lee = Incidert current (A)
matched, determine the current which ———— a)
1A ee)
flaws into the line, a5 V eee
oc $e Solution:
A radio - frequency transmission line
2, = (1200)(300) = 600-0 has a capacitance of 46.8 pF/m anda
Totui tiinis: phase constant of 30°%/m. Detennine the
© The characteristic Impedance of the inductance of the line at a frequency of
line is 600 ohms. 30:MHz
24 = characteristic impedance (£2)
2, = load impedance (2)
|; = current source (A) Vlotinitcera: _amA
PROBLEM 52 =4.3 mA
Vs. = voltage source {V) aiaOiT.
A transmission line has an inductance of Vu= propagation velocity (m/s)
Solution: 253 nHim and a capacitance of 56 f= phase coefficient (radi) ., The incident current is 4.3 mA.
pFim, lf the physical length of the line is f= frequency (Hz)
Since the line is properly tenminated it can 1.5m, calculate its electrical length ata L = inductance (Him)
be represented by the equivalent circult frequency of 265 MHz. {= capacitance (F/m)
below,

1-19
The Question Bank Series — Communications Engineering (Volume 2 - Problems) Question Bank | —‘Transmission Lines

oat ee Bede
A radio frequency 50 ohms transmission
eS ey
@o.= characteristic impedance (©)
A radio frequency transmission tines has |, _ SWR-1 line is connected to a coil with internal
a characteristic impedance of 75 chms. SWR 44 resistance of 20 ohms and Inductive Sclution:
lf the line is terminated by an aenal with Se reactance of 5 ohme at a frequency of
an Input impedance of /2 ohms, “a4 10 MHz. The line Is to be matched to the Poa
calculate the SWR of the line. 1-087 load by means of a quarter-wave line eae ae ere
i and a stub that are connected across P.. =(1x 10") 75
the load, Calculate the reactance of the P.-75 uw
Hatintivnn: Pret at Pine
=i Po =Poe— Por stub, = v
+ = reflection coeficient Py = (1 : 1 Peo t= Soe
re as +t
Prat = reflected power {W) > Holatioves: are
P.. = incident power (WW) Po= (1 0.67 )Pne t= S0= fy
Zy = characteristic Impedance (2) P, = 0.56P,,, ¥, = load admittance (5) 50475
% = load impedance (Q) P. —56%P 4 = load impedance (£2) c=—02
SWR = standing wave ratio A ie Kini = stub reactance (Q) P,, = FP,
ee -, The percentage of the incident power ee a eta) cose (0.2) (75 uW)
Solution dissipated in the load is 56 percent. \
Solution:
, x
P,.=3 UW
:

po
7,42, bite) Ae x al .. The power reflected from the load
a. 2-75 The reflection coefficient on a loss-free z, Is 3 WW.
"73475 line of 72 ohms characteristic Y= 1
7=-0.02 impedance ls 60 %. Calculate the load r 20+ 5] ;
14| | impedance (purely resistive) and which Te, (47-12) mS Mai
sweet is smaller than 72 ohms. ; A loss-free line transmission line has an
1=|r| Bere. = 112 m5 inductance of 263.2 nH/m and a
: 1 capacitance of 46.8 pF/m. If the
R= noe Motations: Xora = ae 83.3.0 generator produces a sinusoidal voltage
eae ae ' aa at 3 MHz of 2 V peak, determine the
SWR = 1.04 2, = characteristic: impedance ({2) phase change if the line is 3 meter long
Z, = load imperlance (2) ., The reactance of the stub Is
“, The SWR of the line is 1.04. ¢= reflection coaticient 83.3 ohms. nie
OLE RA?

PROBLEM 56 Sot PROBLEM 59 [} = phase coefficient (rad/m)


A lossless transmission line is Zim Fi A lossless line has a 75 ohms f= frequency (Hz)
connected to.a 10 ohms load. If the a eoeee characteristic impedance and is = inductance (Him)
SWR = 5, calculate the percentage of A ieee connected to 50 ohms load. If the C= capacitance (F/m)
the incident power that is dissipated in since 2, is smaller than 2, Sending end current of 1 mA flows inte
the load. Tis 2 negalive quantity the line. Calculate the power reflected Solution:
Heese fromthe load:
Potarions: ay a Fx eve
—O.6(2Z, +72)=2Z, -72 Mavations: i= 2n(3 x 10")| (2832 x 10")\(46.8 x 10) |
t= reflection coefficient 0.62, -43:2=Z-72 :
Poy = reflected power (W) 4.62, = 28.8 T= reflection coefficient p= 0.02% Aer
Pre = incident power (W) Zz =18.0 Prt = reflected power (WW)
SWR = standing wave ratio Pic = incident pawer (W)
fee = Indident current (A)
.. The load impedance is 18 ohms.

1-20 1-21
The Question Bank Series — Communications Engineering (Volume 2 - Problems) Question Bank | —Transmission Lines

phase change al 3-meter jong Fhita tines Pw =(1- a Li Hotabona:

vit 180°
pl (0.021 rad) | amx is) Pu = power at the load (Wi) Piw = (1-0.47)(56.23) 2, = characteristic impedance (22)
Ps. = power atthe generator (WV)
fl=11:34 Lr = total loss of the line (dB)
Poy = 47.23W 2, = load impedance (2)
SWR = standing wave ratio

“The phase change is 11.44 degrees. Solution: >, The power at the Input of the antenna
Solution:

Py = P10
[bry is 47.22 W.
Z,=500
aT
ate)
An AM transmitter-sends a 50 W of Cie ee (30m) SWR =—* Z,

power into a 75 ohms lossless coaxial A coaxial line having an impedance of t


cable, The input impedance of the Legg = 0.25 dB 50 Q is used ina Cable TV system as Z, =1-6(50)
= 80 42
antenna at the feed-point is 300 ohms Nie drop cable. If the maximum line voltage
Phy = 100(10)." "40! is 15 V and 12 V minimum. What is the Z, =2709
(purely resistive). How much power is
SWR of the line?
radiated assuming unity gain, no Py =9441W swr = 2:
coupling loss and the antenna is Z2
Zane z
lossless?
-. The power that reached the load 30 m Hatattons:
away is 94.41 W. > SWR
Flotiativns: SWH = standing wave ratio
Z,=55176 780
ait} 600
t=reflection coelficent
T= reflection coefficent dit TS Vie = reflected valtage (V]
Pia = Fathated power (WW) Vie = incident voltage (V) +, The characteristic impedance of the
The output power of the transmitter is
Pa. = incident power (W) 100'W. How strong must the signal be Vee= Maximum line volage (Vv) line is 80 ohms.
2, = load impedance (2) at the antenna if the antenna is Vw= minimum line voltage {V)
Zo'= characteristic impedance (£2) connected to a transmission line 50
meters long with reflection coefficient of Solution:
Solution: 0.4 and.an attenuation constant of §
aB/id0m? ahs
Peder
Vlatations ee Met Mt — Mit
Me Wav
re

300475 Piw = power ai the load _1h=


415-12 a4
t= 06
Ps = power atthe generator (W) 4512
Pre= Inddent power (W) 21+0.14
Pad =(1-17 )Pre Ly= total loss of the line (dB) 1014
Pu = (1—0.67 }(50W) T = reflecion coefficient
SWR=1.25
P..=32W Solution:
*, The SWR of the line is 1.25.
“The radiated power from the antenna (ier
assuming unity gain is 32 W. P,.=P.10 “" PROBLEM 65
\
L =| 5——_—
An SWR meter |s Used to measure the
aA ey TB — | es (Som) degree of mismatch on the line. The
SWER meter records 1.6 when the line is
A properly terminated line has a line buggy = 25 08 terminated with 80 ohms and 2.2 when
loss of 1.dB/120m. If the power output of ue the load is change to 176 ohms. What is
the transmitter is 100 W, how much P,. = 100(10) the characteristic impedance of the line?
power reached the load 30 m away?
P,. = 56.23W
The Question Bank Series — Communications Engineering (Volume 2 - Problems)

Votes

Question Bank 2
RADIO WAVE PROPAGATION

1-24
Question Bank 2 - Radia Wave Propagation

beg] 1 Flatations:
How far away is the radio horizon of an
antenna 100 ft high? f= critical frequency (H2}
MUP = maximum usable frequency (Hz)
0 = angle of incidence (ceqrees)
Plotations:
Solutio:
d= racho horizon distance (mi)
jy = transmitting antenna height (it) f, =MUF cosd

Solution:
i. =(17 MHz)cos 75
f= 4.4 MHz
d= Jan, = /2(100)
-. The critical frequency is 4.4 MHz.
d=14.14 mi

¢. The radio horizon distance js 14.14 PROBLEM 4


miles.
What power density is required ta
produce electric field strength of 100
PROBLEM 2 volts per meter in-air?

| What is the maximum line-of-sight


distance between a paging antenna 250 Vlotathins:
ft high and @ pager receiver 3.5 ft of the
ground’? (a= power density (Wim)
& = electric field strength {V/m)
Votations: o¢ = characteristic impedance of free space (12)

-f = radia horizon distance (mi) Solutorr


hy = transmitting antenna height (it.)
hy= receiving antenna height (ft.)

“Soluti jon;

d= 2h,~ [2h +. The power density is 26.5 Wim?


d= /2(250) + /2(3.5)
d=25 mi PROBLEM 5
« The maximum line-of-sight distance lfthe critical frequency is 10 MHz in sky
is 26 miles. wave propagation, what is the best
frequency to use assuming 30° of
radiation angle?
PROBLEM 3
At a certain time, the MUF for Vintuttous:
transmissions at an angie of incidence
/Of 75° is 17 MHz. What is the critical {. = critical frequency (Hz)
frequency? MUF = maximum usable frequency (Hz)
0= angleof incidence (degrees)
OWF = optimum working frequency {Hz}
2-3
The Question Bank Series — Communications Engineering (Volume 2 - Problems) Question Bank 2 - Radio Wave Propagation

Solution N =o. of free electrons perm? = The resultant field strength is 0 Vim. G; = transmitting antenna power gain (unitless)
| = frequency (Hz) 4. =owavelengih (m)
Optimum working frequency is the best
¢ = speed of light (m/s)
frequency to use in sky wave propagation, Solution: at)
1 Be] {= frequency (Hz)
OWF = 859 MUF { past ECE Board Problem
According to the Philippine Electronics Solution:
OWF = 0.85 (20)
OWF = 17 MHz Gode, the Maximum accumulated
Permissible Dose (MPD of radiation _ 3) (Ahh,
where:
. fasxtcoai ‘associated with a radio transmitter) of a 7A 2, ad’

Y (exiory’ | 68 years old man is


MUR ” cost
== 19 | a ate
i cos60 n=£0.99 cq =(70.44)) 22204)
§=90-—30 = 60° Flotatiine: | 1:875(30x10")
«. The refractive index is 0.99,
MPD = Maximum) Accumulated Permissible Dose
E,= 41,78 p/m
“The best frequency to use is 17 MHz.
(rern] where:
PROBLEM & ‘N= person's age
PROBLEM 6 “rem = roentgen equivalent man
_¢ __3xi0"
Whatis the resultant field strength f 160x10°
In skywave propagation, what isthe between Waves traveling in different A= Le7om
entical frequency if the maximum paths (direct and reflected paths) when -Salution:
number of free electrons in a certain the product of the antenna heights = (30P,6,
ionospheric layer is 1.5.x 10" per cubie (transmitting and receiving) above the MPD =5(N—18)
Su = /30(100)(1:64)
meter? reflecting plane is equal to half- MPD = 5(68-18)
wavelength times the total direct E.=70.14 V
distance path between the two antenna? MPD = 250 rems
Matatious: -. The electric field strength received is
“The maximum accumulated 41.78 pVim.
fe = critical frequency (Hz) Matations: permissible dose is 250 rems.
Nes = Maximum number of free electron per m*
&s = direct path field strength (Vim) PROBLEM 11
Solution: & = resultant electric field strength (Vim) ait)
] NS a
Past ECE Board Problem
& = geometrical length difference between '{ Past ECE Board Problem What is the effective radiated power
f= 9,/Non difference between direct and reflected paths(m) Ina VHF mobile radio system, the base (ERP) of a repeater with 450 W.
hand hy = height of transmitting and receiving ‘Station transmits 100 W at 160 MHz transmitting power output, 4 dB feedline
f, = 9/1. 5xi0™ antennas above reflecting plane (m) frequency using half-wave dipole loss, 6 dB duplexer loss, 7 dB circulator
= TMZ d= direct distance path (m) ‘antenna 20 meters above ground loss and feedline antenna gain of 25
Calculate the field strength at a dB?
Solution: feceiving antenna at a height of4
*. The critical frequency for skywave
| meters and a distance of 30-km. (Gain
propagation is 11 MHz. fhe
‘\of antenna is 1.64),
f= 25, sin2x|—| Fotattows:
= (2a J

Ads 9 eee Fan, sin2x| = Poth tote:


ERP = Elective Radiated Power (dBW)
A Py = transmitted power (dew)
In Wave propagation, determine the
refractive index of an ljonospheric layer £= 25, sinn “£5 = electric field strength of the transmitting solution:
with 100 x 10° free electrons per cubic 2=0Wm antenna [\)
meter at 5 MHz frequency. 2h= electric field strength of the receiving ERP a =P (een) ~ Total loss, +Gain,,
where antenna (Vim)
Hlotations: 2n,h, 2/(4)(3)] hand h. = height of transmitting and receiving ERPay = tOlog| “2° ~(44647)425
Se antennas above rellecting plane (m)
= distance of the two antennas (my ERP..,,,= 34.53 dBW
n= refractive index n= 4,
Fr= transmitted power (¥)
The Question Bank Series — Communications Engineering (Volume 2 - Problems) Question Bank 2 - Radio Wave Propagation

Solution: Solution:
ERP = Ing" pao
MPD =5(N-18), rems
ERP = 2839.37
MPD
= 5(50-18)
-. The effective radiated power is MPD = 160 rems
2839.31 W.
¢. The maximum accumulated
== 189.74 mVim f
permissible dose is 160 rams. cos G= —
MUF
ite
18
-. The electric field strength is i, =MUF cos0
Past ECE Board Probiem
A microwave transmitting antenna is ae03 a | 189.74 mVim. f, = (7050.50 kHz)cos35°
600 feet high The receive antenna is Past ECE Board Problem f, = 5.775 MHz
240 feet high, The maximum The total power delivered to the radiator a) 1A) f, = 5775 kHz
transmission distance is of an isotropic antenna is 200,000 W.
Determine the power density ata Past ECE Board Probiem
What is the index of refraction ofa “The critical frequency is 6.775 MHz.
distance of 100 meters.
Matatins: certain substance if a light travel through
the substance at 100 meters ata time is
d = radio horizon distanoe [rmi) Flotertrone: to 140 maters to air? PROBLEM 12
h.= transiting antenna height ({t} Past ECE Board Problem
hy = receiving antenna height (it) go = power density (W/m?) Determine the effective radiated power
Flotations:
P, = transmitted power |W) of 20 KW TV broadcast transmitter
Solution: 1 = distance from the antenna (m} whose antenna has a field gain of 2.
Aa retractve index

Solution
Vp = propagation welocity
3 42h, r «PR; Hotations:
d= ,/2(600) + /2(240) For-an isotropic radiator,
Solution:
d=56.55 mi ERP = Efiective Radiated Power (W)
= Fi >
4nr°
na Pla Pr = transmitted
power (W)
>. The maximum transmission distance Vp[aubstance} G; =anlenna power gaz
is 56.55 miles.
_ 200,000 W
4n(100)° n=a0 =14 Solution:
100
p=159 Wine
ia)
1 he ERP
= PG,
“. The index of refraction is 1.4.
Past ECE Board Probiem ERP =(20 k¥V)(4)
>. The power density received from the
As specified by the Natipnal Committee isotropic radiator is 1.59 Wim’, ERP = 60,000 W
on Radiation Protection Measurement it) oe wheng =
(USA), what would be the maximum
accumulated permissible dose to the ad 0198 Past ECE Board Problem G, = (field gain)’ = 27 =4
whole body of a 50 year old man? Determine the critical frequency yalue of
Past ECE Board Problem an HF signal if its maximum usable ”, The effective radiated power is
Calculate the electic field intensity in frequency is 7050.50 kHz at 35 degrees. 80 kW.
Mototinns millivolts per meter at 30 KW from a5 incidence.
km sounce.
MPO = Maximum Accumulated Permissible Dase dy
Notations:
N= person's age
Flrotertients: Past ECE Board Problem
fem = foenigen-equivalent man
i = critical frequency (Hz) What is the free space loss, in dB,
& = electric held strengtiy (¥/m) MUF = maximum usable frequency (Hz) between two microwave parabolic
P, = transmitted power (WW) antennas 33.0 kilometer apart operating
8 = angle of incidence (degrees)
f= distance from the antenna (m) at 7.0 GHz?

2-5
The Question Bank Series — Communications Engineering (Volume 2 - Problems) Question Bank 2- Radio Wave Propagation
‘)
Motations:
PROBLEM 21 PROBLEM 23 PROBLEM 25
L. = path loss (dB) Past ECE Board Problem Past ECE Board Problem AGILA satellite is located 36000 km
fie = trequency (GHz) Ifthe maximum number of free electrons Ifthe velocity of propagation in a certain above the earth's surace, Assuming
d = distance between two antennas (km) ina cenain ionesphenc layer is 2x 10 medium is 200 x10", what is its frée space condition what is the path
per m3, what is its critical frequency? refractive index? loss of the signal ifthe operating
Solution: frequency. is 3 GHz?

Lp =92.4+20 log fy, +20 log-d,_, Votations: Platatints:


Flotation;
Le =82.4420 log 7 +20 log 38
n = refractive index f= refractive index
Lp =140.89.dB Nha: = Maximum number of (ree electrons per m? c= speed of fight (m/s) L = free space path loss (dB)
= propagation velocity (m/s) d= distance from the antenna (km)
.. The free space path loss is Solution: { = operating frequency (MHz)
140.89 dB. Solution:
Solution:
f= 9a) Nini
a a f-av2x07 L, =32.4+20logd,,. + 20!0gf,,,,
o

=
Past ECE Board Preblem
In skywave propagation, if the critical
f= 12.73 MHz
_-ax10® L, = 32.4 + 20log(36000) + 201og(3000)
frequency is 25 MHz, what is the best “The critical frequency is 12.73 MHz.
200% 108 L, =32.4+91.13+69.54
n=1.5
frequency to use? Assume radiation L, =193.07 dB
angle of 45°
-. The refractive index is 1.5.
ae ees The free space path loss is
Hotatinns: Past ECE Board Problem 193.07 dB.
What is the refractive index of an PROBLEM 24
{, = critical frequency (Hz) ionospheric layer with 100 x 10° free
Past ECE Board Problem
MUF = maximum usable frequency (Hz) electrons perm? Assume frequency to
What is the propagation velocity ofa
ND)’
be at 20 MHz.
0 = angle af incidence (degrees) signal ina transmission line whase A certain earth station transmitter output
inductance and capacitance are 5 phim poweris 10 KW, transmitting antenna
Solution: Hotationrs ' and 20 pFim respectively? gain 100 dBi, recelving antenna gain 75
dBi, uplink and downlink frequency of
2.4 GHz and 2 GHz respectively: What:
n= relractive index
Motations: is the received signal level at the.
N=no. of free electrons perm?
satellite?
f= frequency (Hz)
Vp = propagation velocity (m/s)
Solution;
L = inductance (Hm)
Hotations:
C= capacitance jF/m}
cos i= fe
MUF 8iN L = free space path loss (dB)
f; _ 25 MHz h= ft -=—— Solution:
MUF = Yeatiy Z d = distance from the antenna (km)
cos 0 cos 45. f = operating frequency (Miz)
MUF=35.35 MHz | (B1)(100 x 10°) US 1
ge P: = power transmitted (W)
The best frequency to use = OWF
n= = mois
20x408
ae
PAe Pa = power received (WW)
1
OWF = 0.B5MUF = (0,84)(35.35) n=0.99
Solution:
OWF = 30 Mrz
“The refractive index is 0,99.
Note: OWF = Optimum Working Frequency V, =100°% 107 mis L=32.4+20logd_, + 20logt,,,

«The best frequency to use is 30 MHz. uplink loss


-. The propagation velocity of a signal
is 100 x 10° mis. L, = 32-4 + 2010g(36000)+ 20l0g(2400)

2-8 2-5
The Question Bank Series —Communications Engineering (Volume 2 - Problems) Question Bank 2 - Radio Wave Propagation

L, =32.4491.134 67.60 PROBLEM 29 ” The transmission range is using


e=aso1 ™ Antenna heights of transmitting and
ionosphere transmission is 693 km.
L, =191.13 dB m
receiving antenna are found to be 20
meters and 10 meters respectively.
“The voltage induced at the feedpoint ih
Priaany= 10log| ar 70 dBm is 65.63 mV. What is the maximum distance of the
two antennas if tropospheric The antenna is beam 10° above the
transmission is used? ground for optimum transmission.
Pareana ‘2 Piyeeai atGry =
PROBLEM 28 Determine the transmission range, if
Piety= 70 dBm +100 dBi- 191.13 dB ionosphenc transmission is used and
A mobile radio system base station is Flotatinns: virtual height ts 200 km,
Pim = 21-13 dBm installed 50 meters above the ground
[_ (24134)
and transmits 150 W at 54 MHz. If the d= raiho horizon distance (mi)
Aine transmitting antenna gain is 5.17, how Fotations:
Fron) =1x107] 10: fy = transmitting antenna height (ft)
far will the receiving antenna be, if the
h, = receiving antenna height ({t)
' antenna height is 30 m and the received d = transmission range considering the curvature of
Poo = 771 HW field strength is 22 wim. the earth (km)
Solution:
a = earth's radius (kom)
“The received signal level of the h = wrtual height (km)
Hotations: d= (2h, + 2h,
antenna is 7.71 wW, fi = angle of transmission
= /2(20)(3.28) + j2(10)(3.28)
Eo = electric field strength of the transmitting Solution;
iO)
1A ag antenna (W) d=11.4548.1
The two half-wave dipole antennas are
Eg= Gece Redd strenci of the recenang antenna d=19.55 miles considering the curvature of the earth
positioned for maximum transmission {vim} vals)
4
Ga)
4

and reception of signal. If the input h, and h-= height of transmitting


and receiving ~, The two antennas are 19.55 miles
power at the feedpoint is 150 W, what is antennas above reflecting plane (m) apart.
the voltage induced at the feedpoint if d = distance of the two-antennas (m} d=2((ex70)| (1 57-0,175)— sin” isa)
the antennas are spaced1 km apart and P; = transmitted power (i)
operating
at 126 MHz? & = transmitting antenna power gain ah eed) d=1604 km
The antenna is beam 30° above the .. The transmission range considering
Solution: ground for optimum transmission,
Flotation: the earth's curvature is 1604 km.
Determine the transmission range, if
5, =, 30R-G lonespheric transmission ts used anc
V. = voltage indured (V) virtual height is 200 km Assuming, the ah) 10d
& = elecinc field strength (¥/m) z,= (20)(150)(5.17) earth is flat.
The signal propagates in fresh water
ka = effective length of the antenna (m) £, =152.53V
= transmitied power (W) with an average conductivity of 2.5 Sim.
Gy =transmatting antenna power gain Wes [ae Hotta: Determine the attenuation coefficient if
=A 5 ac }
the relative permittivity of the medium is
Solution: TR = transmission range (km) 65 and signal frequency is 3 kHz.
d= E.) 4zh,h, | h = virtual height (km)
Vy = hy)Ele ee
“(2 } (eoA J B= ange of transmission (degrees) Hlotwtions:

V, = {5.91%mys | =) d= | sas 4n(50 m)(30 m) |


Solution:
= attenuation coefiicient (Np/m)
we at
| 9 556pt |
\U me J
V, = 65.63 mV for flat earth approximation to = angular frequency (rads)
& = conductmity (5/m)
a 7a m d=153,4313'm
aah
tang

a ES, 7 .¥(30)(150)(1.64) “The antenna is 153,313 meters away


TR- 2(200) ,
1,000 from the mobile station. tan30
TR= 693 km

2-10 2-1/1
The Question Bank Series— Communications Engineering (Volume 2 ~ Problems) Question Bank 2 - Radio Wave Propagation

Solution; S Flotattons: att)


:10 3 ee

(a) First itis necessary to evaluate the rata ofme | (1.2)(40,000)" a= charactenslic impedance of free space (02)
What is the critical angle of the signal
when it passes from glass, with relative
o 25 &, = 99:34 mV/m 2, = characteristic of the unknown medium (£2)
ef O? penmittivity of 7.8, into-air?
wr 22(3000)(65)(6.854x10°7) e, = relative permitinity
for 1
where:
OE 8 Solution: Notations:
os ee D5
lay |(2x)(3000)(2.5)(47x107) f 250x410
cate Se He = critical angle (degrees)
.. The electric field strength received is Bz = relative permittivity of air
a@=P=172,020 ees Er = relative permittivity.of glass
99.34 mV/m.

“. The attenuation constant Is Solution:


172.02 x 10° Np/m. er
f
A VHF radio is to be established via the 8, =sin"' 4
ionosphere: Assuming the earth is flat “The relative permittivity is 2.51. Ey
ed 01 1 ie} with the critical frequency of & MHz, the f
4 microwave radio link is installed on a angle of elevation is 45°. Calculate the 8, = sin} is
hill 100 m high above the ground. The optimum working frequency ttt Bey 7.6
Mikey

link is-10 km from the ground station, is


A shortwave transmitter uses a skywave
set to transmit 10 W from a parabolic
lLotattons; propagation. What is the angle of
dish having a gain of 3000 over
incidence if the MUF is 19 MHz and the «, The critical angle of the signal Is
isotropic. Calculate the signal strength
critical frequency is 5. MHz? 74",
received if the height of the ground MUF = maximum usable frequency (Hz)
station is 10 m operating frequency at f= critical frequency (Hz)
250 MHz. 0 = incidence angle (degrees) Flotation:
hited
ha peek
Solution: MUF = maximum usable frequency (He) ATV broadcast station has a
Mota tients: transmitting antenna located 20:m
fe = enitical frequency (Hz)
MUF =f seca above the ground. What Is the height
E; = electric fleld strength of the transmitting = angle of incidence (degrees)
above the ground of the receiving
antenna (VV) MUF =f. sec(90—f) antenna 30. km from the transmitter?
Solution:
6a = electnc field strength of the recetving MUF =5 x 10°sec(S0-
45)
antenna (Vim)
hy and hy = height of transmitting and recelving
MUF = 7.1 MHz Mur = —c Histations:

antennas: above reflecting plane (nt) cosh,


For ionospheric transmission OWF is 15% d= radio horizon distance (km)
d = distance of the two-antennas (m} lower than MUF, thus
f 4

8, =coe hy = transmitting antenna height (m)


Pr = transmitted power (W)
G = transmitting antenna power gain
OFW = 0,85(MUF) = 6 MHz tal h. = receiving antenna height (m)

6, =cos =] Solution:
Solution “. The optimum working frequency is L. 19,
6 MHz.
£, = ,/30P,6
0,=74.7 d= A7h, +./17h,
et
&, = /(30)(10)(3000)
5, = 948.68 V
PROBLEM 35
An unknown medium wes used in
“The angle of incidence fs 74.7",
pio teste] 1
communication system, If the
(ae ) characteristic impedance of that medium me [30 ,/17(20)|
“laa |
wt

SA
is 150 ohms, what is the relative
1 17
permittivity?
he= 7.9m
2-12 2-13
The Question Bank Series — Communications Engineering (Volume 2 - Problems)

“. The height of the receiving antenna is


7.9 meters.

aed tie bey


What is the angle of incidence of the
radio wave signal as it moves from air to
glass, if the angle of refraction is 8.2°7

Flotations:

8), = angle of refraction (degrees)


QO. = angle of incidence (degrees)
E¢) = relative permittivity of air
Eo = felative permittivity of glass

solution:

sino, fe
sing, Er

sing, = (fn,

|
\
sin6, = (7a)1i
ens 1 ;
6,.= sin! i sin 8,2

8, = 2.93"
.. The angle of incidance is 2.93°.

Question Bank 3
SB ahr

2-14
The Question Bank Series— Communications Engmeering (Volume 4 - Problems)

”, The height of the receiving antenna is


7.9 meters. ‘

att]
What is the angle of incidence of the
radio wave signal as it moves from air to
glass, if the angle of refraction is 8.2°7

Hotaltons:

6, =angteof refraction (degrees)


02 = angle of inddence (degrees)
Ey = relative permittivity of air
Ee = relative permittivity of glass

Solution:

i
sinf jt
sings Ven

sing,=|
es |sin8,
|—*
Ven,

sind, =| , /——
qh eet
7a ee 8.2

8, on (Ve sin8.2

6, =2.93°
2. The angle of incidence is 2.93°.

Question Bank 3
ANTENNA
_

2-14
Question Bank 3— Antenna

PROBLEM 1 {= frequency (Hz)


‘| Two halfwave dipole antennas are
positioned for optimum transmission and Solution:
reception. Calculate the open-circuit =
voltage induced in one A/2 dipole when
fon . s n t
20 W of power at 250 MHz is radiated to
from another 1/2 dipole 100 km away.
3x10"
fe
aff
oeite
Rictattins:
a
Coy = 31.83'm
\= open circuit voltage at the feedpoint (Vv)
&= electric field strength (\/m)
~. The effective length Is 31.83 m,
Ly = effective length (m)
P; = radiated power (WW)
Gy = antenna power gain
PROBLEM 3
r= distance from (he antenna (m)
To minimize interference, a 500 MHz
Solution: dish needs to have a 1-degree
beamwidth. What is the corresponding
gain in dB?
Vi Elon
W= (0.314 mV/m)(0.38 m)
V= 119.32 pV Viotatins:

atiete :
G = power gain (dB)
0P.G
ea G, = 1.64
0 = dish diameter (im)
= wavelength (m)
_ /30(20)(1.64) i = beamwidth (degrees)
=0.314mVim
400 x 105 Solution:
c
Pisin 3x 10° mis 2
at 250
x 10° Hz
2m
G- 1olegé| 2
fy = 2 = 2 038m 2
i ™
G= 10log6{ “2*)
* The voltage induced at the feedpoint G=447 08
is 119.32 V.
where:
bat]I p- lek — 70K 39,
+ T
For an antenna ina form of a linear
‘onductor or an array of conductors,
Calculate the effective length at 3 MHz. -. The power gain Is 44.7 dB.

| Motisticns; a S|
A-dish designed for operation at 150
f. te = effective length (m) MHzis operated at twice that frequency.
f= Speed of light (m/s) By what factor do gain change?
4.= wavelength (m)
The Question Bank Series ~ Communications Engineering (Volume 2- Problems) Question Bank 3— Antenna

Hatations Vlatationa: Flotations: _axto?_Y


_ | 200x10° |
G = power gain (dB) i/D= aperture nuinber | = eflective length (im) oe dei
A, = wavelength (m) {= focal distance (m) c= speed of bght (m/s) Ag = 04179 m*
D = mouth diameter (m) + = wavelength (m)
Solution: $ = angular-aperiure (degrees) 1 {= frequency (Hz) «. The effective area of the antenna Is

ey
0.897 m’,
Solution: Solution:

eS
Goa ea at 2 x frequency, «=

Bo
|
A
a 0.25car( 8) dite) Rt)

al—
(sz)
Ga 1¥ =4 times i
Determine the gain of an antenna with
0.25 cat) — respect to a dipole if its gain with

ol—>
2
\ tespect to isotropic is 50 dBi,
fg = 9.09 m

of
-. The power gain increased 4 times.

oo
>
o
i

yop iy
where: Potetinns;

ae)
3) ey “The aperture number is 0.48. G__3x10'm/s
== = 2867mi G= gain referred to dipole (dBy)
What is the power gain in dB ofa f 10.5x10?Hz G= gain referred to isotropic antenna (dB)
uniformly illuminated parabolic reflector an -. The effective length is 9.09 m.
whose half-power beamwidth is 3 Solution:
What is the effective area of a Hertzian
degrees?
dipole antenna for unity efficiency at 3
MHz? PROBLEM 9 G(dBd)= G(dBi)-2.14 dB
Votatinas A ceriain antenna has a gain of 7 dB
G(dBd) = 50 dBi -2.14 dB
Plotatitnes:
with respect fo an isotropic radiator. G(dBd) = 47.86 dB
$= beamwidth (degrees) What is the effective area if il operates
G = power gain at 200 MHz?
Aa = effective area (m*) ” The gain of the antenna with respect
to dipole is 47.86 dB.
4. = wavelength (m)
Solution: Mutations:

Solution: ih) 140


_ 142 G= power gain (dB)
‘“e Ay, = 0.11912 Ave = antenna elective area (me?) If an antenna has a length of 0.125)
3.142 Ay, = 0.11 90100)
Ae = isotropic antenna effective area (m*) than what {s the radiation resistanca?
Jc Ay, = 1190 m?
4. = wavelength (m)
JG = 47,33 Solution: = Foninianes:
Gig = 10logG = 20109,/G = 20log(47,33) where:
G= fon, Rog = radiation resistance (2)
G=33.50B oS
.=—=————
= 100m
Sxi0 m/s
| = antenna length (m)
*F 3xt08Hz
Piss

~, The power gain is 33.5 dB. Aun = GAvso 4 = wavelength (m)


“ The effective area of a Hertzian dipole Avy = (5.01)(0.179)
Solution:
is 1190 m?. Ay, = 0.897 mi
PROBLEM 6
where: For antenna length not in excess of
In aperture antennas for microwave PROBLEM &
communications, what is the aperture AS (0.1251),
G= log" =5.01
number if the angular aperture is 55°? For an antenna in a form ofa linear 2

conductor, what is the effective length at 47 Ried ~790| +]


10.5 MHz? Abc = an AJ

3-5
The Question Bank Series —- Communications Engineering (Volume 2 - Problems) Question Bank 3—Antenna

Solution: Ve ifartpniag, Solution:


Resi ~ 790{ 2125)
A
Aun = 0.11932 A = signal wavelength (m) Tt Fs
Rog
= 12.34 2 f=[) [2 = 18
4A,
aan
4(310)
Agg =0.418(150}77 Solution:
f=C.91
”, The radiation resistance of the Agy = 2,677.5 me
antenna is 12.34 Q. where: 74 =360°
2. The reflector coupling factor Is 0.91.
_¢ 3xi0®m/s 377. = 34 (260 )=135°
PROBLEM 12 ~f -2xi08Hz Pry ye
What is the horizontal beamwidth of a a=150m .. The phase separation is 135 degrees.
pyramidal horn antenna if its width is Past ECE Hoard Problem
one wavelength? -. The effective area of the antenna is A TYRO installation for use with a C-
2677.5 m’. ete) band satellite (download frequency at 4
GHz), has a diameter of about 3.5
Past ECE Board Problem meters and an efficiency of 60%,
Holatrores: A400 feet antenna is to be operated at
dT | 6,200 KHz. Whatis the wavelength at Calculate its beamwidth,
0 = horizontal beamwidth of a pyramidal hon this frequency?
Past ECE Board Problem
{degrees} A helical antenna has the following
Potatoes:

w= wath of the pyramedal hom (mi) dimensions: pitch is 1/3 wavelength; Vista teva:
3, = wavelength (m] ib = beantwidth (deqrees)
diameter is 44 wavelength and turns is
27, What is the gain in dB? A = wavelength (m) D = dish diameter (rm)
Solution: Vi. = propagation velocity (m/s) 4. = signal wavelength (im)
Hotations: i= frequency (Hz)
Solution:
gente G = power gain (dB) Solution:
D = helix diameter (m) 7OR
N = umber of turns _V5__axiot = Di
$= pitch (m) ~ f 6200 kHz » — 10(0.075)
A = wavelength (m) 4=48.39.m Seay
b=15

oon)
Solution: -. The wavelength of the signal is
where!
48.39 m,
-. The horizontal beamwidth of a 2x08
= ato? = o.o75 m
pyramidal horn Is 80°. x
PROBLEM 17
Past ECE Board Problem “, The beamwidth of the antenna is
ed 11 G Sona 5) A microwave communications uses 1.4 degrees.
At 2 MHz, what is the effective area of Plane reflectors as passive repeaters.
the Hertzian dipole for unity efficiency? The diameter of the parabolic antenna is
G=19.2dB 18 feet while the effective area ts 310 PROBLEM 19
ft”. Determine the reflector coupling Past ECE Board Problem
«. The gain of the helix antenna is factor,
Hotateans: & A 500 KHz antenna radiates 500 Woof
19.2 dB. power. The same antenna produces a
Acs = effective area (m=) Flatations: field strength equal to 1.5 mV/m. Ifthe
A= wavelength (mn) power delivered by the antenna is
bei) Md [= reflector coupling factor increased to 1 KW) what would be the
c= speed af fight (m/s)
Past ECE Board Problem 0’ = diameter of the parabolle antenna (m) expected field intensity?
What is the phase separation of two A= antenna effective area (m=)
antennas 3/8 wavelengths apart?
eering (Volume 2 - Problems) Question Bank 3— Antenna
The Question Bank Series — Communications Engin
In watt, Solution: «The effective radiated power of the
Flotaticns: antenna is 375.18 W.

= = electric field strength (Vim)


ERP(J8W)=1Dio| yyiw
(Wy) | From P=FR
=

Py = radiated power (1H!) ERP(dBW)|


Then P x? PROBLEM 24
r= distance from the antenna {m) ERP(dBW) 1 Wee" al Increase in radiated power Past ECE Board Problem
4 The transmitting antenna has a gain of
Solution! =(3.4)'= 11.56 times
31:98} 10 dB; and power output of 6000 W.
ERP(dBW) =1 Wieg’'|® il | Determine the effective radiated power.
/. The radiated power increases 17.56
ERP =1577.61W times. Notations:
With P; = 500 W, + The effective radiated power is ERP =eflective radiated power ('W)
—=4.5 mVim 1577.61 W. PROBLEM 23 Gy; = antenna power gain (dB)
With 2P, = 1 kW (with constant r) Past ECE Board Problem P., = Input power (W)
, = SO). (2) | Determine the effective radiated power
a8) 1 ea ofan FM transmitter if the input power to Solution;

ope
Past ECE Board Problem the final amplifier is-370 W with 60%
What is the length in feet of an antenna efficiency if the field gain of the antenna ERP=P_G,
=(V2)(1.5)=2.12 mvim wire for 5 MHz ifthe antenna is 3,5 is 1.3, assuming 100% transmission line

$3
wave in length? efficiency. ERF =(6000)(10)= 60,000 W


«The electric field strength measured where |
when the power is increased to 1 kW Pleteitione: Hotties: G. = log" “sese =40
is 2.12 mV/m.
ERP = effective radiated power (W)

2
A= wavelength (m)
= propagation velacity (m/s) G=anlenna power gain -, The effective radiated power of the
ate) Way er {= frequency (Hz) 7 = efficiency antenna is 60,000 W,
Past ECE Board Problem
P= power (W)
What is the effective radiated power Solution:
(ERP) of a repeater with 250 W Solution: ai 1A ee)
transmitting power output, 3 dB feedline _ Vp _ 3xil 0° ERP Past ECE Board Problem
joss, 4 dB duplexer loss, 5 dB circulator f 5xi0® re
What Is the approximate length of a half
loss and feedline antenna gain of 20 G=(1.3)
4 =60 m= 196-6 ft Wave dipole antenna radiating at 6450
dB? kHz?
Length = 3.54
Plotertiias= Length = 3.5(196.8)= 688.6 ft
Flotations:

ERP = Effective Radiated Power (W)


“The length of the antenna is 688.8 ft. n= 60% / =antenna length (rm)
P,, = input power (¥)
Ly = total loss (dB) 4. = wavelength (m)
= total antenna gain (dB) c= Saeed of light (m/s)
PROBLEM 22 f= frequency (Hz)
Past ECE Board Problem Th= Fut x100% p= Fa 100%
Solution:
Ifthe antenna current increases 3.4
Fy ee
Solution:
times, how much does the radiated Pout = MP Pa = Pou
ERP(dBW) =P,,(dBW)-L; +G; power increase’? =(0.6)(370W) P, =(1)(222W)
250 L= c _ 3x10'm/s
ERP(dBW)= 10log—~— 12+20
Vatations:
Fig = 222 W P, = 222W f 6450 kHz
=46.51m
ERP =31.98 dBW ERP = PG = (222 w)(1.3)°
P = power radiated (W) 4, 46-51
| = antenna current (A)
ERP = 375.18 W 9 pra 2
R=resistance (£2)
f= 23.26 m= 76.26 ft.

3-8
Question Bank 3— Antenna
(Volume & - Problems)
The Question Bank Series— Communications Engineering
Solution: Vatu tions: PROBLEM 31
with end effect (5% shorter), Past ECE Board Problem
Gas 18
f= 0.95(76.28) k
* 150.55 MHz
Gia = power in dB What is the gain, in dB of a helical
fats = frequency (GHz) antenna with diameter of W/3, 10 turns at
€=72.5 ft. 4 =1.99 m=6.54 fl De = dish diameter: (leet) ptch of Wd, used at 100 MHz?
The antenna length considering end
a 6.54 ft = 1.635 ft Solution: Potaticms
effact is 72.5 ft.
with end effect (5% shorter),
G(dB)=7.54 20log f,),. + 20logD, G = power gain (dB)
= 0,95 (1.635 ft) OD = helex diameter (rm)
PROBLEM 26 G(dB)= 7.54 20log1'8+ 20log6
t= Roo te N= number of turns
Past ECE Board Problem G(dB)
= 28.17 S = pitch (m)
Determine the approximate length of a © The antenna length considering end
simple whip quarter wave antenna at }.= wavelength (m)
effect is 1.55 ft. “The power gain of the parabolic
UHF frequency of 450.25 MHz. antenna in dB is 28.17. Sclution:
PROBLEM 28
Notations: Pens)
Past ECE Board Problem G[dB) =1010g15| x— | eo}
Find the approximate width ofa gin) 14a KAS WA?
1 = antenna lengli (m)
pyramidal horn antenna that will operate Past ECE Board Problem fay (10){ )
A. = wavelength (rm)
at 10,000 MHz intended to have a A dipole antenna requires to be feed
¢ = speed of fight {m/s} beamwidth of 10 dearees. with 20 KW of power to produce a given G(dB) =10i0g15) x]
a)
| —
t 4

f= frequency (Hz) signal strength to a particular distant


th A,

PLadéfienis point. Ifthe addition of a reflector makes


Solution: the same field strength available with an G =16.10B
8 = horizontal beamwidth of a pyramidal horn input power of 17 kW. Whatis the gain
_t__3xi0'm/s (degrees) in dB obtained by the use of the Where:
~f 450.25 MHz w= width af the pyranidal horn (m), reflector? (Gain referred to this
particular dipole), ==‘ \N=10,S=- a,
4,= 0.666m A. = wavelength (m) 3 4
222,18
HMiptations:
poh 2.16 ft. Solution: -. The power gain of the helical antenna
4 4 in dB is 16.7.
6 = power gain (dB)
f=0.55 ft
ERP = effective radiated power |W]
with end effect (5% shorter),
f= 095/055) PROBLEM 32
Solution
l=0.52:%. Past ECE Board Problem
ol 4 x 10° |
Ree) _ ERP. The frequency of operation of a dipole
“The antenna length considering end | 10,000 x 10° | 1 2okw °°? 1kW antenna cut to-length of 3.4m.
effact 0.52 ft.
10 ( ERP: |
Floriticery
w=0.24m
itt):
] ha ea
w= 240m G, - ERPS), (b1aikW | = antenna length {m)
Past ECE Board Problem
What is the length of a simple quarter 20 kW J A= wavelenath (m)
“The width of the pyramidal horn
wave antenna using a VHF frequency of antenna is 24 cm.
in
# c= speed of light (m/s)
150.55 MHz? G(dB) = 10log) f= frequency (Hz)
\,

G(dB) = 2.6 dB (dB) Solution:


Vlatiationa: PROBLEM 29
A é
Past ECE Board Problem “The power of the antenna in dB is 2.6. (sibrgae
{= antenna lersth {m)
Determine the gain ofa 6 ft. parabolic
2 2
4 = wavelength (m}
dish operating at 1800 MHz. h=2(3.4)=68=5
C
c= speed of light (m/s)
f= frequency (Hz)
3-10 3-111
The Question Bank Series - Communications Engineering (Volume 2 - Problems)
Question Bank 3 - Antenna

_¢ 23 x10"mis Solution: Viotateonis: PROBLEM 38


“68 68 dB gain = 20 ibeiee The output power of an Fm transmitter
f=44.12 MHz \ Rew = radiation resistance (02) is 25 W. Ifthe gain of the antenna
Ry = ohmic resistance (€2) referred to isotropic is 50, what is the
dB gain =20 log 1 = antenna efficiency
-. The operating frequency of the dipole ERP of the antenna?
antenna is 44.12 MHz, dBgain =-6
Solution: Motationa:
where:
he ate Vo = 41 pV SE EIRP = elfective Isotropic Radiated Power (dBW)
Rag ta
Past ECE Board Problem V=2 pV P,, = power input (W')
What is the effective radiated power Pie &. = gain with respect to Isotropic antenna
(ERP) of ajrepeater with 200 W .. The power gain of the receiving q red EAP = Effective Radiated Power (dBW)
transmitting power output, 2 dB feedline antenna Is -6 dB.
Reo tie ses]
a f 1
loss, 2 dB duplexer loss, 3.08 circulator Solution:
loss. and feedline antenna gain of 10 and J 0.96
dB? PROBLEM 35 Ry=30
EIRP =P,G,, EN) = 1250 W
Hlotations:
The actual length of the Marconi
antenna used in AM transmitter is 34m. “The ohmic resistance of the antenna EIRP
Is37. : EIRP
f = 10log———
og +W = 30.97 dBW
What is the operating frequency of the
ERP = effective radiated power (W}
transmitter, if the velocity factor is 0.9? ERP=EIRP — 2.1408 = 30.97 — 2,14
Py = power input at the feedpoint (W)
P, = transmitter power output (W) PROBLEM 37 ERP = 28.83 dBW
Flotutiona What is the gain of a halfwave dipole
Salution: -, The effective radiated power is
antenna if the antenna has an efficiency
L, = antenna length-(m) 28.83 dBW.
of 92%?
ERP =(F,,)(G) A= wavelength (m)
ERP = (39.8)(10)=398 W Va = phase velocity (mis) Hatations: hate)
Ra et
where {= frequency (Hz)
0 = directivity gain (cB) A satellite tracking device uses a helical
Pi (dBW) = Pd BW) — total losses antenna that is operating at 1.2 GHz. If
Solution = 2.14.08 for half-wave dipole
P.(dBW) = 101ag 22° (24243) ie a
A G= power gain (absolute value) the optimum diameter of the antenna is
8 em and 10 tums, what is the gain of
+) = efficiency
Po (dBW) = 23-7 the antenna?
4= 4L=4(34)=136m
F. (dBW) = 16 dBW = 39.8 W Solution:
G=10dB=10 vee Poatatiins:
ST ebay D=214d6
-. The effective radiated power is 0.9(3xi0° D(dB)=10logD 6 = power gain (dB)
358 W. pei, O80")136
os wz 0 = helix diameter (am)
41
D(dB) 2.44
D=] og" = log =|99"| aa== |= 1.64 N= number
of tums
$= plich (cm)
PROBLEM 34 .. The operating frequency of the
transmitter is 1.98 MHz. G=nD=(0.92)(1.64) 1. = wavelength (cm)
Past ECE Board Problem { = frequency (Hz)
G=1.51
Determine the dB gain of a of receiving = speed of light (ms)
antenna which delivers a microvolt PROBLEM 36 © The power gain of the antenna is
signal to a transmission line over that of Solution:
What is the chmic resistance of the 1.51.
an antenna that delivers a 2 microvolt
signal under identical circumstances. antenna, if the efficiency of the antenna
is 6.96 and the radiation resistance is 72
1,
The Question Bank Series — Communications Engineering (Volume 2 - Problems) Question Hank 3= Antenna

«The beamwidth ofa helical antenna is f=requency (Hz) et he |


oa 50) (roo) 28.9 degrees. c= speed of light (m/s) Calculate the actual langth of a half
G = antenna power gain (unitless} wave dipole operating at a frequency of
G=15,78 dBi TA a 67.25 MHz,
Solution:
4 certain helical antenna with twelve
turns is used to operate at a frequency Votatious:
of 1.2 GHz. What is the total length of
the antenna? L = antenna length (m)
2. = wavelength (m)
Tatai Was
Solution:
L =anterina length (cm)
©, The gain of the helical antenna with
respect to isotropic antenna is
N= number of tums
5 = pitch (em)
Lam =5 (0.95)
15,76 dB. }. = wavelength (crn)
1 = frequency (Hz)
ties ~ 256 (0.95)-2.12m
a) c= speed of light (m/s) where:
~, The parabolic dish diameter is A
A helical shape antenna with eight turns Solution: ee! eae 446m
is used to track a satellite that Is
480 cm. fF 67.25x10
operating at 1.2 GHz. Ifthe spacing for L=NS
the antenna is: 10 cm, what is the . The actual length of the antenna is
beamwidth of the antenna? Ni _ (12)(25) ei) fH} 2.12m.
Eg erent Calculate the operating frequency of a
half wave dipole ifthe actual length is
Motations: 3.5 meter. PROBLEM 45
An antenna with a radiation resistance
G = power gain (dB)
of 5022 and a loss resistance of 342,
D= helix diameter (cm) TMotatigyis:
receives a 150 watts of power from a
N= number of tums transmitter. How much power is radiated
§= pitch (cm) L = antenna length (m) into space?
2. = wavelength (em) A, = wavelength (im)
1 = frequency (Hz)
6 = beamwidth (degrees) Solution: Hiotation:
.. The antenna length is 75 cm.
Solution: Lam = 5 (0.95) Pou = power radiated (W)
a1H eg Pe, = input power (1)
ik = Peat = (2)(3-5) 1 = aMmenna efficiency
An earth satellite station needed an 0.95 0.85
‘c= rar mi Rou = radiation resistance ((2)
antenna thal can amplify 2500 times
Ar = total resistance (Q)}
bigger than the input signal. If the foo 3x10"
engineers assigned agreed to build an Ay = ohmic resistance (£2)
& Td?
antenna with parabolic reflector, anc
operates at 3.4 GHz. what is its
f=40.7 MHz Solution:
diameter?
Geo at? “The operating frequency of the
Pas = TP,
f 12x10! antenna is 40.7 MHz.
P., = 0.9434(150W)
A= 250m Plotintiness?
Ply =141-51W
De= -2 _a0m
ab

4 A = wavelengih (em)

3-14 3-15
The Question Bank Series — Communications Engineering (Volume 2 - Problems) Question Bank 3— Antenna

where: Vlotarttoms; P, =EIRP=P,G “The effective isotropic radiated


= Rnd
ty = R
7] = antenna efficiency
P, = 25W(1.64) = 41 W power is 16.72 dBW.

Fag = tadiation resistance (£2)


== ag BO Ay = ohmic resistance (€2)
.. The electric fleld strength is i)
V"RL+R, 5013 2.44 mVim,.
Determine the actual length ofa
= 0.9434 Solution: Marconi antenna operating ata
ti) 1h frequency of 1625 kHz.
.. The power radiated into space is
147.67 W. y= — Pest 100% A transmitter supplied 125 watts of
rad 7 power to.an antenna with an efficiency Plovtcadivana:
of 95%, What Is the power density
n= {= \xt00% measured at a distance of 30 km fram
PROBLEM 46 F220 the antenna In the direction of maximum
L = amenna lengtty (in)
An antenna with a radiation resistance 1) =0.973x 100% radiation? Assume the transmission line 4. = wevelenglh (m)}
of 750 and a loss resistance of 512, 1 =97.3% is lossless and perfectly matched. c =speed of light (mis)
receives a 150 watts of power from a
transmitter, How much power is Solution:
“The efficiency of the antenna is
wasted? Motutions;
97.3 4.
b= (0.98) “ [= |(o.s)
2 = power density (Wim=)
HHoetatiaus:
ait de EIRP = Efiectve Isotropic Radiated Power (W) 3x10"
L=>=— (0.95
r= distance from the antenna (m) : 416250) }
Po = power dissipated (W) A half-wave dipole with a gain of 2.14
P,, = input power (W) dBi is fed, by means of lossless,
Solution:
L, = 43.85.m
matched line, by a 25 watts transmitter.
1) = antenna efficiency
What is the electric field strength of the
Rog = radiation resistance (£2) signal measured 15 km from the p= EIRP .. The actual length of the antenna is
43.85 m.
Rr = total resistance (£2) antenna in free space in the direction of 4or*
Ra = ohmic resistance (£2) maximum radiation? _ -425(0.95)
=10.5 nWim? a faa eyes
Solution: E 4x(30x10")
Votateane: 4 parabolic antenna used in the satellite
. The power density is 10.6 nWim’. earth station has beamwidth of 3°. If the
Pa We (i ie n)P,
P; = EIRP = Effective Isotropic Radiated Power (\) antenna |s operating at a frequency of
py = (1- 0.9375)150 P., = input power (W) 14 GHz, what is the minimum diameter
of the antenna?
Py =9.375 W G = antenna power gain (dB) PROBLEM 50
where: E = electric field strength (Vim) A dipole antenna is designed to have
r= distance from the antenna (m) 94% efficiency. Determine the EIRP of
7 — Rr this antenna if itis fed by a 50 watts
Flatatiens:

” Raa +Ry Solution; transmitter. @ = beammiih (degrees)


i= 75
= 0.9375 4. = wavelengtiy (m)
7545 f= frequency (Hz)
Motations:
c= speed af fight (ris)
.. The power wasted is 9.375 W.
EIRP = Effective Isotropic Radiated Power (dBW)

enfants
Solution:
Pate): ]aa eee Solution:
What is the efficiency of the antenna if EIRPane = 100g] ONE 8)
the radiation resistance is 72 © and the
loss resistance is 2027
For half-wave dipole,
G=2.14 dBi = 1.64 EIRP... = 16.72 dBW D =70( eg© \=70|
70 910
34x10")_| .=0.5m

3-16 3-17
Question Bank G=— Antenna
ering (Volume 2 - Problems)
The Question Bank Series - Communications Engine
Salution:
Tlatutions: Galn relative to Ssotropié antenna
«The minimum dish diameter Is 0.5 m. Gq —-68 dB + 30 dBi — 24 dBi
G= power gain referred to dipole (BD)
Aen =kA, eae Gain relative to half wave dipole antenna
, 4) G= power gain relerred to isotropic (dBi)
PROBLEM 53 Py = input power-of dipole antenna (W) Gigy =24 GBI-2,14 GBI

A parabolic antenna used inthe satellite Ay 071 P2= input power of Yagi antenna (W) Gyye = 21.86 dBd
earth station has beamwidth of 3°. What
is the gain of the antenna if the antenna Solution: “. The antenna power gain referred to
afficiancy is 85%? Aw = 4.95 mi? dipola is 21.86.

“The effective area of the parabolic eT ey


FTotantrems: antenna is 4.95 m’.
(40
Gain =10)09| *| A Yagi antenna with front-to-back ratio
G = anterina power gain (unitless) of 10 cB radiates power of 10 watts in
= beamwidth (degrees) PROBLEM 55: Gyan = 3 GBD the opposite direction. Determine the
A. = wavelength (m) power output of the transmitter, ifihe
The signal field strength 10 km from the Gig = Gia5 + 2-14 0B
D = dish diameter (rm) gain of the antenna is 10 dBi (assume
Yagi antenna in the optimum direction is Gy = 3 42:14 =5.14 dBi lossless transmission line and perfectly
{2mVim, and 180.degrees from the
Solution: matched).
optimum direction at the same distance
the field strength is reduced to 2 mV/m. /. The power gain of the antenna with

t= 70{ +) equation | What is the front-to-back ratio of the respect to isotropic antenna is
5.14 dB. Voatattons:
Dp antenna?
D 2
F/B= front-to-back ratio
G= “(2 equation 2 PROBLEM 37 P= antenna radiated power (WW)
Hotatinns:
EIRP = Effective Isotropic Radiated Power (iW)
Substitute (1) in (2) Two antennas-are fed with equal power:
F/B = front-to-back ratio P,, = pewer input at the feedpoint (W)
lf the power density measured of the
=5373 E = electric field strength (Vim) antenna under test is 2 Wim? and 8
Gar (2) =" e Wim? for the standard antenna. What
Solution:
Solution: is the gain of the antenna -under test

amt
The actual gain is (Pp

G = (5373)(0.85) = 4567
referred to dipole if the gain of the
standard antenna is 40 dBi?
(F).,
F, wx 10log) He
=|

ate
Fe R
«The actual antenna power gain is i =| Ca)s.
4567. Pntativnss Fe=Pylog™ =Sanngetile|
(Oleg Lia)
G = antenna power gain Pe =100 Watts
Ra er (E) =15.6 dB fl = power density (W/m?) EIRP=P.S
A parabolic antenna used in the satellite _ EIRP
\B AB
Solution: P= =
earth station has beamwidih of 3°,
Calculate the effective area of a 3m . The front-to-back ratio Is 15.6 dB.
(100) _
dish with an illumination efficiency of 0.7 ' G., = 10log* Part 10 =10 Watts
al 3 GHz is
ade)
4 tT Note
Gain relative to standard antenna
A ‘Yagi antenna must be fed with & KW The output power of the transmitter is caval {9 the input
Flotations: of power to produce the same field S.= 10io9| hue2 power-althe feedpoint of the antenna, provided that the
transmission line is hossless.
strength at a given point as that ofa
Aer = effective area of the parabolic dish (m?} half-wave dipole antenna fed with 10 kW G,,=-6 dB
A.= surface area of the parabolic dish (im?) of power. Determine the gain of the Yagi “The output power of the transmitter
k = illumination efficiency antenna relative to isotropic, is 10 W,

3-18 2-13
The Question Bank Series — Communications Engineering (Volume 2 - Problems) Question Bank 3- Antenna

ee) Pa =(1-n)F,, itd] yd “The effective length of the antenna is


The power radiatéd on a dipole is 2 W. If 100m.
FP, =(1-0.7)6 KW A vertical whip antenna is located in
the input power to the antenna is 2.6 W field strength of 7 mVim. lf the effective
and the current fed is 2A, calculate the P= 1.5 KW
length of the antenna is 1m. Calculate PROBLEM 64
ohmic resistance of the dipole. the Voltage induced into the antenna.
Note A haltwave dipole requires to be fed
Power lossiin the antenna Is equal to the power with 10 kV of power to produce 2 mV/m
Hota ThoKs: dissipated by the ohmic resistance. Hotatiins: signal strength 4 km from the antenna.
The field strength is measured again
Pp = power dissipated (WW) +, The power loss of the antenna is e = voltage induced (V) and the result is the same, but this time
Pra = radiated power (W) 1.5 kW. a reflector is added to the antenna and
€ = electric field strength (¥/m)
Py, = Input power (W) Ley = effective length (m) the input power is only 3 KW. Whatis
| = input current (A) the gain of the antenna referred to
PROBLEM 61 isotropic with the additional reflector?
Ay = ohmic reststance (02) Solution:
A half-wave dipole is designed ta
Solution: intercept maximum power at 300 MHz.
Calculate the effective aperture ofa HHetatians;
half-wave dipole:
P= Pad * Po G = antenna power gain (dB)
Py =F Fiea P, = input power of the reference antenna (W)
Hotatrors: P. = input power of the standard antenna (W)
FR, = Fy Ea
= Pa To Fad
2. The voltage induced is 7 mV.
G = antenna power gain (unitless) Solution:
Aw= dipole antenna effective area (m-)
26-2 450 Aco= isotropic antenna etfective area (m*} bea 0) 1) Gas, =t0loa 5
4, = wavelength (im) An aerial thatis mounted vertically is
operating at 3 MHz is 100 m high
¢: The ohmic resistance of the antenna Solution:
(assume that the ground is a perfect Gy, =10log| (2)- 5.23. dB,
is 0.15 ©,
conductor). tt carries a current that
vanes linearly from a maximum of 10.4 Gg =Gyp, +2.14 dB
athe) AFL at the base to 0 A to the top. Determine Gyq = 5:23
+ 2,14 =7.37 GBI
| the effective height of the aerial,
The power supplied to the antenna is §
kW. IF the efficiency of the dipole ts 0.7, “ The antenna power gain with respect
determine the power loss of the to isotropic is 7.37 dB.
Hovations:
antenna. aia
nals
|

boy = actual keagth (m) PROBLEM 65


A, =0.13 m* Irean = mean qurrent (A)
y

Notations:
|= maximum current (A) An artenna with parabolic dish reflector
where: has a beamwidth of 2° and a gain of 30
Ly = effective length (m)
Po = power dissipated (W) e dBi. Calculate the power density ata
Pru = radiated power (W) Ais = ax Salution: range of 5 km at an angle of 1° to the
Pn = input power (W) direction of maximum radiation if the
G=1.64
1) = antenna efficiency powe: radiated is 1.8 W,
_¢ 3x0! Dey iranh = Lon!

Solution: f 300x107 Cy en =L phy togan =~100/ 705 ==50 m Plotatiinn

Py Is Fy = Fiod .. The effective area of the dipole


Since the ground is a perfect conductor
where:
antenna Is 0.13 m’. Ge) = antenna referred to isotropic
Lo, = 2(50) = 100 m fo = power density (Wim?)
Pa = NP,

3-211
The Question Bank Series — Communications Engineering (Volume 2 - Problems) Question Bank 3- Antenna

Solution: 1 ey
A helical antenna is used to track
Gyq = 30 ABI satellites in space.If the antenna
Gx, =10logG consists of 15 turns, spacing 15 an), and
diameter 19.1.m, whatis the operating
G=1000 frequency of the antenna to transmit the
EIRP 1.8(1000) signal efficiently? fy
21.5
Zz

-G, = 1000) —— | = 45141.6


ante éa(SxTOey : | 32 }
= srt Mita tions! Gyyy = 10l09(45141.6)
Giyp = 46:55 dB
at 1 degree the gain Is 3 dB below § = pitch or spacing (em)
its maximum, therefore 2. = wavelength (em) . The gain of the antenna when the
= 0.5(5.73) 7 = frequency (Hz) frequency is changed
to 21.5 GHz is
c= speed af fight (m/s) 46.55 dBi.
p=2000"
mi Solution.
The power density is 2.86 u.Wim*.
c_ 3xiof
a 0.8
PROBLEM 66 f= 500 MHz
A ‘half-wave dipole antenna is used to where:
operate at54 MHz. Considering end
effect, whatis the effective area of the
antenna? i. =S(4)=15(4) = 60 om =0.6 m

“The operating frequency of the


Hosea ccnes: antenna is 500 MHz,

G = antenna power gain


Aue = dipale antenna effective area (rm?) ad]
A BET]
Ave = istropic- antenna effective area (rm)
A satellite earth station uses an antenna
A= Wavelength (im) with parabolic reflector that nasa gain of
30 dBi at 3.2 GHz. Whatis its gain ifthe
Solution: operating frequency is changed to 21.5
GHz, assuming the other parameters
G= Aon remain the same?
‘ae
G=1.64 for half-wave dipole
Hlotert mes:
considering end effect
G = antenna power gain (dB)
jets = 5 —— 5 6:28 'm i= frequency (Hz)
fact 54x10" OD = dish diameter (m)
=6=-—
2 1.44
528° C= speed of ight (m/s)
Aun 4n ( an
Solution:
Ay, = 3.64 me 2

G= (|=)
>. The effective area is 3.64 m’, c #

3-22
The Question Bank Series - Communications Engineering (Volume 2 - Problems)

Votes

lh
a
Bank 4

3-24
Question Bank 4 - Noise

PROBLEM | NF =nolse factor (absolute value}


The signal-to-noise ratio is 30 dB at the
Solution:
input'te. an amplifier and 27.3 dB at the
NF, —1 i" NE -1
output, What is the noise temperature of NF =NF, +
the amplifier? G, GG.
- §=4 5-1
NF=3 a (10)(20)
Flotation:

NF =3.32
F = noise figure (dB) F=(10logNF
T= equivalent noe temperature (K)
Ti. = standard noise room temperature (290 K)
F=10log(5:2) =5.2 dB
NF = noise factor (absolute value)
-. The total noise figure is 6.2 dB.
Solution:

PROBLEM 3
{= ) {Ss
ln). ae An amplifier operating over a 4-MHz
bardwidth has a 100-0 input resistance,
F=(30-dB-27,3 dB) It is operating at 27°C, has a vollage
F=27d8 gain of 200 and has input signal of5
F=10logNF Vine, Determine rms output noise
signal?
DNF =o E
Wisteniitnea:
NF= oo 2?
Ven = rms noise voltage {¥)_
NF =1.86
k = Boltzmann ‘constant = 1.38x 109 J/K:
“Ts =T, (NF —-1) T= holse temperature (kK)
T, = 290(1.86-1) BW = effective noise bandwedth (Hz)
ose f= equivalent noise resistance (£2)

_ Solution:
Hl. +, The equivalent noise temperature is
«2a K.
Vous = AV; = (200)(2.57x10'°)
Bynow Vig = 5 15x10 7V = 515 pV
A three-stage amplifier has the following where:
fowergains and noise figures (as ratios, \,, = (4RTBWR,,
not in decibels) for each stage: Stage 1,
power gain = 10, noise figure = 3; Stage = [4{1 38x10 )(27 +273)(4x10*)(100)
2, power gain = 20, noise figure = 4;
‘Stage 3, power gain = 30, noise figure = V,, = 2.57x10%V = 2.57 lV
5, Calculate tne total noise figure.
“. The output ns noise voltage ts
515 pV.
{Mola Viana:

; a = power gain (absolute value)


F= equivalent noise fiqure (dB)
The Question Bank Series — Communications Engineering (Volume 2 - Problems) Question Bank 4 - Noise

ad bed) 794) ag _ (BkQ)(20 ka) Viotatines:

Determine the noise current fora diode Two resistors, 5kQ.and 20 KO are at eT 5k +20 KO
NF = noise factor (absolute value}
with a forward bias of 1 miA overa 100 27°C, Calculate the thermal noise R,=4000 0 S/N = signal-to-noise ratio
kHz bandwidth? voltage fora 10-KHz bandwidth if they
.. The thermal noise voltage is 0.84 iV.
are in series,
Solution:

Flotation:
Vota CREM:
ed 00) 13 Fide) =(= -(§)
A npat AN output
I= rms shot noise current for a diode (A) Calculate the overall noise figure
¥, =rms thermal noise voltage [V). referred to the input ofa mixer stage ‘s
lie = direct diode current (A) 15- ¢] - 60
|, = reverse saturation current (A) k = Boltzmann constant = 1,38 x 107 J/K that has a noise figure of 15 dB N inet
preceded by an amplifier that has a
ge = charge
of an electron = 1.609x 10°17C T =nolse temperature (K)
BW = noise bandwadth (Hz) BW = effective noise bandwidth (Hz) noise figure of6 dB and an available (§) =15+60=75 dB
Ry = equivalent nolse resistance (£2) power gain of 20. dB. N Jaca
Solution: Ss {s
Solution: | =1oa| =|
Piotietiuna: VN ie iN
hn = V2llue + 21,080 V, =,/4kTBWRY F = noise figure (dB) [F =| (SS | -are227766
|, = J2(txt0°*)(1.609x10-"*)(100x10°) & = amplifier gain (absolute value} Nou 10
NF = nose facior (absolute value)
= 0.0057 pA -. The input equivalent signal-to-noise
V, = 2.035 iv
Solution: ratio Is 75 dB or 31622776.6.
-. The noise current generated by the where:
diode is 0.0057 pA. R,=5 k+20 k= 25 kn
NF = NF, + ac 4
ae Ma
it) “. The thermal noise voltage is 31,62 -1 What is the noise level ina voice
2.035 pV. NF =3.98 + ae channel if the S/N is 71.25 dB? Express
A transistor amplifier has a measured noise level in pWWpO?
S/N power of 10 at its Input and 5 at its NF=4.29
output. Calculate the transistor's NF. PROBLEM 7 In dB:
Floatation:
Calculate the thermal noise voltage of F=10:log NF = 10 lng 4:29
Flat tiens: the two resistors, 5 kQ and 20 kf) at F=6:32 d8 Pep = total noise level in picowalt psopharnetric
27°C if they are connected in parallel. where: {p¥ip0)
NF = noise factor (absolute value}
4 F, 6 = 3.98
NF, =log™'—! =log-'——
S/N = signal-to-noise ratio (absolute value) Solution:
HHutationa: ee to 40
Solution:
Va = rms noise voltage (\V) NF, = log’ bo" = 31.62
k= Boltzmann constant = 1.38.0 10%9 NK Prop = 0.5610g"'

a
s\ 20
T =nojse temperature (K} G=loq —=100
ea li, _ 10 BW = effective noise bandwidth (Hz) 10
5: += equivalent noise resistance (C}) Pe = o.seiog'| a
-. The overall noise figure is 6.32 dB.
‘out
NF=2 Solution:
PROBLEM 9
-. The transistor's NF is 2.
What is the input equivalent signal-to- «. The noise level in pWpoO Is 42,
Vp = y/4(1.38x107°)(300)(10x10")(4 ker) nolse ratio for a receiver with a nolse
figure of 15 dB and an output signal-to-
Vj = 0.81 pV folse ratio of 60 dB?

A4
The Question Bank Series —Communications Engineering (Volume 2 - Problems) Question Bank 4 -'Noise

at) 13 PROBLEM 13 a1 ait 18)


Ifthe occupied baseband of white noise What is the overall signal-to-noise ratio |fall the stages in a tandem circuit have Determine the effective noise
test signals 60 —4028 KHz, determine in dB ofa tandem connection with the identical signal-to-noise of 15:08, then temperature of an antenna system at
the bandwidth ratio. follawing individual signal-to-noise ratio: what is the overall signal-to-noise ratio the far end of the feedline ifthe effective
First stage = 10 dB) second stage = 15 in dB of the connection when there are 5 sky noise temperature is 200 K and
dB and third stage = 20 dB? | | stages? antenna and feediine inss is 10:
Fotations:

BWA = bandwidth ratio PilaLitt tenes: Hinletinna?


Voice channel bandwidth = 3,1 kHz Hojations:

; S/N = signal-to-noise ratio (dB) 7, = elective noise temperature af antenna and


Solution: Ny = overall nose.in tandem connection M = number of Greults in tandem connection feeding, relerenced to receiver antenna (K}
5/N= signal-ta-nolse ratio (dB) = loss in feciline and antenna (absolute value}
BWR = 10log Solution: Tay = Chective sky temperature (K)
| eas baseband of white noise test signal Solution: i F
volta channel bandwidth ‘Overall S =| = -Ma, Solution:
5 5 yr N \ de

BWR = 10leg{ a} ni—(overall) ) =10log—


ome
31
a Ss
Overall NS 15-10log M _ (L=1)280 + Tay
BWR =31.07 dB s
—[overall) = 10log)
(esi
———— ¢
Be L
5s
nt ) 9 0142s | Overall N =15-10log5 ae (io —1)290 + 200
“The bandwidth ratio is 31.07 dB. 1 10
= (overall) - 8.48 dB ‘Overall * -8.01 dB T, = 281K
PROBLEM 12 where:
+ The overall signal-to-noise ratio is “The effective noise temperature is
Calculate the equivalent noise 281 K.
6.01 dB.
temperature ofa receiver with a noise N,=N, +N, +N,
figure of 1.5 dB. N, = 0.18 +0.032S + 0.018
PROBLEM 15 ate
N,=0:142S
Mlatations: In psophometrie noise measurement, Past ECE Board Problem
=)§ =10dB
= 10log=5 what is the psophometric noise power in An amplifier operating over a 2 MHz
F = noise figure (dB) (a) man | pWo ifthe psophometer reads 10 mV af bandwidth has an 80 ohms input
Ta = equivalent noise temperature (K) Ss | foise voltage? resistance. It is operating at 27 degrees
= —_ = 945 Celsius, has a voltage gain of 200 and
NF = noise factor (absolute value}
"log 8 a an input signal of 6 1Vj4.. Calculate the
Solution: 2») = 45dB= 10log—5
s Flatatum: output ms noise.
& oN,
5 ‘pWa = plcowatt psophometric noise power
NF =log™ 5 N; =eloge = 0.032S
] Tishiateiialt:

Solution:
Va = rms noise voltage (\V)
Flog’ =1. 44 (s=| =10log—§
Tia rest
a, s er =20dB
pWp= (psophometric V}* xia
6000
k =Boltzmann constant = 1,48 % 107 /K
T =noise temperature (K)
T, = 290(1.41- |, =< = 0018 BW = effective noise bandwidth (Hi)
1) = 119.64 K (10x10° yy;
log” 3} PWp= x10 =0.167 x 10°""
R. = equivalent noise'resistance (2)
.. The equivalent noise temperature is 600
“. The overall signal-to-noise ratio is Solution!
119.64.
8.48 dB. “The psophometric noise power in
pWp is 0.167x 10°. For input noise voltage,
Vj = /4KTEWR,,

4-7
The Question Bank Series —Communications Engineering (Volume 2 - Problems) Question Bank 4 - Noise

Tota ties: Flotation:


Vn= \/4(1.38x10* )(300)(2x10")(80)
Va= 1,63 nV F = noise figure (1B) TLP = test level point (dB)
NF = noise factor (absolute value} dim = dB above reference noise (€ message)
For the output noise voltage, _ 12 a
S/N = signal-to-noise ratio (absolute value) i =0.16
Vay = AVq = (200)(1.63 :V) Solution: I 3 22 a
Vou = 325.6 nV Solution:
” The diode current is 398 mA.
TLP =dBrnC-—dBnco
©. The output rms noise voltage is
325.6 iV. F=10 log NF dBmCO=dBmC-TLP
F=10 log 5 dBmCO = 14-(-4) at)
TA 3 ee:
F=6.9dB dBmCO = 18 damco
di)
1 3 Past ECE Board Problem
A4-stage amplifier has stages with the
Past ECE Board Problem where: where: following specifications: Stage 1, power
An amplifier operating over a 5-MHz dBmC=dBm+90 =-76 +90 gain [s 10, noise figure is 2; Stage 2,
bandwidth has a 100-ohm input power gain is 20, noise figure is 4,
NF = (§)NJnpa
dBm = 14
resistance. It is operating at 27 degrees _ 100 _ Silage 3, power gain is 30, noise figure
Celsius, has a voltage gain of 200 and “The signal level in dBmCO is is 5. Calculate the total noise figure of
an input signal of 6 Vn, Calculate the N ouaput
18. the system, assuming matched
output rms. noise. condition.
“. The noise figure is 6.9 dB.
Vlotatéane: i1144 ee Flatations:
PROBLEM 20 Past ECE Board Problem
Vi = rms noise valtage (V) A diode generator is required to produce NF = noise factor or noise ratio (absolute value)
k= Boltamann-constant = 1.38 x 1024 1/K Past ECE Board Problem 12 microvolts of noise ina receiver with F =noise figure (dB)
T= nose temperature (K} Convert noise factor of 4.02 to an input-impedance of 75 ohms anda G = amplifier gain (absolute value)
BW = effective noise bandwidth (Hz) equivalent noise temperature, Use 300 noise power bandwidth of 200 KHz.
A, = equivalent noise resistance (£2) K for environmental temperature. Determine the current through the diode Solution;
in milliamperes.
Solution: Paidations:
“ge .NFL-
NF=Ne, Nt 4, NE}
Notatinas:
For input noise voltage, Ts =equivalent noise temperature (K) 4—1 5-4
Vi, = /4kTBWR,, Te = relerence noise temperature (K} I= ems shot noise current fora diode (A) NF=2 =
+0Lo * 3 (10)(20)—— ie.

NF = nolée factor (absolute value) 1p = reverse saturation current (A)


lic = direct diode current (A) F = 10 log
NF
V,= 2.88 pV Solution: Q. = charge of an electron = 1.609 x 10°C F = 10 log 2.32
Vee = AM, = (200)(2.88 nV) = 575.5 pV BW = noise bandwidth (Hz) F=3.65 08
T, =T, (NF-1)
Solution: The total noise figure is 3.65 dB.
T, = 300/4.02—1)=906 K
-. The output rms noise voltage is
575.6 lV.
-. The equivalent noise temperature is 1, = (20, (A, =1,.)BW.
‘806 K. 2
PROBLEM 24
ated Mae) do=
20,50 Past ECE Board Problem
Past ECE Board Problem PROBLEM 21 (0.16x10°)" Two resistors rated 5 ohms-and 10
4 transistor amplifier has a measured ohms are connected in series and are at
S/N power of 100 at its input and 20 at Past ECE Board Problem 2(1.609x10"*)(200x10") 27 degrees Celsius. Calculate their
its output. Determine the noise figure of When measuring a voice channel at a 4
\., = 0.398 4= 398 mA combined thermal noise voltage fora 10
the transistor dB test point level, the meter reads -76 kHz bandwidth.
dBm. Calculate the reading in dBrnco.
-
The Question Bank Series - Communications Engineering (Volume 2 - Problems) Question Bank 4 - Noise

Hotetions: bod 0813 8d ., The equivalent noise temperature is Solubon:


129.2 K.
Past ECE Board Problem F=10log NF
Var =nms thermal noise voltage '{V)
A transistor amplifier has a measured
k= Bellman constant = 1.38% 1022 ik S/N power of 60 at its input and 25 atits F=10 log 4=6.02
dB
T = noise temperature (K) edt) Need
outpul Determine the noise figure of the Where:
BW = effective noise bandwidth (Hz) Past ECE Board Problem
Ar = equivalent noise resistance (2)
transistor.
An amplifier with an overall gain of 20 () (ee)
d5 Is impressed with a signal whose = AN Gingut | 1 pW pa
Solution: Flotections: power level is 1 watt. Calculate the (S) ~ f AW:
Vi. = VAKTBWR, power output in dBm, UN Jsdat eat
F = noise figure (dB)
Vir = ,4(13axt0™) (27 +273)(10x10")(15) S/N = signal to noise ratio (absolute value)
© Pictirteaa: -. The amplifier noise figure is 6.02 dB.
Vi= 0.05 Vv Solution:
where; G(dB} = amplifier gain {dB}
‘Poot = oulput power (lf) PROBLEM 30
R;=5+970=15 ohms
P= input power (W} Past ECE Board Problem
4.3 stage amplifier
is to have an overall
“. The combined thermal nolse voltage ‘Solution: noise temperature ne greater than 70°K,
is 0.05 pV. The overall gain of the amplifier is to be
G(dB)= P,,, (dBm) +P, (dBm) at least 45 dB. The amplifier is to be
gt) eee} p.,, (dBm) = G(dB)-P,, (Bm) builtby adding a low-noise first stage to
an existing amplifier with existing
Past ECE Board Problem “The noise figure of the transistor is P,.. (dBm)= 20 +30 characteristics as follows: Stage 2 has
A transistor has a measured S/N power
of 60 at its input and 19 atits output
5.05 dB.
P,., (dBm) = 50 dm 20 dB power gain; 3 dB noise figure.
Stage 3 has. 15.8 power gain and 6 dB
Determine the noise figure of the where: noise figure. Calculate the maximum
PROBLEM 27
transistor,
Past ECE Board Problem
P,(dBm)= 10log{ 7. }-20 noise figure (in dB) that the first stage
can have.

Vlotatieats: A satellite receiver has a noise figure of


1:6 dB. Find its equivalent noise «The output power is 60 dBm, Hotatianis:
F = noise figure (dB) temperature.
S/N = signal to noise ratio (absolute value) NF = noise factor absolute value}
PROBLEM 29 Te = equivalent noise temperature, K
Hotaticaai
Solution: Past ECE Board Problem Ty = 100m temperature (290°)
Te =equivalent noise temperature The signal power of the input to an G = amplilier gain (absolute value)
SIN T. = reference noise temperature “amplifier is 100 microW and the noise
F=10log iS Vt NF = noise factor, power is 1 microW. At the output, the
signal power is 1 W and the noise power
F=10log
60 Solution: is 40 mW. What is the amplifier noise
19
figure? ;
F=4,99=5 dB 7, = Ta(NF—1)
Plotations: Ter Tas
Te = 290(1.445—1)= 129.2 K
-. The noise figure of the transistor is
6 dB. where; Solution:
F = noise fiqure (4B)
NF = noise factor (absolute value) Overall T, =70 K
NF = log” i
S/N = signal-to-noise ratio (absolute value) T, 70
Overall
Vere NF= 1 +2
T, =1+——
+ 390
NF =log-'{7*
Overall NF = 1.24
NF= 1.445
4-10 4-171
The Question Bank Series — Communications Engineering (Volume 2 - Problems) Question Bank 4 - Noise

Overall G; = 45 dB Solution: Solution: Woo =/4KTBWR,


Overall G-= G,(dB)+G,(dB)+G,
(dB) $_ Signal Power ‘F=10log NF=10 log 10 Vga = 4(1.38x10*)(400)(100x10") (300)
45dB=G,+20415 N Noise Power F=10dB8 Vio =8,139x10"" vat
G,(dB)=10 dB =10 log G, since N «= BW (from: N=kTBW)
For
vlnaseries Grout,
a
ae “
=10 izthen if BW is doubled, N is also doubled The noise figure is 10 dB.
G, =log”
—— Signal to noise ratio is decreased by — Ts on Erect
@2(98) -20 & pus " 4 aA Var = (6.785x10°7) + (B.139x10-7)
G, = log” ae= = 100 <The sipnaltonolve ratio ts halved: Past ECE Board Problem if <;
Express the ratio in decibals of power Vor = 9-868x10" volt
NF = NF, + NE,-1 NRy-1 ratio 50 isto 10 Watts. For max. power,
: PROBLEM 32
G SiG, Vos Ver (50% efficiency)
NE = NE NF,-1 NF, —1 Past ECE Board Problem 2
+ r 5 Determine the shot noise for a diode Solution: f pe
G, GG, with a forward bias of 1.40 mA over an 5 ae
895-7
NP eae 3.88—1
Ra ee 80 KHz bandwidth. (q = 1.6x 10 raised P. 50 2b MioS
1 10 (10)(100) to minus 19 C) Ratio in dB =10log—p= 10log| 3 io R, (500)
NF, =1.1375 a Ratio indB= 6.99 ne P= 0.496 {W
F,=10log NF, =10log 1.1375 Motativais:
Th tio in dB is 6.99. >, The power produced at the load is
F, =a 0.06.08 |, = rms shotinoise current for a diode (A) Se akc ait 0.496 TW.
where la = direct diode current (A)
See ey Lard Bty q: = charge
of an electron =1.602x 10-17C PROBLEM 35
eee = Ge Lee ese aenomaat He) Past ECE Board Problem Bl
F. 6dB : The resistor R; and R; are connected in Past ECE Board Problem
NF; =log” a = log: (ee = 3.98 Solution: series at 300 Here and 400 If voltage change Is equal to twice its
Te degrees K temperature respectively. If orginal value, what is its comesponding
I, = 20,/y.BW
* The first stage amplifier maximum Ri is 200 ohms, and Az is. 200 ohms, Sa
find the power produced. at the load
noise figure is 0.56 dB. (ee j2(1.6x107*)(4.40 mA)(a0x10") (RL = 500 ohms) over a bandwidth of
100 kHz. Solution;
|, =6 nA = 0.006 A
PROBLEM 31 Change in dB = 20 log (#)- 20 log 2
Past ECE Board Problem “- The shot noise for a diode is 6 nA. Notations:
What is the effect on the signal to noise Change indB =6
ratio of a system (in dB) ifthe bandwidth Vn = ims noise voltage (VV)
is doubled, considering all other
ea) ey k=Boltamann constant = 1,38 x 1077 /K “The corresponding change in dB is 6.
Parameters to remain unchanged except Past ECE Board Problem F=noise temperature (K)
the normal thermal noise only. The S/N Given a noise factor of 10, what is the BW = effective noise bandwidtiy (Hz)
will be ; noise figure in dB? R= equivalent noise resistance (€2) a) Ne ed
Past ECE Board Problem
Matitiontas Hobie a: Solution: A 20,000 ohms resistor is al room
a ios ate temperature | (290 deg Kk). Calculate H the
S/N = signal-to-noise ratio F = nnise fiqure (dB) Var = JAK TBWR, threshold noise voltage for a bandwidth
of 100: KHz.
BW = bandwidth (Hz) NF = noise factor (absolute value) Vnq = 4/4 (1.-38x10 *)(300)(100x10")(200)
Vi =5:756x10° volt

4-12
The Question Bank Series - Communications Engineering (Volume 2 - Problems) Question Bank 4 - Noise

Tatationa: F, =F, =F, =10logNF ‘BW = elfective noise bandwidth (Hz) it]: NA
Ar = equivalent noise resistance (£2) The equivalent noise temperature of the
ne
F.

Va = rms noise voltage (V) NF, =Iog7—t


= amplifier is 25 K. What is the noise
k = Boltzmann constant = 1.38% 10°? JK 1=leg 10 Oo io Solution: figure?
T = noise temperature (K) NF, = 1.995 = 2
BW = effective noise bandwidth (Hz) kT =4.002 x 107’ @ T= 290K
R= equivalent noise resistance (02) “The total noise figure is 3.24 dB. (20 kf)(50 kQ) Hintiations;
= 14.28 kn
7 20kN+50kQ NE = noise factor (absolute)
Solution:
PROBLEM 39 NV, = J4KTBWR; 1. = equivalent noise temperatiire,K
T, = foam temperature (290°)
\, = \@kTBWR Past ECE Board Problem
Vp, = \/4(4,002x10 *"}(100kHz) (14.28x10°) G = ampilier gain (absolute)
For an equivalent noise bandwidth of 10
o? \(20, 000)
v= \4 (1.38x10 **)(290)(1 ooxi MHz in a satellite sysiam and a noise V,, = 4.78 iV
power of 0.0280 pW. determine the Solution:
Vy, = 5.658 pV noise density in dBW.
» The total thermal noise voltage in
“The threshold noise voltage is parallel connection Is 4.78 iV.
6.658 uV. Hotations:

N= noise power (W) at)


1
bedi eT EW = Bandwedth (Hz) ‘Past ECE Board Problem Ind:
A receiver connected to an antenna F=10 log NF
Three amplifiers are cascaded together. Solution: whose resistance ts 50 ohms has an F=10log1.086 = 0.36 dB
Determine the total noise figure in dB if
equivalent noise resistance of 20 ohms.
What ig the receiver's nolsé
the noise figure of each amplifier is N= N +. The noise figure is 0.36 dB.
equal to'3 dB and the amplifier's gain is o BW | temperature?
10 d8 each 42
, = 0280 = 2.8x102'W/He Hotitions: bide) fe
10x10
Hotations: In dBW, Past ECE Board Problem
NE-= noise factor (absolute value) The signal ina channel is measured to
2.ex107! Te = equivalent noise temperature (K) be 23 dB while noise in the same
NF = noise factor (absobule value) N, d (dBW) ==\{ 10log pce
a 1W
G = ampliier gain (absolute value) Th = foom temperature (290°K) channel is measured to be 9 dB. The
F= noise figure (dB) N, (dBW) =~205.53 G6 = amplifer gain (absolute value) signal to noise ratio therefore: is

Solution: «The noise density is -205.63 dBW.


“Solution:
Votatinas
Ta4 = Ta (NF—1)
NF=NR et NG =f S(dB) = signal power (dB)
G, SG,
aie):
] aye) Ton = 290(1,6-1)= 174K N(dB) = nolse power (dB) -
2-1 2-1 Past ECE Board Problem where: SIN(dB) = Siqral-noise ratio (dB)
Fo2+
Nia? an Folio) 214 Two resistors, 20 KO and 50k are at
ambient temperature. Calculate for a
T, = 290 K Solution:
In dB, bandwidth equal to 100 KHz, the thermal R
DNF =1+22- 1422-46
F(dB) =10logNF =1Dlog 2.14 noise voltage for the two resistors f R, 50 = (a8) =§(48)-N(2B)
connected In parallel.
F=3.24 dB
where:
Flotations:
+ The receiver's noise temperature is
174K.
= (dB) = 2348-948
G,;=G,
=10 dB =10log G
10
G=log!——=10
ne 0
V. = tms noise voltage (V} = (48)
5
=(dB)= 1408
k = Boltzmann constant = 1.38 * 107? WK
T= nose temperature (K)
4-14 4-15
The Question Bank Series — Communications Engineering (Volume 2 - Problems) Question Bank 4 - Noise

©. The signal-to-noise ratio is 14 dB. |x. = ditect dlade current (A) beg teY . The noise temperature is 870 K.
Q. = chargeof an electron =1.602 % 10:0 A certain company manufactures an AM
BW = noise bandwidth (Hz) receiver with a noise temperature of
PROBLEM 44 PROBLEM 49
450°K. Calculate the noise figure of the
Find the noise voltage applied to the Solution: feceiver. Calculate the bias current of the diode if
receiver inpul ifa 3002 resistor is it produces 150 nA noise current over a
connected in series toa 300 9 antenna i= ji 2q,BW 300 kHz bandwidth.
Platintionat
of a television receiver. i

lee 3a, BW NF =nolse factor (absolute value) Vlotatiens


Flotatians: J. = equivalent noise temperature (K)
e (g0x10° 77 J, = room temperature (290°) |), = ems. shat noise current for a diode (A)
n = ms noise voltage (V) Me 2(1.602x10°" ¥250x10°) lge = direct diode current (A)
k= Boltzmann constant = 1.38x 10° JK le = 101.24 MA Solution: qe = carge of an electron =1,602 x 10°C
T = noise temperature (K) ; BW = noise bandwidth (Hz)
BW = effective noise bandwidth (Hz)
. The current through the diode is NE £1 Solution:
R= equivalent nose resistance (2) 101.24 mA. a

Solution: 450
Soe i 2G
PROBLEM 46 NF 290 "

Va = V4kTBWR Two resistors in series (Ry = 500; In dB: Se


(150x107
ee Sr
kT = 4.002 x 107! @ ambient temperature T; = 350°K) and (Ro = 100n : © 2(1.602%10-'" (300x107)
T=200K Tz = 450°K). Determine the total noise NF. = 10logNF
ve, = ,/akTBWR voltage, over a bandwidth of 120 kHz. NF,, = 10leg(2.6)=41d8 :. The bias current of the diode is.
234.1 mA.
V,, =/4(4.002x10°")(6 MHz)(300) ~The nolse figure of the receiver
is
Votattans: 4.1 dB,
Va, =5.4 uv
a) ee
By voltage divider rule Vp = mms noise voltage (V)
R, k= Boltzmann constant = 1.38 x 1077 Kk Noise voltage at the input of an amplifier
ath
Masia =Ron” T =noise temperature (i) is 1 pV. whatis the signal voltage if the
: anh BW = effective noise bandwidth (Hz) Calculate the noise temperature if the input signal-to-noise ratio is 20 dB.
Input signal-to-noise ratio is 20 and the
V pe Sai R= equivalent noise resistance (2
sl 30090077 = 2) output signal-to-noise ratio is 5.
Flgeatsoms:
Ve Serv Solution;
Viotaticns: 5 = signal power (W}
Hie oe eee voltage at the receiver is Vo= IV, +E, N = noise power (WW)
TM, NF = noise factor (absolute value) Vv = nolse voltage (V)
4kBW = 4(1.38x10-7*)(120x10°) T. = equivalent noise temperature (I) Vs = signal voltage (¥)
o = foam temperature (290°K)
a ae 4KBW = 6.624x107* Solution:
Var = /4kBWiR,T, +R, T,) Solution:
A diode noise generator produces 90 nA
ina receiver with an input impedance of Vv
6.624x10°""| (50)(350) + (100)(450) is Vy,
nl t= | —|=20log) =
75 ©, resistive and a noise power
bandwidth of 250 kHz. What must the Vy = 643.4 nV
np 29 _ 4
5 a ed
current through the diode be? 1, = T.(NF- 1) (=)
T, = 290'Ki4 —4) ey) IN
Flotintions:
.. The total noise voltage for two
resistors in series is 643.4 nV.
Vs =(V,)log aa
T, =870'K
L
I= mms shot noise current for a diode (A)
4-16 4-17
The Question Bank Series — Communications Engineering (Volume 2 - Problems) Question Bank 4 - Noise
BW = effective noise bandwidth (Hz)
Solution: Solution: R = equivalent noise resistance (©)
V, ==(ieid®
(fx10™ Veg"}log if| (2)
20
=
kT =4.14x 107)
@ T= 300K N=kTBW Solution:
V, =10 pV V,, = ¥4kTBWR N=(1.38x10°**)(290)(2.5x10°)
Vi, = JARTEWR
. The signal voltage is 10 pV. Vp, =/4(4. 14x10°*"}(200 kHz)(73) N=10x10-"W
Vp, = 492 nV Vj, = y4(4.1124x10
7" (75 MHz)(30x10°)
itt) a 2. The nolse power of a resistor is
40x 107° W. Vy, = 192.4 WV
A receiver has a noise power of1 nV at -. The noise voltage generated Is where:
the input of the receiver. What would be 492 nV.
the signal power if the signal-to-noise Is ad
DA eee) kT =(1.38x10) (26°C +273)
40 dB? kT =4.1124x410""
att):
1 49 ee} '| What is the noise voltage given that
R = 200 kQ at a temperature of 27°C Rye = 5410+15=30 kf
A three stage amplifier in cascade has and noise bandwidth of 54 MHz?
Vio tititinas?
the following parameters. Stage 1 has a -. The noise voltage generated is
gain of and nolse factor-2; Stage 2 has
§ = signal power (\W) 192.4 iV.
a gain of 6 and noise factor 4; Stage 3 Hotations:
N= noise power (W) has.a gain of 15 anda noise factor of 6.
Find the overall noise factor of the ¥,, = ning noise voltage (V) tea ee
Solution; system. i = Boltzrhann constant = 1.38x 10? J/K

Gm
T =nolse temperature (HK) Calculate the noise vollage generated
BW = effective noise bandwidth (Hz} by3 ke, 10 kG, and 21k, resistors
——

otitions: connected in parallel. Given that the


R= equivalent nose resistance (£2)
temperature is 17°C, and noise
(2 NF = noise factor (absolute)
G = amplifier gain (absolute) Solution:
bandwidth of 15 MHz.
5 = (Njlog Za
S=(Njlog 10
Solution: KT =4,14x4107' @T= 300K
40 V,, =V4kTBWR
Hotetioian:
$=1x107
1x log
og '—
10

S= 10uW NF, = NF, + NE,-1 NR =4


Vi = [a(4 taxto")(54 MHz)(200x10°) Vn = ims noise voltage {V/)
k= Boltzmann constant = 1.38% 10% 1/K
G, GG,

«The signal! power is 10 WW. 1 fn 6-1 Vy, = 423 nV T =nbise temperature (K)
NEE =12 poe BW = effective noise banchvidth (H2)
(8)(8) R =equivalent noise resistance (£2)
NE, =2.725 “. The noise voltage is 423 pV.
PROBLEM 52
Solution:
What would be the noise voltage ¢, The overall noise factor is 2.725.
ae) DL)
generated of a 73.9) resistor at room Vi, = /4RTBWRa,
temperature (27°C) over the bandwidth Calculate the noise voltage generated
ofan FM channel? PROBLEM 54 by SkO, 10k, and 15 kQ, resistors Vp, = 4(4.002x10°*")(15 MHz)(2079)
connected in series, Given that the
What is the noise power of a resistor al
room temperature (290°) with noise temperature is 25°C, and noise Vp, = 22:3 wV
Phitertints:
bandwidth of 2.5 MHz? bandwidth of 75 MHz.
where:
Va = ms noise voltage (V)
k = Boltzmann constant = 1,38 x 1073 J/K Hotatione; kT = (1.381 oy (IMC +273)
VLotatims:
T = noise temperature (K) kT =4.002x 10°"
BW = effective noise bandwidth (Hz) N = nolse power (iW) V, = rms noise voltage (V) Roy = 3||10)/21=2079 2
R= equivalent noise resistance (02) k= Boltzmann constant = 1.98 © 10°73 WK k = Boltzmann constant = 1.38 x 10 J/K
7 = noise temperature (K} = noise temperature (K} .. The nolse voltage generated Is
Bil = effective noise bandwidth (Hz) 22.3 jV.
4-18 4-19
The Question Bank Series - Communications Engineering (Volume 2 - Problems) Question Bank 4 - Noise

Solution: kT = (1,38x10) (17°C + 273) PROBLEM 62


id tt)
1 0S ety kT= 4.002 x 10° " @ room temperature
A mixer with input resistance of 200.9
The shot noise current ona diode is 25 Vv, = V4KTBWRo R, =P, i R, =300 | 20 = 18.750 has an equivalent noise resistance of
nA if 2.5 mA direct current is flowing thru
Vp, = /4(4.002x107")(10 kHz)(4960) Ry =R, +R, = 18.754 100 = 118.750) 100 ©. What is the S/N at the input ifthe
it, What is the effective noise bandwidth effective noise bandwidth is 25 kHz, a
of the diode? source voltage of 2 Vand with internal
Vp, = 0.9 WV .. The thermal noise voltage at the
resistance of 20 7
where: input is 0.218 pV.
Vlotetions:
ate) TS Flotatieas:
1, = ms shot noise current for a diode (A) Inca TVRO circuit the antenna is
le = direct diede current (A) POBE connected directly into the amplifier, To Vy = Tins noise voltage [¥)
Qe = charge of an electron =1,602 x 10°79C (2)()(1605x10" )(20x10""7) match the input impedance of the k = Bollamann constant = 1.38.% 10? 1/K
BW = noise bandwidth (Hz) antenna and the 15 kQ2 input resistance 7 = nose temperature (K)
Rp = 4.98 kQ of the amplifier a coupling system is BW = effective noise bandwidth (Hz)
Solution: kT =(4 38x10) (17°C +273) used with 10:1 ratio, The amplifier has R= equivalent noise resistance (C2)
kT = 4.002 x 107" an equivalent noise resistance of 150
L, = f2ly-q.B ka, What is the noise Voltage seen at Solution:
-. The noise voltage of the receiver is the amplifier’s Input terminal? Given that
Wil = 0.9 pV. the effective noise bandwidth is 20 KHz
ius = Ry = eee 200
hee at room temperature (290°K)}.
se R +R, M 200420 *
(25x10°°)" PROBLEM 60 = 1.82pV
~ 2(2.5x103)(16xi0%) A mixer with input resistance of 300.1
has equivalent noise resistance of 100
Flotations:
Ry =F | R, = 200 || 20=18.202
BW = 781.25.kHz Vj, = rms noise voltage (V} R,=Rp +R, = 16.24 100 = 1718/20
0) What is the noise voltage at the input
k= Boltzmann constant = 1.38 x 107? JK kT = 4.002 x 10" @ ambient temperature
when the effective noise bandwidth is 25
-. The effective noise bandwidth of the 7 = noise temperature (K) T=290 K
kHz, the Source voltage is 2 nV and with BW = ‘effective nose bandwidth (Hz)
diode is 781.25 kHz.
internal resistance of 20 (27
‘A= equivalent note resistance (£2)
V, = /4kTBWR,
A, = equivalent noise resistance = 15022
any ee
Vatations: Jf =input resistance of the miner = 15 kD Vas 1 (4.002x1077)(25 kHz)(118.2)
A radio receiver uses a single parallel A, =antenna input resistance = 15 ko2
tuned circuit, designed to resonate at Vn = rms noise waltage (V]
Vy, =9.217 uv
1605 kHz. The Q-factor of the receiver k= Boltzmann constant = 1,38 10? 1K
Re =RaliR +R,= 157.52 » ya a
is 50 and the channel bandwidth is T = noise temperature (K) eles eel =703

Z|
limited to 10 KHz. What's the noise BW = effective noise bandwidth (Hz) ‘Solution: Vv, 0.217
voltage of the receiver at the input at
room temperature (290°K) if the
A= equivalent noise resistance (£2)
V, = /ARTEWR, = 10log 70.3 = 18.47 dB

os,
Z|
capacitance of the tuned circuit is 20 Solution: ue
pF? V, = 4(4.002x10-*")(20
kHz) (157.5x10")
”. The S/N at the input is 18.47 dB.
V,, = 4KTBWR, Vi =7.4 BV
Hloatations:
Vi = 4(4.002x10*")(25 kHz)(118.75) PROBLEM 63
Q = quality factor (unitless)
Where:
Vi. =0.218 pV kT = 4,002 x 107" @ room temperature What is the effective noise bandwidth of
Ay = parallel resistance (©) T=290K
C= capacitance (F) a parallel tuned circuit with 5 kQand
where: 250 nF?
(@y = angular frequency = 2nf (rad/s) «. The noise voltage is 7.1 pV.
Ri = 300-9, Ra = 10002,
f= frequency (Hz) Fataticns:
R= 20 2 and V,=2.0 nV.

4-20 4-21
The Question Bank Series - Communications Engineering (Volume 2 - Problems) Question Bank 4 - Noise

BWea = effective noise bandwadth of parallel tuned Viotetiors: Alternate Solution


circuit (Hz)
Q = quality factor (unitless)
R, = paralle! resistance (Q) {= resonant frequency (Hz)
S/N = signal-to-noise ratio
M = number of circuits in tandem connection
C= capacitance (F)
Ay = parallel resistance (£2)
Solution: C= capacitance (F) thus Overall - - (2) — Mos
: 1 Vj, = ms noise voltage (V)
BWor =Re We V5= Var log 1S = 224.5 uv
k = Boltzmann constant = 1.38 « tO? /K
Overall - =30-10log M
T = noise temperature [K) voltage level seen at the secondary winding
BW 1
= 200 Hz BWex = chective nose bandwidth (Hz)
“4(5000)(250x107) aay,
aR 218415
15 a5) §
Solution: Overall WN =30-10log4
-. The effective nolse bandwidth is
Ve = 449 nV
$
Overall a =24 dB
200 Hz. a
” xfC
eT) _. The overall signal-to-noise ratio in
aed
A eee! Ry = peak ed 2 srl f9 79.6 ko) 1 tandem connection is 24 dB.
2n( 5x10" )(20x10-") Me = i575 7
44guV
What is the effective noise bandwidth of 10
@ parallel resonant circuit that has-a Q- V, = /4KT EWR Wa =44.9 nV
factor of 100 and resonant frequency of bat)
|A
5 MHz? V, = /4(4x107")(15x10°)(79.6x10") _. The antenna EMF is 44.9 iV. What is the output S/N ratio if the noise
Vi=44 iV facter is 20 and 25 dB input S/N ratio?
Vlatealicmss
-. The noise voltage is 4.4 pV.
PROBLEM 67
HMotaticress
BWer = effective noise bandwidth of parallel tuned Four telaphone circuits ara connected in
circuit (Hz) tandem, Whatis (he overall S/N ratio if NF = noise factor (unitless)
i, = resonant frequency (Hit) PROBLEM 66 : _|-8ach circuit has a S/N of 30 dB. 5 = signal power (W)
Q = quality factor (unitless) N = noise power (W]
Ina TVRO circuit the antenna is
connected directly into the amplifier, To

:
- Solution: match the input impedance of the
Hota tics:
Solution:
antenna and the 15 Kt input resistance $= gional power (WW)
BW,, = nf of the amplifier a coupling system is
NF =_(N)
20 used, The amplifier‘has an equivalent
= SNINE noise voltage of 7,1 .V at the input,
Determine the antenna EMF to produce
m2 400 \N a
a.S/N of 30 dB if the transformer
BW,,, = 78.54 KHz coupling has a turns ratio.of 10:1. Given s
that the effective noise bandwidth is 20
= 10log—
ar (F] = 20(316.23) = 6324.56
-. The effective noise bandwidth is
78.54 kHz. KHz at room temperature (290°K}. ae
109) - 1000 where t
2
Hatalions: Overall S/N connected in tandem (3) =10" = 316.23
a98 eed
A parallel tuned circuit with 20 pF tuning
capacitor has an effective noise
A, equivalent noise resistance = 150
k
A, = input resistance of the mixer = 15 k02:
fe)
S) (Nn) 4000_
GR) we: eae
.. The output signal-to-noise ratio Is
6324.56,
bandwidth of 15 KHz. Whatis the noise Ts
voltage across the circuit at room A, =antenna input resistance = 15 KO
: Lil =10log250
= 24 dB
temperature (290°K) ifthe resonant CSN Tas
frequency is 5 MHz and a Q-factor. of
60?

4-22 4-23
The Question Bank Series - Communications Engineering (Volume 2 - Problems)

PROBLEM 69 Ee
F=10" =100
An AM receiver with RF amplifier, mixer
and an AF amplifier has an overall noise
figure of 60. The mixer has anoise Tuer = To(F -1) = 290(100 - 1)
factor of 20 and power gain of 30, and Thamar = 290(99)= 28710°K
AF amplifier has noise factor and power
gain of 30 and 50 respectively. What is 28710
Waele ane = 60+
J, ir

the noise factor of the RF amplifier if its 10


power gain is 507 T, =2931'K
Plotectiais: .. The effective nolse temperature is
2931 K.
NF = noise factor (absolute)
G = amplifier qain (absolute)
ad)
1 eral
Solution: Past ECE Board Exam
In a microwave communications system,
NFmoenr
vow, ~1, NFyeAF = 4 determine the noise power in dBm for
nether GaP GkiGe
N = NE a
an equivalent noise bandwidth of 10
MHz.
Ne = Ne
NFee =1 NR =4
commas Gare a Vlatations:
20-1, 30-1
NF, =60 -|
50 (3050) N =snoise power (W)
NFa: = 59.6 k = Boltzmann constant = 1.38 x 1078 J/K
T = nolse temperature (K)
BW = effective noise bandwidih (Hz)
-, The noise factor of the RF receiver is
69.6.
Solution:

ate)
1A N=kTBW
A TVRO parabolic dish reflector mixer N= (1.28x10")(290)(10x10°)
stage has a noise figure of 20 dB and is
N=4.00210° "Ww
preceded by an LNB with a power gain
of 10 dB and an equivalent noise In dBm,
temperature of 60°K. What is the
effective nolse temperature at the Input Nugm = 10log———- Question Bank 5
of LNB?
Nagm = -103.98 dBm AMPLITUDE MODULATION
Hotationa:
-. The noise power of the microwave
system ts -103.98 dBm.
NF = naise factor (absolute value)
F = noise figure (dB)
Te = equivalent noise temperature (K)
Ty = room temperature (290°K)

Solution:

4-24
The Question Bank Series — Communications Engineering (Volume 2 - Problems)

at)
A ne

F=10'0 =100
An AM receiver with RF amplifier, mixer
and an AF amplifier has an overall noise
figure of 60, The mixer has a noise
Tuner = T, (F-1) =290(100-1)
factor of 20 and power gain of 30, and Tuer = 290(99) = 28710°K
AF amplifier has noise factor and power
qj 28710
gain of 30 and 50 respectively, What is T, = Ths $e = 60+
the noise factor of the RF amplifier ifits See Gaia 10
power gain is 50? T, =2931'K
Hatations: .. The effective noise temperature is
2931 K.
NF = noise factor (absolute)
G =amplifier gain (absolute)
PROBLEM 71
Solution: Past ECE Board Exam
Ina microwave communications system,
= = determine the noise power in dBm for
NF, = NFag + Nimo — 1, Nar an equivalent noise bandwidth of 10
Gye Spice Sar
MHz.
Ng =F, —( Mie=1, Mise) RF oer
P Tota tienes!
20-1 30-1
NF, = 60 (a es)
N=ynoise power (W)
NF,- = 59.6 k = Boltzmann constant = 1.38 x» 102 }/K
T= noise temperature (K)
., The noise factor of the RF receiver is
BW = effective noise bandwidth (Hz)
69.6.
Solution:

AY N=kTBW
A TVRO parabolic dish reflector mixer N=(1.38x10*)(290)(10x10°)
stage has a noise figure of 20 dB and is
preceded by an LNB with a power gain N=4,002x10 “Ww
of 10 dB and an equivalent noise In dBm,
temperature of 60°K. What is the
effective noise temperature at the input Naam = 10log
4.002 x 107'* Question Bank 5
of LNB?
1x 104
Noam =-103.98 dam AMPLITUDE MODULATION
Motations:
“. The noise power of the microwave
system is -103.98 dBm.
NF = noige factor (absolute value)
F = noise figure (dB)
T, = equivalent nolse temperature (K)
T, = rom temperature (290°K)

Solution:

4-24
Question Bank 6 —Amplitude Modulation (AM)

dt)
aes
Enteerie
mee
E ‘a

J08
Past ECE Board Problem E. Ey c
Determine the power saving in percent 2 z 2
when the carrier is suppressed in an AM

are
m=

mn
signal modulated to 80%, Ee E.

Potattons: _ Ep+E +E _ EF HEE +ES


P, = saved power (W) Re E E,
Pup = suppressed power (W) Substituting values:
Py = total power transmitted ('W)
im = modulation index
m, =~ = 0.468
‘Solution:
Then
r Pp. M=m, x 100% =46,8%
OP, = Free
”. The total percent modulation (M) is
= *x100 46.8%.
oF 1.32P,

“%P, = 75.76 %
bint) te)
where: 4 An SS6 transmitter radiates 100 Win a
75 ohms load. The carrier signal is
2 _ (o.By modulated by two tones, with
Pp = pis | P, 1.008") frequencies 2 kHz and 3 kHz with equal
magnitude and only the lower sideband
P= 1,32P, is transmitted with a suppressed carrier.
What is the peak voltage across the
_.. The power saved in percentis load?
75.76 %.
Hotetiems;
= PROBLEM 2
PEP = peak envelope power (W)
| Find the percent of modulation of an AM Fy = load resistance (2)
signal if 8 V signal carrier is modulated Vp = peak voltage ()
by three different frequencies with
| amplitudes 1V, 2V, and 3 V.
Solution:

— Motations: Vi 3,
Pep =—is =F
“hy = total modulation index RL aR,
= (otal percent modulation Vp = /PEP(2R,) = /100{2)(75)
_E = carrier voltage (¥/) Vp = 122.5V
'E, £2 E) = modulating vatages (V)
_ Solution: where :
5}
= \,
Vaus
© My = ym + m5 + my
|
5-3
The Question Bank Series — Communications Engineering (Volume 2 - Problems) Question Bank 5 — Amplitude Medulation (AM)

.. The peak veltage across the load Is Solution: a1 © The efficiency of the transmitter is
122.5 volts. A measure of steepness of the skirts or
60%.
ap, =| Psa)
82) x 100% the skirt selectivity of a receiver is the
oye)
:] 04 ee sR) shape factor. What is the shape factor of
we, =( O88)10
tS
a receiver whose 60-d8 bandwidth is
An AM transmitter delivers 100 W 42 kHz and a 6-dB bandwidth of 3 KHz? Past ECE Board Problem
carrier power, when the supply deliver &
An AM transmission of 2 kW is. 100%
104 from a 15V. What load impedance %F, = 33.93 % modulated, Ifitis transmitted as an SSB
is required by this amplifier in order for it where: Tletutenns: signal, what would be the power
to deliver the rated power? For 100 percent modulation, (m=) transmitted?
_ SF = shape factor
BW = Bandwidth (Hz)
Hotatinies: Hotattans:
Solution:
Z, = Inad impedance (02) P; = total power transmitted (W)
SF BW noe ul 12kHz a
Vee = peak supply voltage (V} Pr = carrier power-(W)
«. The sideband power percentage is BWoiw SkHz
Pea = sideband phwer ('W')
Poe = carrier power (W)
33.33% or 1/3 of the total power m = modulation index
transmitted. The shape factor of the receiver is 4,
Solution:
“Note: The lower the shape factor the better the skirt Solution:
aide) 1a ‘selectivity,

An AM receiver uses a double tuned IF


transformer with coupling constant of at)
1.5 ke, The quality factor of the primary
and secondary winding is 60 and 30 A200 V without modulation and 250 V
with modulation is measured across a
(18) respectively. What is the bandwidth of
the transformerif the IF is 455 MHz? 50 ohms resistive load using true-RMS
where:
Pate 2{100) reading meter. What is the overall For 100% modulation, (m= 1)
7,=1125.0 efficiency without modulation if the
Notations: transmitter draws 1.0 KW of power from P.=15P,
’. The load impedance required to \ the ac line? = Pes ie
deliver the rated power is ke = critical coupling factor Meas 4.5
Q, = quality factor at the primary
1,125 ohms,
Q, = quality factor at the secondary Pitatictie:
P= 2000 W
BW= Bandwidth (Hz)
dit 11) kose = optimum coupling factor P, = oulput power without modulation (1) “The sideband power is 500 W.
P, = source power (W)
In AM, what is the sideband power
percentage assuming 100% modulation
Solution: 1) = alficiency a
Y. = output voltage without modulation (V)
and with camer and total power of 100 es Fives 1
R= load resistance (£2) The de power supply ofan SSB
Wand 150 W respectively?
transmitteris 24 V. Whatis the average
k. = 0.024 Solution: power of the transmitter ifon voice
Hoatactitini:
peaks the current achieves a maximum
Koy = 15k. = 1.5(0.024) of 9.3 AP
P
P, = sideband power percentage} Kop = 0.036 1 =i.
P. x 100%
Pes = total sideband power (WW) BW = kof, = 0.036(455 kHz)
Py = total power transmitted (W)
BW = 16.38 kHz 1= BOW 400%
= 80%
Flotation:

m= modulation index =| oe ° PEP = peak envelope power (W)


«The IF bandwidth is 16.38 kHz. v2 (200) Vs = amplifier supply voltage (V)
Ree = = BO0 W ly = peak current (A)
eR 50 ee
5-4 5-5
The Question Bank Series —- Communications Engineering (Volume 2- Problems) Question Bank §— Amplitude Modulation (AM)

Solution: Tlotateow: Hosta teas:

f. f, 1.51MHz 600 kHz


1
Paw =qPEP 1
to PEP if, 6800kHz 1.51MHz ip = deviation from.a perfect 90° phase shitt fy, = frequency of the image signal (Hz)
f= radio lrequency (Hz)
=212 { = intermediate frequency (Hz)
pe 1(223.2) to 5 (223.2) =f, +2 = 600 kKHz4-2(455 kHz)
‘Salution:

dB suppression = 20 log cot (] Solution:


P.,, = 55.8 to 74.4 W al
=a
4B suppression = 20 log cot [ f= f+ 2f
fey

where: «The image frequency rejection ratio


in dB is 38.57 dB
7 f,, = 650 KHz + 2(455 KHz}
PEP = Va lu = 24(9.3) dB suppression = 27.2 dB = 1560 kHz
PEP = 223.2.W
PROBLEM 12 “The maximum suppression of the .. The Image frequency is 1560 kHz.
“The average power of the transmitter unwanted sideband in dB is 27.2 dB.
An SSB system transmits a peak
ranges from 55.8 to 74.4 W. envelope power of 100 W into a 75
ohms load. The camer signal is PROBLEM 16
Note: For single tone modidation average power is
modulated by two tones, with
aA
equal to PEP, for two tone modulation average An AM signal has 4 carrier frequency of
frequencies 2 KHz and 3 kHz with equal An AM transmitter has an output power 400 MHz and amplitude of& Vp. It is
power is one-half the PEP, bul for random voice
magnitude and only the lower sideband of 100 W at no modulation with modulated by a sinusoidal wave with
modulation average power is in the range of 34 is fransmitted with.a suppressed carrier efficiency of 60%. What will be the frequency of 2 KHz and peak amplitude
PEP to-1/3 PEP. Determine the average power into the output power with 97% modulation? of 2 Ve. What are the lower and upper
load
sideband frequencies?
PROBLEM 11 Notations:
Hatations: Plettsoas:
Past ECE Roard Problem
An AM receiver is tuned to broadcast Py =total power transmitted (W)
{ise = lower sideband irequency (Hz)
station at 600 kHz, Calculate the image Pars = average power (WV) P, = Ps = carrier power (W)
PEP = peak envelope power (Wf) m = modulation index fica = upper sideband frequency (Hz}
rejection in dB, assuming that the input
jc = camer frequency (Hz)
filter consists of ons tuned circuit with a
Qof 40, Solution: Solution: fn = modulating frequency (Hz)

VMotations:
as gir ae
PEP 100
BaP]afi) Solution:
fee =f tn
ae =S50 W fag = 400-0.002
m=0.97 . P.=P,

pan oa
IR = bnage frequency rejection ratio
Q = quality factor ‘ fea = 399.9998 MHz
f, =-signal frequency (Hz) . The peak envelope power ofa two fig = fh
{= intermediate frequency (Hz} tone modulation is 60 W.
Cicn = 400 + 0.002
{4 = image frequency (Hz)
Note: For single tone modulation averiige power Is-equal P, =1,47F, =1.47(100)= 147 W fusp = 400.002 MHz
Soiution: le PEP, but for two tone modulation average
power ls ane-half the PEP,
“The output power of the transmitter ~ The lower and upper sideband
is 147 W. frequencies are 399.998 MHz and
IR= a = 1+ 2p? 400.002 MHz respectively.
image PROBLEM 13
IR = /1+(40)' (2.12) In the SSB, determine the maximum PROBLEM 15
suppression of the Unwanted sideband if ait id
, IR=84.8 A standard AM receiver is tuned toa
the deviation from a perfect S0°phase Station at 650 kHz, If the intermediate Past ECE Board Problem
IR(dB) = 20log984:8 = 38.57 dB shift is 5°? frequency of the receiver is 455 kHz, Given an audio power of 500 W which
whatis the image frequency (high-side will modulate an RF amplifier to 50%
injection)? modulation. Find the DC power Input.
The Question Bank Series— Communications Engineering (Volume 2 - Problems) Question Bank 5— Amplitude Modulation (AM)

Flatations En —Emn 10-2


PROBLEM 19 m=
Evy FE, 1022
Pp
Py = power required from the audio amplifier (WV) Past ECE Board Problem =10h ee m=0.67
P, = DC power input to the final amplifier trom the An IF transformer of a radio receiver Pon =| o9( ts |

p-10(°)0e(ee
power supply (W) operates at 455 kHz, The primary circuit -. The modulation index is 0.87.
has a Q-of 50 and the secondary has. a
Solution: Q of 40. Find the bandwidth using the

mr
optimum coupling factor,
a. p,=(1x 10°)og"| PROBLEM 22
P, = are Past ECE Board Problem
For 50% modulation,
Plototiniss: /P, =15.85: mW A DSB-SC system must suppress the
P)=0.125P, ke = crilical coupling factor
Pag, = 12 08m -8 dB =4 dam carer by 50 dB for its original value of
10W. To What Value must the carrier be
Q.= quality factor at the primary reduced?
P= Fu =500 W Pasir a= 10109 Pep
vac |}
= SOS 26°" 0.425 Q, = secondary respectively
PF =4000 W BW= Bandwidth (Hz)
host = optimum coupting factor Pao =(1 x 10°)og"| 4 Flatiatarnia:
“The DC power input is 4000 W.
Solution: i Pay = 2.512 mW Po = carrier power (\W!)

PROBLEM 18 BW =k.of fee we Solution:

AGB transmitter supplies a total power BW = (0.0336)(455 kHz) Suppression = iol Feo.)
of 120.W to-an aerial. Suppose the BW = 15.288 kHz m=, (hee 4(2.512) Cinew)
camer is amplitude modulated to @ Kea = 1.5K, = 1-5(0.0224) Fs 15.65
depth of 80%, how much power is Ponoay = —otal, TOW
wasted by the carrier signal? keg = 0.0336 m=0.8 ae og" 2) 100000
10
k ee _ =. The depth of modulation is 0.8.
Peinaay
= 0.1 mW
Hotatnans: i {O,0, WV 50)(40)
k= 0.0224
Pr = total transmitted power (WW) Aa “The carrier power after suppression
P. =carrier paver (Wf) is 0.4 mW.
“The bandwidth of the IF transformer termine the medulation index of the
m = modulation index is 16.288 kHz. jgnal given the following parameters;

Solution:
Email =10 Vo

Em = 4 Vp PROBLEM 23
PROBLEM 20 An SSBSC signal is modulated with twa
me audio tones having frequencies of 1200
Pp. -P,( 1 x A spectrum analyzer is used to. measure and 2700 Hz. The receiver is mistuned
- Motiitions:
the magnitude of a carnain signal with so that the BFO js at 2.9993. MHz. What
respect to its frequency. Suppose the
Py equipment shows a carrier at 12 dBm,
Es = mocilating voltage (peak) (V) will be the output frequencies ofthe
Ena = maximum envelope voltage (V) detector ifthe signal is LSB and the IF
with two sidebands 4 dB below the of the receiver is3 MHz?
carrier, what is the depth of modulation Fan = minimum erwelape voltage (V)
120 of the signal?
c
A (0.8) Totattons:
T+ att
Hetatrons:
pe, Ean
= fax = error frequency (Hz)
P.=31W ~ Eman,
{. = carrier frequency (Hz)
= Gamer power (W)
En = Ege —2E., = 10—2(4) BFO = beat frequency oscillator (Hz)
-. The carrier power Is 91 W. Pog = sideband power (W) fii & {i> = output Irequencies of the two audio tone
‘eo =2V
m= modulation index or depth of modulation
(Hz)

5-8
The Question Bank Series = Communications Engineering (Volume 2 - Problems) Question Bank 5 — Amplitude Modulation (AM)

Solution: PROBLEM 25 =load resistance (£2) Solution:

nad
foe = &, -BFO An AM signal supplies 150 W of carrier
power when itis modulated by a single _ Solution: From
fp = 3 MHZ- 2.9993 MHz sine wave toa depth of 70%, Suppose {
fae = 700 Hz the total transmitted power increases by (Fram'P = FR
10% when another sine wave
simultaneously amplitude modulates the For two-tone modulation
LSB: carrer, determine the modulation index
{=f te of the second sine wave. =e A Py = 125 kW
f, = 2700-700 + The rms. current is 11.18.4.
Py
mt te) |
= |2)|—|-1
f, = 2000
fs =1200-700 Pidisittma:
PROBLEM 27
f, = 500 Py = total power transmitted (WW)
A superheterodyne receiver has an |F of
P. = carer power (il!) my = 0.707
“The output frequencies of the 455 kHz and is experiencing image-
m = modulation index
detector is 500 - 700 Hz. channel interference on a frequency of
4570 kHz. The recelver is tuned to : For single tone modulation
Solution:

bea) ee for single tone modulation


P= 110 kW

= fae
‘ “Mojations:
Past ECE Board Problem i 2

A transistor has a power dissipation P.,- Pel : e) =150(141.245) 1; = signal frequency (Hz)
rating of 30 W. Assuming that the _ {= intermediate frequency (Hz) 100
transistor is the only element that Py, = 186.75 W “fai = image irequency (Hz) my = 0.45
dissipates power in the: circuit, calculate
for wo tone modulation thus,
the power an amplifier, using this Solution:
PL, =1 ALPS) = 1,1(186.75) = 205.425 W 2 2
transistor, could deliver the load if it M3 =4/m7" — Im
Operates as Class-A with an efficiency f, =f, +2f
of 30 %.
E
ee a {205.425
ra [eee
{50
eens f, =f, -2f = 1970 ~ 2(455) - 660 kHz my = y(0.707)* - (0.48)"
mo =0.545
m,
= 0.86
Votationa: + The receiver is tuned to 600 kHz.
me=miim,?
“The modulation of the second
P,
Pj =
= output power (WW)
input power (W)
m; = m2 —m? = (0.86? - 0.72 ed 0) 1 ea
sinewave Is 0.545.

1) = efficiency m=0:5
j An. AM transmitter with a 100-kKW camer id fe): ee)
Py = power dissipation (W] transmits 125 WW wien two-tone
+The modulation index of the second modulated. Calculate the modulation Past ECE Board Problem
Solution: sinewave is 0.5, index of the second sine wave if the When measuring the selectivity of a
total transmitted power is 110 when receiver, you discover that a signal level
P,=nF PROBLEM 26
| modulated by a single sinewave. of 450 microV on an adjacent channel is
Po =1(Pp +P)
required to give the same output as 4
Past ECE Board Problem | microV signal on the channel to which
P, = 0.3(30+P,) What nms current must a wire carry to the receiver is tuned, Calculate the
O.7P, =9 deliver an average power of 500 Wtoa
Flatationss
adjacent channel selectivity in dB.
P, = 12.86W 4-ohm loudspeaker?
Pr = total output power transmitted (1)
P-= carrier power (W) Mlotatiemat
*-The output power of the amplifier is Potatoes: M= modulation index
12.86 W. m= total modulation index K= adjacent channel selectivity (dB)
P= average power (W)
|= rms current (A)
5-10 5-11
The Question Bank Series —- Communications Engineering (Volume 2 - Problems) Question Hank §— Amplitude Modulation (AM)

Solution: Vola tions: P, = 41.65 W Solution:

= 450.x 107 } R, = load resistance seen at the collector (2) Gee Ooa| |
K=20Ie9| aad |
Veq = collector voltage (V} in

K= 53068 P.= carrer power (W) ree Pai 25


Fs ae 0.6
a1) an loa”! |
‘> The adjacent channel selectivity is
Solution:
P, =83.3W log (| : (eI
63 dB.
From Pi=25W
-The-audio power required to have
Mien aoe 100% modulation is 41.65 W.
PROBLEM 30 SRG V2Re *- The power supplied at the second
stage is 2.5 W.
4300 'V without modulation and 350 V Let
with modulation are measured across a Vee
= Vp
Bd) Na ek
50 ohms resistive load using true-RMS thus Past ECE Board Problem PROBLEM 35
reading meter, What is the amount of ‘The Values of Vinx and Vij, as Ttead from
power supplied in the transmitter when an AM wave on an oscilloscope are 2.6
Past ECE Board Problem
modulated ifthe efficiency is 60%? An AM transmitter supplies a 10 kVY of
‘and 0.29 respectively. Determine the
carrier power to a 50 ohms load. It
percentage modulation.
operates at a carnar frequency of.1.2
Notations: MHz and ts 45% modulated by a 3: MHz
“The load impedance as seen from the sine wave. Calculate the RMS voltage of
—-

Hotations:
Pr= Ps = oulput power when there’s modulation collector Is 4.8 0. the signal.
() M = percent modulation
View = maximum output voltage (RMS) Fear = Matomum peak envelope {\)
P, = power supplied (W) ea) a ey Ene = minimum peak envelope (V)
PDadiitionts:
R= load resistance (£2) Past ECE Board Problem
Py = total power transmitted (WW)
The output of a power amplifier of an Solution:
Solution: Pe = camer power (W)
AM transmitter is 50 Wowith an
m = modulation index
efficiency of 60% and is collector-
modulated. How much audio power is gg — Ema —Emn 100% mr = total modulation index
Ernx Sania
- Mona r (350)°
required to be supplied to this slage for
aaa Rees aco. 100% modulation? 2.6-0.29
"6M= ———__——— «100%
Solution:
P, = 2450 W 2.64 0,29
To = 79.9%
P, = FP, _ 2450 Hotations;

cohtehag tht
tT 0.6

(ono) 16 Wy25
+. The percent modulation is 79.9 %.
P, = 4083.33W P= power required trom the audlo amplifier (W)
P =(10x10°)/1
0.85)
P, = de power input to the final amplifier from the
“The power supplied in the transmitter pawer supply (\W} iat) NBT PL = 136125
W
is 4083.33 W. Psp = total audio’ power
A 3-stages RF amplifier of an AM
1) = efficiency
SOR
ee
transmitter has an oulpul power of 25
kW. What is the power supplied af the
PROBLEM 31 Solution: second stage if the gain of the last stage Vis = 9/PR = ,/(13612.5 W) (50)
A-transistor RF amplifier is required to From | is 10 an? Vi _ = 825 V
produce 15 W of carrier power when iri?
operating from a 12 ¥ supply. What is Peg =P; “ The RMS voltage of the signal is
Hotations:
the load impedance as seen from the 2 825 V.
collector?
Form=4 Gas = power gain (dB)
P, = 0.5P, Pea, = output power of thelast stage (WW)
P, =0.5(83.3) F., = output power of the second stage (|)

5-12 5-13
The Question Bank Series —- Communications Engineering (Volume 2 - Problems) Question Rank 5 — Amplitude Modulation (AM)

PROBLEM 36 aA eT) ate)


1A et) IR(dB) = 29.93 dB
Past ECE Board Problem Past ECE Board Problem An AM transmitter has an output power _& & 124.5 1004
An AM signal has a modulation carrier An AM signal has the following of 30 kW. If the last stage of the RF Pet, f, 1004 1215
power of 10 watts. The percentage of characieristics: carrer frequency = 150 amplifier has a gain of 20 dB and an
modulation is 90%. The total sideband MHz; modulating frequency = 3 MHz: efficiancy of 75%, haw much current
p= 0.39
power is ‘ peak carrier voltage = 40 volts; and does this amplifier draw from a 125 V fy, =f, +26 = 100.142(10.7)
peak modulation voltage is 30 voits. supply? {, =121:5 MHz
Calculate the peak voltage of the lower
Wiotatesias:
sideband. frequency.
Ylotations: “ The image frequency rejection ratio
Pegr = total sideband power (W) is 29.33 dB.
m= modulation index Hatatemes: P= supplied power (W)
Pe = carrier power (W) Pa = output power (W)
Vso = peak-voltage of the sideband (\) n= efficiency PROBLEM 42
Solution: E. = peak carrier voltage (V) |= collector current (A) An amplitude modulated signal has a
En = peak signal voltage (V) Va = collector voltage (\V) maximum positive envelope voltage of

SS
_mép,2 _= (0.90)*5 (10)
Pes;SAT = Solution: Solution:
14 V anda minimum of 2V. Whatis the
total transmitted power if the carrier

ob he: iio
poweris 10 kW and suppressing one of
Psp = 4.05 W y= ME, _ (0.75)(40) P 40W the sidebands?
petra ests as= 3270 mA
2 2 ke V., 125
-. The total sideband power Is 4.05 W, Veg = 5.
where Fiotatiens:
E= 30
m -e—_— ==
P. 0

J
ai)
18 iy E, 40 Pp, =
ey
=——
O75
=40 W Pr = total transmitted power (1V)
meO:75 Pe= carrier power (W)
4 Class C amplifier has a carrier output
Eran = Maximum envelope voltage (\!)

ke
powerof 100 W and an efficiency of +} The current drawn from the power is
75%, when operating froma 20°V
-- The peak voltage of the lower Ema = minimum envelope voltage (V)
sideband is 16 V. 320 mA. m = modulation index
supply, If the audio power is 25 W, what
is the impedance seen looking into the
Solution:
power amplifier fram the modulation
ditt] Ue) PROBLEM
41
transformer secondary?
An AM signal has-a carrier frequency of Past ECE Board Problem
3 MHz and amplitude of 3 Vor. What is A receiver has two uncoupled tuned
Mataticna: the amplitude of the sidebands if the circuits before the mixer, each with a Q
of 75, The signal frequency is 100.1 _ 14-2
modulation index is 0.67
14+2

bp
2, = impedance seen looking inte the power antplifier MHz and IF is 10.7 MHz. If the local
P, = audio power oscillator uses high-side injection, what m= 075
Vee = collector supply voltage Flotation: is the image rejection ratio in dB? >
m
PB = P, ( + =)

Solution: Ets = amplitude of the sideband Hotatnians:


Ec = carrier valtage (peak) f- 2
P, =10 lav} 22
vz _= (20) m = modulation index IR = image frequency rejection ratio

ery:
25S 16.9 f= qualty factor
ize 25 Solution f= signal frequency (Hz) P.=14.47KW
f= intermediate frequency (Hz)

fee
*- The impedance seen looking into the
Ee; 0.85) fi = image frequency (Hz) + The total power transmitted with ane
power amplifier is 16 (7. 5 = 0:9 Yok
Solution: of the sidebands suppressed Is
11.41 kW.
*: The amplitude of the sidebands is IR(GB) = 2010g,/7+ Q2p?
0.5 V.
IR(AB) = 20log,/1+(75)" (0.39)°
5-14 5-15
The Question Bank Series = Communications Engineering (Volume 2 - Problems) Question Bank 5 - Amplitude Modulation (AM)

PROBLEM 43 Ty = reference temperature (°C) _ Solution: Vlotativsis:

The average collector current ofa T= operating temperature (°C)


Peat = total sideband power (WW)
transistor RF amplifier operating Glass m,., =m, = im," + m3”
P. = camier power (i!)
Cis 250 mA. Ifthe efficiency of the Solution:
amplifier is 50% and operating at 50 V
iM = (o.20)' 4 (0.80) m = modulation index
supply, how much audio power is f= t.+k&(T-T,) My =™m, =0.82
Solution:
needed for 100% modulation? f, = 150 MHz +(1x10)(180 MHz}(34 C-20°C)
{, = 150.0021 MHz “The effective modulation index is
Pest Se
Pm?
PTesturtidinsts 0.82
-- The transmitting frequency is
= supplied power (W) 150.0021 MHz,
— 2Poer _ 25)
P. = audio power (W)
ate)
| et Som? (os?
m = modulation index ') Past ECE Board Problem Pl= 40 kW
V.. = collector voltage {¥) PROBLEM 45 Galculate the power in one sideband of
|= collector current (A} “an AM signal whose carrier power is 50 .. The carrer power is 40 kW.
A standard AM radio can receive signal
watts, The unmodulated current is 2A
from 565 — 1650 kHz. lfan AM receiver while the modulated current is 2.4.4,
Solution: i5 tuned at B54 kHz, what is the
PROBLEM 49
2
frequency of the image signal?
Past ECE Board Problem
ae > Ps Hotations:
Ifa transmitter supplies 8 kW to the
P, =0.5P, =0.5(12.5)
Matationa:
Pisa = sideband power (W) antenna when it is unmedulated,
P.= carrier power (W) determine the total radiated power when
P, = 6.25W fa = image frequency (Hz) m = modulation index modulated at 30%
where: f, = signal frequency {Hz}
m=1 {= intermediate frequency (Hz} Solution: Vitotuetttijad:
P, = VI. = 50(0,25) Solution:
P,=12.5W {=f 2h
Py = total radiated power (W)
P. = camer power (WV)
=
f, =854 kHz +2(455 kHz) m= modulation index
-‘. The audio power needed for 100% Pag
aCe). 11.045 W
modulation is 6.25 W. f, = 1764 KHz
where: Solution:
«The image frequency is 1764 kHz, m= [2(i? -1) rat

fob Ai
aE P, = Rl + =|
Past ECE Board Problem m= [2| (1.2)'-1|-0.94
A radio transmitter has to operate at a aA ae E) with m= 0.30
temperature of 34 degrees Celsius. If its Past ECE Board Problem Fre 24 44 i 2

ake
jar 0.3
signal is derived from a crystal oscillater An AM transmitter is modulated by two Pp, =8 Kw OZ)
with a temperature coefficient of plus 1 | 2
sine waves at 1.5 kHz and 2.5 kHz, with
ppm per degree Centigrade and it modulations of 20 percent and 80 “The power of one sideband is 11 W.
Py =68,36 kW
transmits at exactly 150 MHz at 20 percent respectively, Calculate the
degrees Centigrade, find the effective modulation index.
transmitting frequency Weg) A The total transmitted power is
6.36 kW,
An AM broadcast transmitter has a
Platiaténny: sideband power output of 5 kW.
Pataticma:
Calculate the carner power if the
met = elective modulation index or total moctulation modulation index is 0.5.
{y= transmitting frequency (Hz) inclex
1, = transmit frequency @ 20°C (Hz) m) &m: = modulation index of the twa signal
k= temperature coefficient

5-16 5-17
The Question Bank Series — Communications Engineering (Volume 2 - Problems) Question Bank § — Amplitude Modulation (AM)

me _ (125)
p,= p= (i)
edith We) ep= (Oy
REr= "3(50)
Vo SkHZ = SV) zene
An LC tuned circuit is used to detect an
AM radio frequency band ranging from
P, = 62.5W PEP =1W
From

535 to 1695 kHz. What is the bandwidth


m= Sm
at 1695, if its bandwidth is 12 KHz at 535 -- The audio power supplied is 62.5 W. -. The PEP across the load is 1 W. Ve
kHz? m = 0.85 @ 1.25 kHz
_ Vi asks
PROBLEM 52 at) 1 er my
Solution: Ve
Past ECE Board Problem | GB radio wave signal is developed using 0.854, = Vb penne
BW = Bandwidth (Hz)
What is the range of the upper sideband a full-carrier DSB-AM. For high fidelity,
of an amplitude modulated signal whose the system requires a frequency 1
L = Inductance (H) carrier is 2.5 MHz and the intelligence response from 50 Hzto 15: kHz. What
_ Vo sitte a (Vs 25htie)
Mm:
C= Capacitance (F) component is 50 Hz to 20: kHz? bandwidth would itused to satisfy the =r Ve
requirement?
Solution:
eesti Flotations:
Phetiattiiaii:
" 2rJLC
USB = upper sideband frequency (Hz)
f= carrier frequency (Hz} BW = bandeadth (Hz)
fa = modulating frequency (Hz) fignay = Maximum modulating frequency (Hz) “>The modulation index at 2.5 kHz is
BW, = 12kH2| | 1695 Solution
0.426,
ba) \ 535 ) Solution:

BW, = 21.38kHz USB =f. +f, BW =2f, = 2(15 kHz) aL ed)


USB = 2.5 MHz + (50 Hz to 20 KHz) BW =30 kHz In.an AM transmitter, the final RF power
*: The bandwidth at 1695 kHz is USB = 2.50005 MHz to 2.52 MHz amplifiers high-level modulated. The de
24.36 kHz “The bandwidth needed for high supply voltage is 46 V with a total
~The upper sideband frequency is fidelity is 30 KHz. current of 3.5 4, How much AF power is
from 2.50005 MHz to 2.62 MHz. required for 100% modulation?
PROBLEM 51
A Class G collector modulated amplifier PROBLEM 55
Pua tiene:
has an output carrier power of 100'W ed)
)A ee) | The frequency response of an-audio
and an efficiency of 80%. How much “signal that amplitude modulates the
audio power must be supplied to the An SSB transmitter generates USE Py = input power. (WV)

bibl
signal with Voc. = 10 V, Determine the Carrier signal is 6 dB down at 2.5 KHz Ve = supply voltage (V)
amplifier for 100% modulation? from its level at 1.25 kHz. What will be
peak envelope power across a 50 0) [= input current (A)
the modulation index due to a 2.5 kHz P, = AF power (W)

iq
load resistance.
Motations: signal with the same signal at the input,
ifa 1.25 KHz signal modulates the Solution:
Plotatinns: carer to 85%’?
P= supplied power (W)
P, = output power without modulation (\W) P,, = Vel = (48)(3.5)
P, =ae alidio power (W PEP = peak envelope power (W)
My V5 = AM wave peak voltage (\V)
Hotalions: ., = 168 W
re ee,
m= modulation index A. = load resistance (€2) Vetaiie, Vi cane = modulating voltage (V) For 100% modulation, AF modulating
Ve = carrier voltage (¥) power is one half the input power,
Solution: = modulation index
Solution:

"ep a
Fe Ve
pep — Now — Me
2 2
Solution: p, -Pa
2
_ 1682
P,_ 100
Pp, =-2=—— = 125W
Reet Vesa is 6 dB down from its level at 1.25 P, =84W
4 08 KHz, therefore;
5-18 5-19
The Question Bank Series —- Communications Engineering (Volume 2 - Problems) Question Bank 5 = Amplitude Modulation (AM)

“The audio power required for 100% solution: Vlotations:


modulation Is 84 W. iN a id
Y= Vippp 178 Pj = total power transmitted (WW) A.CB transmitter supplies a-total power
p 2 3 Pecg = Single sideband power (W) of 200 W to an aerial, Supposed the
PROBLEM 57 V, = 89 P= carrier-power (W) camer is modulated to a depth of 70%
An AM transmitter supplies 20 KW of m= modulation index using DSB-FC AM. By how much will
carrier power to-a 75 ohms load. Ving = 0.707¥, = 0.707(B9V) the signal-to-noise ratio be improved if
Calculate the maximum instantaneous Vin, = 62.92V Solution: the modulation technique used is SSB
voltage the will be developed across the instead of DSBFC AM?
Vv (62.9)° for SSB system
load. FER Smears’
R 75 ==ae
P.
Psa 4 Flotation:
PEP =52.8 W
Hatations:
“assume m= 1 for worst-case condition
“ The peak envelope power developed
Pog = total sideband power (W)
P= camier power (W) across the load is 52.8 W. P.= a{ fae | = 4(100) = 400 W P.= carrier power (ht!)
Venus = carrier voltage (AMS) (V) m S/N = signal-to-noise power ratio (dB)

poets St aq
A = load resistance (2) For OSBFC system
Solution:

is
PROBLEM 59 iz
Solution:
Past ECE Board Problem
P. =P. i: =| = 400(1+0.5)
§
What oscillator frequency is needed to
The maximum instantaneous voltage that
heterodyne 626 KHz up to 3:58 MHz?
P, = 600W
will be developed across the load is:.equal
For SSB system
to the peak voltage at the load.
-. The power needed for DSBFC
P.
2
Hotatrones:
transmission is 600 W. Poche STS (0.7) 2 =
200

ieee
P. = man
£, =loral oscillator frequency (Hz) P., = 24.5 W
git)
=) 44a

jaf
f, = signal frequency (Hz)
Vevansy = yPER = /(20 x 10°)(75) { = intermediate [requency (Hz) (s
(a). 10log24.5
og 24.5 —Nyp
A carrier signal is simultaneously
Vesaaas) = 1225 V modulated by two sine waves at 1.5 kHz
Vo =V2(Veiaus))
Solution:
and 0.8 kHz with modulation index 0.3
and 0.6 respectively. Calculate the total
ae
5
—| 4 dB- Ng
=14dB-N,
Assuming high side injection is used
V, =/2 (1225) =1732V modulation index.
For high-side injection For DSB system
“The maximum instantaneous voltage Ratt

Seeks
Netatians: 2 ie

is 1732 V.
f.= 626 kHz + 3.58 MHz
S=P,= P.{1 =) = 200(1- 9%)
f= modulation index \
f, = 4.206 MHz Mr = effective modulation index = 249\W

oo
dt)
1A eT 3
“Solution: = 10l0g249 Ny = 24dB-N,,

zln
What is the peak envelope power (PEP) “ The local oscillator frequency is
ofa SSB transmitter that produces a 4,206 MHz, da

peak-to-peak voltage of 174 V across a m, =m? +m,


Jalsa

zlo
a
75 ohms antenna load?
PROBLEM 60 m, = (0.3) + (0.6) - 067
An SSB system requires 100 W of = (24—N)-(14-N)=10 dB

Zz\o
=
transmitted power for reliable
Vatatioia: * The effective modulation Index is
transmission, How much power is
0.67.
needed if DSBFC is used instead of “The signal-to-noise ratio
PEP = peak envelope power
SSB? Improvement is 10 dB.
Vp-p = peak-to-feah waltage
Vp = peak vattage
‘The Question Bank Series — Communications Engineering (Volume 2 - Problems) Question Bank § - Amplitude Modulation (AM)

-- The local oscillator frequency is eae) yy


at] AS ie] 4.99837 MHz. Past ECE Board Problem
Past ECE Board Problem A transistor RF power amplifier
What is the power in one sideband of an operating class C is designed to
AM signal whose carrier power is 300
a At produce 40 W output with a supply
W, with 80 percent modulation? Past ECE Board Problem voltage of 60 V. Ifthe efficiency is 70%,
What will be the total sideband power of | what is the average collector current?
the AM transmitting station whose

ry
Vlatutions: camer power |s 1200 Wand a m=0.78
Vindteatiits:
modulation
of 95%?
Poa = sideband power (WW) “<The modulation index is 0.78.
P.= carrier power (WW) P,= supplied power (W)
m= modulation index Flatat Inns: P, = output power without modulation (W)
P, = audio power ('W)
PROBLEM 69
Solution: Pegy = total sideband power (iW) t= elficency
P. = carrier power (W) m = modulation indéx A certain amateur radio station is tuned
m = modulation index at 200 KHz with an image frequency at
460 KHz, Determine the intermediate
_ Solution:

(iss
Solution: frequency of the receiver.

Pay = (2201 (20008) 4 wy


2

m'P, _ (0.95) (1200)


Po.
3)
A0W
oro Notations:
Pet = 2 3 R, = 57:14W
«: Power in one of the sideband is 48 W. Popy = 547-5 W
{4 = frequency of the image signal (Hz)
{, = radio frequency (Hz)
o =)
| = intermediate frequency {Hz)
AED ET “ The total sideband power is 641.5 W. i =952.4 mA
Solution:
A filtertype SSB generator uses an - The average collector current is
ideal bandpass filter with a center tit): a eT) 952.4 mA.
frequency of 5.000 MHz and 4
bandwidth of 2.7 kHz. What frequency An AM signal has a carer frequency of ff, 460-200
should be used for the carrier oscillator 115 MHz modulated by a single sine
wave. The modulated signal developed 1ST} 2 2
if the generator is to produce a USB
signal with a baseband frequency a maximum positive envelope voltage of la fadio technician measure 350 V
response having a lower limit of 280 8 Vand a minimum of 2 V. Whatis the without modulation and 400-V with
Hz? magnitude of the peak voltage of the modulation at the outputof an AM *: The intermediate frequency of the
unmodulated carrier? transmitter. with 80 ohms resistive load receiver is 130 kHz.
using true-RMS reading meter. What is
Hoatattons: the modulation index of the signal?
Matations; PROBLEM 70
fost = carrier frequency (Hz) Past ECE Board Problem
E-= carrier voltage (\V) Vistatjen
BW = bandwidth (Hz) What is the local oscillator frequency
4, = lower limit irequency of the signal (Hz) Er = maximum envelope voltage (V')
Eph = menimum envelope voltage (V) P= total output pawer (WW) range in commercial AM broadcast if it
P= carrier power (W) is equal to 455 kHz?
Solution:
Solution: E; = voltage with modulation (\/)
E. = voltage without madulation (V) Hotations:
pret +(e ‘ .] Bee
ae cee ™= modulation index
o 3 1]
L= local oscillator frequency (Hz)
eae E.=5¥V Solution: f= signal frequency (Hz)
f=
osc to c + 280K]
2 f= intermediate frequency (Hz)

foe = 4.99837MHz *: The peak amplitude of the carrier is


5.
5-22
The Question Bank Series - Communications Engineering (Volume 2 - Problems) Question Bank 5 — Amplitude Modulation (AM)

Solution: Flotations: - 400% 10° Aa “The impedance seen looking into the
power amplifier is 1.83 9.
+ At (100
L=trh DR= Dynamic range log [aa
For f, = 540 kHz Py GV)= strong signal
Pa & Vo= weak signal PROBLEM 76
f, = 540+ 455 “The sensitivity of the recelver is 1 pV. Past ECE Board Problem
f, = 995 kHz Solution: Calculate the highest and lowest
For f, =1600 kHz frequenciés for a subcarrier signal al
_ Strong signal PROBLEM 74 629 KHz with bandwidth of + or —500
f, = 1600+.455
Weak signal What is the shape factor of the filter if 10 kHz.
f, = 2055 kHz kHz bandwidth Is measured below 6 dB
RP
(| == 20g ¥Ms ) and 30 kHz below 60 dB?
*- The local oscillator frequency is 995 P, a Vy Fatatiix:
kHz to 2065 kHz.
Hlitirtenina:
Mi = tog tata) bo, =subcamier frequency (Hz)
Ve 20 fi = highest frequency (Hz)
ay:4 a ed. 70 SP =shape factor {= lowest frequency (Hz)
i (0. 5 w)(toa"d1 2) B= Bandwidth (Hz)
A superheterodyne receiver tunes at the
frequency range from 25 to 50 MHz “Solution: Solution:
V,= 1,58 mV
What is the IF frequency of the receiver
ifthe range of the local oscillator is
10 MHz to. 35 MHz. -. The strongest signal that can be
SF uf=iBocas f=fup
+ BW
f; =629+ 500-1129 KHz
present without blocking is 1.68 mV. 30 kHz
f, =629- 800 = 129 kHz
Motatiens: 40 kHz
PROBLEM 73 “The highest and lowest frequency is
f, = local oscillator frequency (Hz) -. The shape factor of the filter is 3. 41129 kHz and 129 kHz respectively.
A receiver can handle a maximum
f= Signal frequency (Hz} signal level of 100 mV without
{i = intermediate frequency (Hz) overloading. If the AGC range in
at] ee itn): ag
decibels is 100 dB, what is the
Solution: sensitivity of the receiver? Past ECE Board Problem A tuned transformer has a Keen = 1. 5k:
‘An AM transmitter uses high-level operating at a frequency of 455 KHz.
af, +f | modulation. The RF power amplifier The primary Q is 65 and the secondary
Vloterticnes: draws 12 A froma 22 V supply, putting
since fy > fq thus low side injection is used Q is 75. What is the bandwidth of the
oul a carrier power of 140 watts, What tuned transformer?
Ge h=f OR = Dynamic range (dB) | impedance would be seen at the
f=f.-( View = Strongest sagnal (\’) modulation transformer secondary’?
§=25 MHz—10 MHz Vein = Weakeast signal (\) Flatations:

f= 15 MHz
Solution: Flotiiticna: = critical coupling factor
Koo= optim coupling factor
“Intermediate frequency used is = impedance (£2)
The sensitivity of the receiver is equal to Q,,= quality factor at the primary
15 MHz.
the weakest that the receiver can faithfully Vec = supply voltage (V) Q, = quality factor at the secondary
amplify, |: = collector current (A) BW = bandwidth of the transtormer (Hz}
i, = resonant frequency of the tuned circuit (Hz)
PROBLEM 72 DR = 20g st ) Solution;
A receiver has a sensitivity of 0.5 pV Vinin-
Solution:
and a blocking dynamic range of 70 dB.

eo")
What is the strongest signal that can be
present along with a 0.5 LV signal
without blocking taking place? 20 y= 22V_
a2 = 1.830
22

5-24
The Question Bank Series— Communications Engineering (Volume 2 - Problems) Question Bank 5— Amplitude Modulation (AM)
bid h8)1
ke =< — = 0.0143 Bi _) 2
(1000) 950 W Hotattons:

(65)(75) 4 transmitted signal 5 meters in


wavelength is received by an antenna 4 Ver = source voltage: (VY)
Keg = 1.5Ke coll having a 50-2 resistance and a 0.01 Pr="carrier power (\W)
+The sideband power Is 250 W. Ri = load Impedance (£2)
Koy = 1-5(0.0143)= 0.0286 H inductance. What is the capacitance
of the tuner shunting the antenna coil at
BW = Koil
BW = 0.0286/455 kHz) = 13,013 Hz
this point? ate) 0 ed Solution:
A certain radio transmitter has a carrier yz
Motarienc: power output.of 150 Wand an efficiancy foaee
= The bandwidth of the tuned
transformer is 13,013 Hz. of 80%. How much power |s dissipated 2R
i, = resonance frequency (Hz) in the transistor with BO% modulation? V2 (12)
L = inductance (Hi) ets Se,
PROBLEM 78 C= capacitance (F} fe * 2P, 2(85)
Patations: R, =0.65 2
A 12 W of power is being supplied in the solution:
modulating amplifier for 100 percent Py = audio power (W)
“The load impedance is 0.86 ©1.
modulation. What is the carrier power P, = OC power (without modulation) (W)
output of the radia transmitter at an = catrier power (4)
efficiency of 75507 Pe = output power (1) PROBLEM 83
Hes ae 1 1 = efficiency
Hutatipiis:
(2nf,)PL [ 2n(60 x 10°) (0.01) Po = power dissipated (W) An AM mobile transmitter supplies 5 kW
of carrier powerto a 50 ohms load. The
C=0,704 F Solution: carrier signal is modulated by a3 kHz
Pa = audho power '(W) sine wave toa depth of 80 % ata
FP, = DC power (without modulation) (W) ¢ 3x 10° frequency of 10. MHz. Determine the
Pa = olliput carrier power (W}
where: f= =
i ; —=60 0 MH MHz P, =P, -P, sideband power.
eri

1 = elliclency +: The capacitance of the tuner is Pa = 247.5W —198W = 48.5W


0.704 fF.
Mostetions:
Solution:

aL Pen = sideband power (WW)


From ‘oat °
P, = (150) 4.080 =i98W Pc = carrier power (W)
me Past ECE Board Problem @ m= modulation index
Psy SBO =—P,
5 TC Assuming 100% modulation, what would
be the transmitted power in the upper P, 198
m=1, Py = Pea and Pe = Ps
sideband of an AM signal if the carrier
P= y )= OB
= 247. SW Solution: Pp
Mi

power |s 1000 watts? Poa = m=


P. =0/5P_
~ The power dissipated In the 1a
P, aO5
FE:
Minti finns
transistor is 49.5 W. Pea = (0 ay _ 0 4600
12
P= Gee 24 Pyro = upper sideband power (it!)
ba 8) 18S) 9d
5 Pe = carrier power (WW) The total sideband power is 1600 W.
E BS m= modulation index Past ECE Board Problem
Fe , An AM transmitter uses a high level
P= nP. Solution: modulation. The RF power amplifier at)
7 |
funs fram a 12 volt source, putting out a
Past ECE Board Problem
P, =0.75(24)=18W The transmitted power of the lower and Carrier power of 85 watts, with an
efficiency of 85%, what load impedance
Ifthe percentage modulation of an AM
upper sideband is equal. amplifier is 88% and the modulating
i$ required in order for it to deliver the signal is 1 volt, the carrier has an
© The output carrier power is 18 W. tated power? -
Riese m'Pe amplitude of
Usb ~ 4

5-26 5-27
The Question Bank Series —Communications Engineering (Volume 2- Problems) Question Bank 5—Amplitude Modulation (AM)

Flotations; Solution: Flotattona:


ae) Tet
E,, = peak modulating valtage (V) Pe = carrier power
Past ECE Board Problem
Ec = peak carrier voltage (WV) An AM signal has the following Ve = camier voltage
‘m= modulation index characteristics; the carrier frequency is n= modulation index
Solution: 150 MHz; the modulating signal
frequency is 3. KHz; the carrier voltage is Solution
m=ae™:
Est
£2 5m
ar ni
60 volts, where the modulating signal
voltage (E.,) is 30 volts. Find the paak
P, = (0.6)(50)(0.25) 2

voltage of the lower side frequency. Pp. =7.5W R= Meus


Eterna i R
0.88 .. The output power of the amplifier Is
Flotatiene 75W
The carrier voltage is.1.14 V eae =\(5 ¥10° (80) = 500 V
Viewsat) = peak voltage of the lower-sideband (VW)
\ ae =m...
Ec = carrier voltage (V) A Tt
bad te 1M Ded E. = peak modulating voltage (\/) Past ECE Board Problem
Vines = 0.8(500) = 400
m = rmorulation index An AM transmitter
is rated 100 W at 100
An AM receiver uses low-side Injection Ve = V2 (Vaws)
for the local oscillator, with an IF of 455 o6 modulation, How much pewer
kHz. The local oscillator is operating at
Solution: required for the carrier? Ve = V2 (400) = 585.7 V
2,1 MHz. What is the image frequency
m
oc
Vocak(LSF) 7 a 2
of the signal? Votithies: -. The peak voltage of the modulating
signal Is 565.7 V.
m

Veeik(L SF) =2 Py = total sideband power (W)


Hotatons:
30 Pe= carrier power (W)
Vonak (USF) Tat 13V m= modulation index PROBLEM 90
flo = local oscillator frequency (Hz)
f, = radio frequency (Hz) where:
Solution:
An AM signal has a carrier frequency of
{= intermediate frequency (Hz)
EE, =me, 3 MHZ and amplitude of2 Vpea,. What is

ort)
f,, = frequency of the image signal (Hz) the sideband frequency if the bandwidth
of the AM signal is 20 KHz?
Solution: ' .. The peak voltage of the lower side
frequency is 15 V.
with m=1 (100% modulation) Vita teas:
for low-side injection
P, a SPE
PROBLEM 87

hik
f=f,-fo fies = lower sideband frequency (Hz)
TOW
f=fo+h T= isu = upper sideband frequency (Hz)
f= 2.140.455 = 2.555 MHz
The average collector current of a a5" t: = carrier frequency (Hz)
transistor RF amplifier operating Class P, = 66.67 W

Cat
Cis 250 mA. If the efficiency of the BIN = barelwidth (Hz)
for low-side injection
amplifier is 60% and operating at 50 V
ol 2f
supply, what is the power output of the The carrier power is 66.67 W. Solution:
fy = 2.555 MH2— 2(0.455 MHz) amplifier? BW
{ise =f, - =r
f, =1,645 MHz PROBLEM 89
Flatadinai:
ficp = 9-0.02=2.98 MHz
An AM mobile transmitter supplies 5 kW
-. The image frequency is 1.645 MHz. of carrier power to a-50 ohms load, The BW
fica =f, +——
Ps=DC power (no modulation) (W} Camier signal is modulated by a3 KHz 2
Py = oliiput power (WW) sine wave to a depth of 80 % ata fieg= 3 + 0.002 =3.02 MHz
Vee = collector voltage (V) frequency of 10 MHz, Determine the
Ic = collector current (A) peak voltage of the modulating signal. ©. The lower and upper sideband
1 = efficiency frequency is 2.98 MHz and 3.02 MHz
respectively.

5-28 5-29
The Question Bank Series —Communications Engineering (Volume 2 - Problems) Question Bank § — Amplitude Modulation (AM)

a1 ea @m=08 Solution: PROBLEM 96


2
Part ECE Board Problem : An AM transmitter (DSBFC) transmits
PrP )
At 100 % modulation, the sum of the P, = Ve 40: kW with no modulation. How much
effective voltages in both sidebands is z p, = 15(8)=120 W power will it transmit if modulation index
equal to % of the unmodulated is 80 %7
camer voltage. P, = 40 44028) | 52.8 kW m
z
Fa =tans

Fy Hotcrtinas:
Flatations:
_ © The total power transmitted is P, (288) 120) = 43.35W P; = total output power (W)
62.8 kW.
Peay = total sideband power (W)) Pe = carrier power (W)
Pr = carrier power: (WW) J The audio power or power supplied m = modulation index
Vea = total sideband voltage [V) a) he by the modulating amplifier is
Ve = carrier voltage (V) 43.35 W. Solution:
Past ECE Board Problem
m= modulation index 2
100 % modulation in AM means a P,= Pe 1» : |
Mm

Solution:
corresponding increase in total power by ae te)
2 2
Peat
=2 m
Pos
g
—oB
R
NV; Past ECE Board Problem
A collector modulated class C amplifier
P, = 40 xt of | = 52.8 KW
‘ i
Hotactiona: has a carrier output power of 100 W,
Vege ic m* Ve and with an efficiency of 60%, calculate
FRE uae Pr = total sideband power (WW) the transistor power dissipation, «The total power transmitted Is
Pe = carrier power (WW) assuming that all the power dissipation 62.8 kW.
with m=1 (100% modulation)
m= modulation index that occurs comes from the transistor.
1
Ven z= Ne r
Solution:
PROBLEM :97
Ve = 70.7 V. = 719M. Votitanes: Past ECE Board Problem
m A filter-type SSB generator uses an
P, -F(10 Po = output power without modulation (W) ideal bandpass filter with a center
«The sum of the two sideband
P; = source power (WW) frequency of 5.0 MHz and a bandwidth
voltages is 70.7 % of the
(@ 100% modulation, m=1 Ra = power dissipation (W) of 2.7 KHz. What frequency should be
unmodulated carrier voltage.
Py =1.5P,5 1] = eficiency used for the camer. oscillator if the
generator is to produce a USB signal
Py = Pe +0.5P.
with a baseband frequency response
PROBLEM 92 Solution:
having a lower limit of 280 Hz?
An AM transmitter (DSBFC) transmits ”. The total power increases by
40 KW with no modulation. How much 50 percent. sa
- P=
P where: P= Fy.+P,
y Bene orFo
power will it transmit if modulation index 6
Votaticus:
increases by 60 %? ‘
a Pa +P, = <4
Py
f, = center frequency (Hz)
An AM transmitter delivers 100 W
to = carrier oscillator frequency (Hz)
Vlotuticns: 100
100 {io = local oscillator frequency (Hz)
carrier power, when the supply deliver 8
Afroma 15V. How much power must be 08 {i = lower limit frequency; (Hz)
Py = total output power (W) P, = 66.67 W
Pc = carrier power (W) supplied by the modulating amplifier far
85% modulation? Solution:
m= modulation index
“. The power dissipated by the
BW
Solution: Flotations:
transistor is 66.67 W.
ho=-5 HfL

@m=0 Ps = OC power (no modulation) (\W’) 2.7-« 10°


Py = audio power (W) +280 = 1630 Hz
Loi
Pr =Pe Voc = collector voltae (V}
fa S fon * fo
le = cofector current (Aj
5-30 5-31
The Question Bank Series — Communications Engineering (Volume 2 - Problems) Question Bank 5 — Amplitude Modulation (AM)

foo = fa — fo PROBLEM 99 Flatations:


f, = MHz- 1630 Hz Past ECE Board Problem
Three audio waves with 100, 200 and Py = total output power (i)
f,, = 4.99837 MHz = 4998.37 kHz
300 volts amplitude respectively, P; = carrier power (W)
simullaneously modulate a 450 volts m= modulation index
* The local oscillator frequency is
carrier. What is the total percent of
4998.37 kHz. Solution:
modulation of the AM wave?

eg) 4 Platt liana:

The voice signal is modulated using


SSBSC technique with two audio tones MM = percentage of modulation
having frequencies of 700 and 1200 Hz. Mi, Me, m3 = modulation indices
The modulated signal has a suppressed my = tolal modulation index
camer frequency in the IF receiver of 3 E. = carer voliage (V)
Mz. The receiver
is mistuned causes E,, = modulating voltage (V’)
the BFO to generate 2.995 MHz
Determine the output frequencies of the Solution:
demodulator if the signal is USB.
my = mt — 1 = 0.70" -0.5"
M=m, x 100% . m, =0.5
Fotations: M=(0.83)(100%) = 83%
-, The modulation index is 0.5.
where:
{, = audio frequency {Hz}
{|= intermediate frequericy (Hz) in; = im? + m3? +m,"
at)
foro = BFO frequency (Hz)
fs = signal component (Hz) Em 100 995
E. 450V
What is the maximum suppression for a
fen’ = output frequency of the demodulator (Hz) phase error of 3°?
m= Enz 200Y "9 44
Solution: E. 450V
Flajationa:
Ena 400V
fe = ffs m, = =———- =.0.67
7 Ep 450V = deviation from a perfect 90° phase shit or
Signal components:
phase error
3 MHz = 700 Hz = 2.9993 MHz my = \(0.22)" +(0.44)° +.(0.67)°
and my = 0.83 Solution:
a
nd
3.MHz - 1200 Hz = 2.9988 MHz =H
dB suppression = 20 log cot (?)
-. The percentage of modulation is
foam = loro —fs 83%. if
dB supression = 20 log cat (5)
When mix with the BFO frequency of
2.9995 MHz the output frequency is: ae d8 supression = 31.64 dB
2.9995 MHz - 2.9993 = 200 Hz
An AM broadcast transmitter output
2.9995 MHz - 2.9988 = 700 Hz poweris 15 kW if the carrier is “The maximum suppression is
‘simultaneously modulated with anather 31.64 dB.
-. The output frequencies of the sinewave at. 50% modulation. What is
demodulator are 200 and 700 Hz. the modulation index of the ather
sinewave if the carrier power 12 KW?
The Question Bank Series — Communications Engineering (Volume 2 - Problems)

| Votes

*/'
pee
Je
ore inv
f
7
Question Bank 6
FREQUENCY MODULATION

5-344
Question Bank 6 - Frequency Modulation (FM)

PROBLEM 1 PROBLEM 3
Using Carson's rule. what is the The operating frequency of an FM
bandwidth of an FM signal with 5 KHz transmitter is 168,96 MHz. Calculate the
maximum frequency deviation and 2.5° frequency of the carrier crystal oscillator
kHz maximum modulating frequency? ifituses three frequency multipliers — a
doubler, a tripler and a quadrupler.

Putations:

rf
Hotation:
BW = Bancvidth (Hz)

an i
Sem = Maximum frequency deviation (Hz) {, = oscillator operating trequency (Hz)
fina = Maxinum modulating frequency (Hz)
Solution:
Solution:
The total multiplication produced by the
frequency multipliers = 2x 3x4 = 24,
BW = 2(Smax a frien) )

BW, =2(5 kHz+ 2.5 KHz) f, = 10888 - 7.04 Miz

er
24

RB
BW =15 kHz

8
+, The bandwidth of FM signal is
The frequency of the carrier crystal

8
oscillator is 7.04 MHz.
“15 kHz.

dite)
] Nae)
A
What frequency deviation is caused by
Calculate the maximum bandwidth
noise in an FM receiver which has. an.
required of a standard FM broadcast
input S/N of 2.8 and the modulating
station.
frequency of 1.5 kHz?

Hotationas
Miotartionn:
BW = Bandwedth (Hz)
6 = frequency deviation (Hz}
Bsn = maximum frequency deviation (Hz) {, = modulating Irequency (Hz)
_Fojrant = maximum modulating frequency (Hz) = phase deviation (rad)
Solution: S/N = signal-to-noise ratio

By Carson's Rule: Solution:

‘BW = 2( Srna * fined) b=4f,


BW = 2(75 kHz + 15 kHz)= 180 kHz B= (0.3652)(1.5x107)
where : 6=547.8 Hz
Fora standard FM broadcast where:
‘Sma = 7 5 KHz
sind rT = Vig —=§ —N
a =15 kHz

g=sinein (FN
~. The maximum bandwidth of standard
FM is 180 kHz.

6-3
Question Bank 6 - Frequency Modulation (FM)
The Question Bank Series — Communications Engineering (Volume 2'- Problems)
PROBLEM 10
yea eae Solution:
g=sin’ 1) Past ECE Board Problem
What isthe rms output voltage ofa PLL fe = 2(fre —f) A system uses a deviation of 100 KHz
FM detector ifituses VCO with
= 20.92 proportionality constant of 200 KHzV f= 2(14 MHz -10 MHz} and a modulating frequency of 15 kHz.
What is the approximate bandwidth?
= 0.3652 rad and if it recelves-an FM signal witha fi, = 8 MHz
deviation of 65 KHz sinewave
.. The frequency deviation caused by modulated? Hotations:
Note: The inch range 6 approdmately teice the
noise is 547.8 Hz. diference between the frequency where lock is
lost and the free-running frequency. BW = Banchwidth (Hz)
Platatinna:
Ona: = Masimum frequency deviation (Hz)
ye Ey = PLL output voltage (V) . The lock range is 8 MHz. Entre) = Maximum modulating frequency (He)
In FM broadcasting, what is the carrier 6 = frequency deviation (Hz)
frequency in MHz of a station witha Solution;
ky = demodulator sensitivity (H2/V) PROBLEM 9
channel number of 285?
Solution:
| Past ECE Board Problem By Carson's tule,
A phase modulator has kp = 2 rad/V BW = 2(8+{,)=2(100 kHz +15 KHz}
Flatationa: What rms voltage ofa sine wave would
6 cause a peak phase deviation of 30 BW = 230 kHz
eral = ky,
N = channel number degrees?
fe = camer frequency (MHz) . The approximate bandwidth is
65 230 kHz.
es pisk = 200 = D425

tonSee =
Solution; Flivtatian.:
= va i

Ee = peak modulating voltage (\¥) 1


£ -B+arg = peak phase deviation (rad) Past ECE Board Problem
285. =023V k= sensitivity (rad) A phase-locked loop has a VCO with a
=
aan ar
free-running frequency of 14 MHz. As.
Solution: the frequency of the reference input is
f, = 104:9 MHz -. The rms output voltage of the PLL is gradually raised from zero, the loop
0.23 V. locks at 12 MHz and comes out of lock
«The carrier frequency is 104.59 MHz. again at 18 MHz. Calculate the capture
range.
a1H
PROBLEM 6 Past ECE Board Problem
In FM broadcasting; whatis the channel A phase-locked loop has a VCO wilh a
Hoatatinns:
number of the station with a given free-running frequency of 10 MHz. As
carrier of 99.1 MHz? the frequency of the reference input is
gradually raised from zero, the loop {re = free-running frequency (Hz)
locks at 8 MHz and comes out of lock fig = lock range {Hz}
Pte ianan: again at 14 MHz. Determine the lock fie = Capture range (Hz)
range. 1) = input Irequency (Hz)
N=channel number
f, = carrier frequency (MHz) Solution:
Hotatrons

Solution: From
fa = free-running frequency (Hz) fea
fin = lock range (Hz) where: fe tntoe:
N=5(f. 47.9)
{n= capture range (Hz)
N=5(89.1- 47.9)
{= Input frequency where lock is lost (Hz)
$= 30' =0.52 rad fog = 2(14-12)= 4 MHz
N=256
-. The rms voltage is 0.184 V. 2, The capture range of the PLL is
- The channel number is 256. 4 MHz.

6-5
The Question Bank Series- Communications Engineering (Volume 2 - Problems) Question Bank 6 - Frequency Modulation (FM)

Note: The capture range Is approximately twice the Solution: Solution: ed 8) J


difference between the free-running frequency and Ifthe frequency fed to the pre-amplifier
the frequency at which lock is first achieved: ofa basic transmitter with multipliers is
oun = 22 x 400% composed of a pair of triplers and a

_cS.
mie

_12 kHz doubler multipliers is 198 MHz. What


PROBLEM 12 frequency should the oscillater operate’?
Past ECE Board Problem ~ AkHz
What is the frequency swing of an FM
broadcast transmitter when modulated “. The modulation Index ofa standard tl Matationa:
FM broadcast is 3.
80%? CS. = 2MB nae foo = Output of muttiplier circuits (Hz)
CS, = 2(0.80)(75)=120 kHz f; =output of oscillator circuits (Hz)
Mot aches? ate) TNS ae]
Past ECE Board Problem “The carrier swing is 120 kHz. Solution:
SoM = percent modulation What is the modulation index of an FM
Gas = actual frequency deviation (Hz) transmitter whose frequency deviation is
TT
Ti] aie]
Gra = maxsmum frequency deviation (Hz) 50 kHz, while its audio frequency is 10
kHz? An FM signal has a center frequency of
Solution: 100 MHz but is swinging between
Hota teres:
100.01 MHz and 99.999 MHz ata rate Multiplier Circuits
gna = Part. 41.00% of 100 times per second. What is the
Snag m = modulation index modulation index of the signal? four (40)(3'3)(2) = 198 MHz
Ge Bn
rate(96M) Byer = actual frequency deviation (Hz)
= 100% f, = modulating frequency (Hz) Fstutions:
_ (75 kt12)(80%)
Siig 100%, Solution: ih = modulation index
B,q:= 60 kHz 0 = frequency deviation (Hz)
». The operating frequency of the
m= Sea fy = modulating frequency (Hz)
oscillator is 71 MHz.
where for FM broadcast, in

_ 50 kHz Solution:
Bmax = 75 KHz
“40 kHz =8
The intelligence frequency is equal te the
~. The frequency swing of an FM rate of change of the carrier frequency.
te TA
broadcast transmitter is 60 kHz.
*. The modulation index of an FM
Fer commercial FM broadcasting the
transmitter is 5,
Therefore f= 100-Hz. maximum permissible range in
modulation Index is 5-— 2500. Whatis
eet the maximum bandwidth needed for this
PROBLEM 15 (| system?
Past ECE Board Problem
Determine the modulation index of a
Past ECE Board Problem maps es
The carrier swing necessary to provide EE
standard FM broadcast having a Plotations:
hypothetical maximum carrier frequency 80% medulation in the FM broadcasting _ [soo
rer |
band is MHz
deviation of +12 KHz and a maximum BW = Bandwidth (Hz)
modulating frequency of 4 KHz.
——_—-___
= 100
400 Hz 6 = frequency deviation (Hz)
Fleatuticins:
fa = modulating frequency (Hz)
Voatations:
“The modulation index Is 100.
Sh = percent modulation Solution:
m= modulation index O,a = actual frequency deviation (Hz)
Gna = maximum frequency deviation (Hz) Oras = maximum frequency deviation (Hz) For commercial FM broadcasting
fp = modulating frequency {Hz} 5, = 75 KHz

6-5
The Question Bank Series — Communications Engineering (Volume 2 - Problems) Question Bank 6 - Frequency Modulation (FM)

mo «The transmitting frequency is edit} aes ei) 10 ter


145.002175 MHz. The output signal of a PM demodulator
fm Calculate the frequency deviation of an
@{=30 Hz FM modulator that has k= 20 kHz/V is 0.54 V What is the sensitivity of the
and operates ata carrier frequency of demodulator if ihe phase deviation |s
m= 2500 ait: edt) 99.1 MHz modulated bya 2.5 Vis sine 307
BW=2(f, +5)=2(6) What FM channel is 107.5 MHz carrier? wave
BW =150 kHz Plotatioma:
@ f=
15 kHz Plotutinaa: Flotatioss
m=5
hy = sensitivity of the phase modulator (rad/V}
N= channel number 6 = frequency deviation (Hz) dh = phase deviation (rad)
BW =2(f,+8) t= carrier freqiiency (MHz) ky = sensitivity of the modulator (kKHz/V) Vo = peak amplitude of the modulating signal (V)
BW =2(154-75) Vin = Peak amplitude of the modulating signal (\V)
Solution; Solution;
BW =180 kHz Solution:
The maximum bandwidth is 190 kHz.
N=5(f, -47.9) Convert phase deviation fram degrees to
§=k,V,, = 20 kHz/V(3.54V) radian,
N=5(107.5-47.9)=298
6= 70.8 kHz
ba) -, The FM channel is 298 where:
360° =2n rad
.) 2 rad 30
Past ECE Board Problem
A portable radio transmitter has to
Vg = V2 (Mana) 40° = ——————_
360
operate at temperatures from -5 ed 0]: Te V,, = V2 (2:5) =3.54V ne
30° =—rad
ae
degrees to 35 degrees C. If its ‘signal is The instantaneous value of the
derived from a crystal oscillator with a “The frequency deviation of the FM
temperature coefficient of +1 ppm per
modulating signal to an FM modulator is
modulator is 70.8 kHz. Key
th ;
-2V, sensifivity ki = 25 KHz/V and
deogree centigrade, and it transmit at operating at a carrier frequency of BB
exactly 145 MHz at 20 degrees C, find
ue transmitting frequency at. 35 degrees
MHz. Determine the output frequency of
PROBLEM 23 [Ze
the modulator
k=" 05a = 0.97 rad
Find the deviation ratio of an FM
broadcast transmitter ifthe frequency of
Plotcetions:
Flotation: the modulating signal vanes from 30 Hz .. The sensitivity of the modulator is
to 20 kHz. 0.97 radiV
tig = output frequency of the modulator. (Hz)
fr = operating frequency at temperature T i, = carrier frequency (Hz)
1, = operating irequency at reference temperature T. ky = sensitivity (Hz/V)
k= temperature coefficent per degree centigrade
Vatatesirs: a CNS ee
En = modulating signal voltage (Vv)
OR = Deviation Ratio A PM transmitter produces a maximum
Solution: Solution: phase shift of 2/3 rad for a modulating
Sr = Maximum frequency deviation (Hz)
Tree = maximum modulating frequency {Hz} frequency of 300 Hz. VWhatis the
F=f rk (l=) i =f, +k,E, maximum frequency deviation of the
system?
fr, = 145 MHz + (1x10°)(145x10"}(35— 20) fq = (88x10°Hz)
+ (25x10°Hz/V)(-2v)
Solution:
f; =145.002175 MHz fu = 87.95 MHz Fhaausivate

Note: The term parts per million (pen) tells how many : The output frequ iy = FM modulation index
cycles (Hz) a crystal may differ from its designated % eee is 74 ae me 6 = frequency deviation (Hz)
frequency for every 1,000,000 Hz af frequency. A y fn = modulating frequency (Hz)
100-ppm deviation on a 10 MHz crystal means
“The deviation ratio of an FM ih = phase ceviation
that the actual frequency could be 10x 100 ppm
= 1000 He plus or minus 10 MHz, broadcast transmitter is 3.75.

6-8
The Question Bank Series —Communications Engineering (Volume 2 - Problems) Question Bank 6 - Frequency Modulation (FM)

Solution: ©. The peak phase deviation Is 4.5 rad, il miata:

From: f= power transmitted (WW!)


ie
a) A eae P,, Pi, Pe... = camer power,
first sideband power,
eats
An FM broadcast transmitter rated second sideband power ,.. (Wi)
ouipul power is 5 W. Whatis the peak | = Bessel coefficient of the sidebands
Fora PM system, modulation index is Voltage developed across a 50 ohms Vy = ,|(5)(50) = 15.8Vime Py = total transmitted power (W)
WV

equal to peak phase deviation, resistive load? P, = uniccounted power (W)


+ The RMS carrier voltage is 4.11 V.
Solution:
MM = trae Matetiong:

Then,
et)
1 a) m=3
Py = output power (1)
b,
is
=—
a
f Vy = peak voltage across the load (\/) Suppose the total power of an FM signal
J, =-0:26; J, =0.34 J, = 0.48;
R, = load resistance (€2) is TW, What is the power of the second J, =0.31
= fn?max sideband if the modulation index is 3? P,, =P 4 2(P.4P,4P,)
Solution: Jo= 0.26; J,= 0.34; J2= 0.49; Js = 0.94
qt

5 =(s00H2)( [x3) = 314 Hz Py =a,'Py + 2(d7P, + 5°P; + JP; )


Notations: P,, =(0.28)' (10)+
de

“The maximum frequency deviation of


the system is 314 Hz. . Py = total power transmitted (W) (2)](0.34)" (10) + (0.49) (10)+(0.31)'(1 0)|
Pc = carrier power (W) P. =9.712W
Vv, = ,/(5)(50)
= 15.8V,,, |= Bessel coefficient of the sidebands
Thus:
bait): 13 ed)
V, (peak)
=, /2 =22.34. Solution: PL =P; - Pr
‘| An FM communication system has a
sensitivity of kf = 3 kHz (VV. How much P =10-9.712-0:288 W
ae

phase deviation does it produce with a -, The peak voltage developed across From
sine wave input of 3 V peak ata the load Is 22.34 V. P, =P, [¥.? +2(J2 + dhe -)| +». The unaccounted power is 0.288 W.
frequency of 2 KHz?
Therefore:
ar Aad: PROBLEM 31
Plotutions:
An FM system has a modulation index P, = 2U,'P, A, certain FM station transmitted a
m = FM modulation index
of 3 and its corresponding Bessel
coefficient; Jo = -0:26; J; = 0.34:
P, = 2(0.49) (7)=34W modulating frequency of 25 kHz and a
frequency deviation of 75 KHz. . What is
6 = frequency deviation (Hz) J2 = 0.49; J; = 0.31. Find the RMS the approximate signal-to-noise ratio at
f, = modulating frequency (Hz) carrier voltage if the total power is 5W, Nowe Ps = Pe fad the | coefficients are considered.
the detector output if the signal-to-noise
= phase deviation (rad) developed across a 50 ohms resistive ratio at the Input to the receiver detector
load. :. The power of the second sideband is is 20 dB?
Solution: 3.4 W.

8=kV,, Vlanitiina; Flotatiors:


od tJ ef
5 = (3kHz/V)(3V)
= SkHz Vy = voltage level at the load (V/) V_ = signal vollage (\!)
An EM broadcast transmitter has a
Vc = voltage level of the carrier (V) V,, = noise voltage (V}
| = Bessel coefficient of the sidebands deviation of 25 KHz and a modulating
Fora PM sysiem, modulation index is frequency of 8 kHz, Calculate the power = peak phase deviation {rad}
equal to peak phase deviation. that is unaccounted, if the total power P; &;, 8, = peak frequency dewation (Hz)
Solution:
Is 10'W. Ja =-0.26; J; = 0.34; Js = 0.49; my = FM modulation index
a d;= 0.31 £, = modulating frequency (Hz)
m, 7 eee m =3.

§ kHz
Jo =-0:26, J) = 0.34; J5=0.49; Jy = 0.31
5 en th
¢ 2 KHz a ES My = dar = |do|Vp = (0.26)(15.8)=4.11¥
6-10 6-11
The Question Bank Series — Communications Engineering (Volume 2 - Problems)
Question Bank 6 - Frequency Modulation (FM)
Solution: “, The modulation index of an FM
» The modulation index of the signal is 6=95.05 MHz—95 MHz =50 kHz
broadcast station is 5,
25, re SO'kKHe
kHz =4p
(Mel (SIN),
f, SkHz.
lv } 20 PROBLEM 33
a tae ED
(wt) “e351
V,
logs
20
= An-FM signal with modulation index of4
and modulating frequency of 8 kHz.
What is the frequency deviation of the
| ‘The output frequency of the modulator
increases by 50, kHz. What is the de
“. The modulation index of the signal is
40.

Me =04 signal? voltage applied to the FM modulator, if


vy | the deviation sensitivity is 25 KHzry? Ula Ky)
since
V, > V, APM signal has a RMS signal of 2 Vat
PTijta tiers
Vv =Fiotatinnk: a frequency of 3/kHz. If the modulation
a =V, =0\trad
fa index of the signal is 15, determine the
my = FM modulation index
B= trequency deviation (Hz) sensitivity of the modulator,
Thus! 6 = frequency deviation (Hz)
{= modulating frequency (Hz) k= sensitivity (KHz/V}
Mn
= 0.4 i, = peak amplitude of the modulating signal (\/) Vlatutions:
6, =maf, Solution:
Solution:
But noise can interfere only if the m, = PM modulation index
frequency of nolse is the’same as the Ra 6
hy, = peak phase deviation (rad)
f tm vo a kg = sensitivity’ (radlV)
frequency of the signal thus,
Som,ftm V, = peak amplitude of the modulating signal (¥)
5, = Mnf, = (0,1)(25 KHz)= 2 5.kH2
é
§=4(B KHz) = 32 kHz Mate Solution:
MG te khe
Vy a) 2215 kHz .. The frequency deviation is 32 kHz. 50k kHz
(s ~ 25
BS kKHz/V m= 9, =KaVin
(als = 20log30-= 29.5 dB mi,
Jor atc
Pa hee -. The de voltage appliad to the FM PN
modulator is 2 V.
The approximate signal-to-noise ratio A.certain FM modulator has ky= 25 13.
at the detector outputis 34 dB, kHz, Suppose the peak amplitude of
the modulating signalis 3V ata path MeL)
frequency of 3 kHz, determine the
PROBLEM 32 modulation index of the signal, The maximum instantaneous frequency -. The sensitivity of the modulator is
of the modulated signal is 95.05 MHz. If 3.64 rad/V.
An FM broadcast station transmits a 3 a5 kHz signal modulates a 95 MHz
kHz test tone, and a frequency deviation Solution: cartler, calculate the modulation Index of
of 18 kHz Determine the modulation the signal. PROBLEM 38
index m= FM madulation index
6 = frequency deviation (Hz} The modulation index of the PM signal
Phitaitionn: is 2. lf a modulating signal that has
Vlolistenen: im = modulating frequency (Hz}
a Vin = peak amplitude af the modulating signal (¥) amplitude
of 50 mV ata frequency of 3
fee = Instantaneous frequency (Hz} kHz phase modulates a 90 KHz carrier
= FM modulation index | i : fe= camer frequency (Hz) signal, what would be the new
6 = frequency deviation (Hz) Sot 6 = frequency deviation [Hz} modulation index of the signal? Assume
1, = modulating frequency (Hz) a=k,V fh = modulation index that the amplitude of the modulating
{s = modulating frequency (Hz1 signal is increased by 50%.
Solution: §=(25 kHz/V)(3)
65758 kHz ‘Solution: Hlotiatiines:
6
ee 15 kHz 65 75 kHz
(%, SkHz i akhz feats m, = PM modulation index
Sette 6, = peak phase devation (rad)
6-12
6-13
The Question Bank Series —- Communications Engineering (Volume 2 - Problems) Question Bank 6 - Frequency Modulation (FM)

ka = sensitivity (rad/V) “The peak amplitude of the For PM system, modulation indexis equal f - & 100 kHz 20 kHz
Ve. = peak amplitudeof the madulating signal (¥) modulating signal is 100 mV. io the phase deviation of the signal. ini eB
BW =2(54+,)
Solution:
athg at BW =2(100 kHz +20 kHz) =240 kHz
k=Spe With a maximum frequency deviation of , The modulation index is 1.5. - The bandwidth using Carson's rule is
Vintakay 75 kHz, what is the maximurn phase 240 kHz.
2 deviation thal can be present in an FM
ky= EUS 40 rad/V fadio broadcast signal, assuming it PROBLEM 42
vaeee cserced Pequeney Pr Sp Determine the modulation index of an ae Tae
m, =kV
Pailin! i ie - FM signal, given that the modulating pin Cid StS tanerite 100 Wat
m, =(40 rad/V)[ 0.05 (1.5)| frequency is 25 kHz and frequency power: If the modulation Index-of the
Fotations: deviation of 75 kHz. signal is 7, how much power is
m,=3
: transmitted at the carrier frequency if
where: Mijn) = Maximum madulation Index Hiatus: Jo= 0.47
Vergriew) = Mingaiay + 9:5 Veil Gru = Mavamum frequency deviation (Hz)
{= modutating frequency (Hz) mm = FM madulation index Photatbous
». The new modulation index is 3. 8 = Irequency deviation (Hz) "a
Solution: {n= modulating frequency (H2} P, = carrier power (W)
tee Pe) The maximum modulation index of the a Su ecae ial
, ‘ = total transm T
A sine wave of frequency 15 kHz See a Mae ai enoy oh Pe .
frequency-modulates a carrier at 107.1 a3 Tere 4 Solution:
MHz. The peak frequency deviation is = Sas, m, = ,
75 kHz. Determine the peak amplitude meats a! From the Bessel Function Table
of the modulating signal if the sensitivity 75 kHz m, = TOKNE 5
of the modulator is 50 radi. fe eS 1500 25 kHz m=7:d,=0.4

Flatutions: “The maximum phase modulation =. The modulation Index of the signe! : Py = tp.
3. P. = (0.4)" (100W) =46W
eplilitaetineticees index is 7500 rad.

aries
es crea EEE PROBLEM 43 i ignal
Aca nomptartiet
is 16 W.
Geen
Vn == peak amplitude of the modulating signal (\') The phase shift of a phase modulated
:
Pen oie 100 kHz and a pees.
= frequency deviation (Hz) signal is 86°, Whatis the modulation modulating index of 5. Calculate the
bandwidth using Carson's rule. PROBLEM 45
Soluti
olution: cea
A.50 ohms dummy load is used to test a
a
5=KiVae, Mlotations: no certain FM station. Suppose the
Hotnsiens: measured power is 12.5 W, what is the
Vie é Ad
ae = phase shift rad)
(rad! BW E= Bandwidth
; (Hz) voltage
ag level of the second
Seatsideband if a

Mi iCON 4607 bal


cian me
Solution:
5 ma ee dean
Inm== Maxaman modulating frequency (Hz)
Tinie Kenemvonielon? Nae
from Bessel Table m = 2: Jz = 0.35.
™ (50 rad/V)(15 kHz) 3 FM modulation index
where: Ag = 86° x -
- i 180 Solution: Hotations
5 = Galin Ab =1,5 rad
% =KaVin no vs = voltage measured at the load (V)
i th P; = total transmitted power (W)

6-14 6-15
Question Bank 6 - Frequency Modulation (FM)
The Question Bank Series— Communications Engineering (Volume @- Problems)

aad ess PROBLEM 50


R= load resistance (2) cae:
An FM receiver sufficiently receives only
The maximum phase deviation given to
Solution: the carrier and the first two sets of 15 kHz the signal by the noise voltage is 0.04
5 kHz radian. What is the signal-to-noise ratio
sidebands. Ifthe total power of the
Vv; = PR signal is. 10 WV (ignore any losses in ihe
signal), deviation is 25 kHz, and the
Using Bessel Function Table
at the detector's input?

V, =(12 5}{(50) =25V modulation index is 10, how much BW


= 2f,N PTotia teenies
Mila Power is transmitied at the carrier
BW = 2(5 kHz}(6)=60 kHz
frequency?
V, = 0.38(25V)=8.75V.., where:m=3 N=6 dn = phase dewation due to noise (rad)
Viv = noise voltage (V)
Flotaticnia: . The bandwidth of the signal is Va = ‘signal voltage (V)
-. The voltage level of the second
sideband is 8.75 Vone- 60 kHz. S/N = signal-to-noise power ratio
Pa, Pi, Pp... = carrer power, first sideband power,
second sideband power , Solution:
aT | = Bessel coefficient of the sidebands PROBLEM 49
Py, = transmitted power (W)
An FM recelver can sufficiently receives An FM transmitter operates with a
only the carer and the first two sets of carrier power of 10 W. What is the total
sidebands. |f the total power of the Solution: signal power if the modulation index of
signal is 20 W (ignore any losses in the the signal is 57
signal), deviation of 25 KHz, anda From
modulation index of 10, what is the P, Pl
JP s2(s? 4244? ..)| Flatadicais:
percentage of the total signal power will
it receive? Po=J7p.
Ve = carrier voltage (V) N

P, =(0.22)' (10)=484 mw P, = camer power (W)


(3) (2—
Hatatinns = | =20log)
“The power transmitted at the carrier
R= load resistance (£2)
6 = maximum frequency deviation (Hz) RA te |
Poy Pi, Peo. = Carter power, first sideband power,
frequency is 484 mW. fn = maximum modulating frequency (Hz)
=) = 20109{ + | = 40dB
second sideband power...
] = Bessel coefficient of the sidebands
m = FM modulation index Ny
; = transmitted power (WN) bate] Solution: -. The signal-to-noise ratio is 40 dB.
Solution; The calculated bandwidth using
Carson's rule is 40 kHz if {he modulating
From frequenecy-of the signal is 5 KHz, Using PROBLEM 51
the same signal, what isthe bandwidth if
P, =P, [s.? + 2(d)? +d? #5? + »)| sidebands with more than 1%-of ihe
An FM receiver detects a 100 mV
signal, A spurious signal is also
signal voltage are considered? detected by the receiver that resulted to
Thus;
Pr, =P. (u," +20, 4 2,*) an S/N of 40 dB at the detectors output.
Determine the minimum S/N ratio at the
P,,s a
=(O.22) a “a ; vi. : delector's output if the modulatin g
‘ ie +2(0.58)
Sg P «2/0:
+2(0.35) FTotativness
fetta = L
POS OUtp!
frequency is 2 kHz and frequency
Li ‘i N= number of significant sidebands 4.16 PENNS GLAD te:
P,, = 98.62% of P. Ves 247.57 V,,
Solution Gate Motatioas:
“96.62
% of the carrier will be BW =2(8 +f.) wie (17.57 -
transmitted. =-1_ =+—_~=308W
‘ a BW ft ik 19 $u = phase deviation due to noise (rad)
Diet $v = phase deviation due to signal (rad)
40 kHz “. The total signal power Is 308 W. Vy = noise voltage (V)
5 -§ KHz =15. kHz Vs = signal voltage (\/)

6-16 a7
The Question Bank Series - Communications Engineering (Volume 2 - Problems) Question Bank 6 - Frequency Modulation (FM)

S/N = signal-to-noise power ratio Solution; att)


14 ee Kop = 1-5k,
tm = modulating frequency {Hz}
f, =f, + 2f An FM receiver uses PLL to detect the kay =1.5(0.0239) 0.03585
signal. What is the sensitivity of the
Solution: f, =99.14 2(10.7)=120.5 MHz VCO in the PLL ifthe output of the BW =kf,
detector is 1.25 V, from an FM signal BW = 0.03585 (10.7MHz) = 383.6 kHz
5)
—|
Bs
=20log—
In=ffo%p? that is modulated to 10 kHz deviation by
(a "5, ae a sine wave? - The bandwidth of the tuner is
Pe Sigg ee f fi 383.6 kHz.
Sais pe een eagas Flotattoves:

9971 «120.5
859918° 400 Su rpee paaeE
Vn = peak amplitude of the modulating signal (¥) dtd
1H ee)
6, 20 IR=4/1+(45) (0.393) =17.74 ky =sensitivity (Hz/V) An FM superheterodyne receiver is
ate
BO KHE _ 509 tp
100 100
IRye = 20log(17.74) =25 dB 6 = frequency deviation (Hz) tuned to a frequency of 86 MHz. Whatis
the local oscillator frequency if low side
Solution: injection is used at the mixer?
= 6, = ihn
-. The image frequency rejection ratio 5
in dB is 25 dB. k=—
f Vv.
Hotertione:

by _ 500 Hz =0.25 rad PROBLEM 53


= OMe gy kHz/V fio = focal oscillator frequency (Hz)
2 kHz 1.25V
the = signal frequency (Hz)
. Vu An FM. detector receives an FM signal {ir = intermediate frequency (Hz)
sing, j= Ve = 0.25 with a deviation of 15 kHz. What is the - The sensitivity of the receiver is
peak-to-peak output Voltage of the @ kH2iV,
thus, Solution:
detector if its sensitivity is 75 wViHz?

(ji) =20605. Hotirticns:


att]: ee)
An FM receiver uses a tuned
From
flo = fer + fe
$)S —201eg—-1 = 1208 transformer operating at 10.7 MHz.
[) 9525 Va = peak amplitude of the modulating signal (V4 What is the bandwidth using the For low side injection
ky =sensitnaty (V/Hz} optimum coupling factor if the pnmary
The signal-to-noise ratio at the 6 = frequency deviation (Hz) circuit has a Q of 50 and the secondary flo =f fe
detector’s Input Is 12 dB. Vos = peak-to-peak output voltage (\) has a Q of 357 fo = 88 MHz- 10.7 MHz
fio = 77.3 MHz
Solution:
Hlatationa:
dt) ha eed
Vo=k 6 The local oscillator frequency is
An FM receiver is tuned at 99.1 MHz. eernra k. =critical coupling coefficient 77.3 MHz.
What is the image frequency rejection Vi. i =| kect = optimum coupling coefficient
ratio if the recelver consists of a single | He |(15
75— || KHz ) Q = quality factor
tuned circuit with Q of 457 V,,=1.125 V B = bandwidth (Hz) PROBLEM 57
A certain FM receiver has a biacking
Vip = 2Mp Solution: dynamic range of 75,da. What is the
Flatatienay
V,, = 2(1.425V) =2.26V maximum signal that this receiver can
|R-= image frequency rejection ratio handle, if the desired signal has a level
, =signal frequency (Hz) >. The peak-to-peak output voltage of of 25 uV.7
fa =image frequency (Hz) the detector is 2.25 V.
f =intermediate frequency (Hz) Flotation:
(= quality factor
DR = Dynamic Range
Vis = Strongest signal (V)

6-18 6-19
The Question Bank Series - Communications Engineering (Volume 2 - Problems) Question Bank 6 = Frequency Modulation (FM)

Ven = weakest signal ('V) FHistatines: Solution: Pladiations;

Solution: IR = image frequency rejection ratio fio = fer + fe f = operating frequency at temperature T
} f, =signal (requency (Hz) f= operating Irequency at reference temperature T.,
DRya 209
= Vax fz =Image frequency (Hz)
fio = 108 MHz +10.7 MHz =118,.7 MHz
k = temperature coefficient per degree
{, Sintermediate frequency (Hz) 1
Ve ie == Va Minti MOlog!)
yf DRys’
ee () = qualily factor fo=
2n/LC Solution:
ees
| 20

[75 Solution ata lemperature of 36°C


Vous
mi = 251M se] log™!|
& — (2xf) Cc
f, =f +kf,(T-T,)
IR = J1+(Qp)° patties
2 aa
_s

Vier = 140.6 mV f, = 101 MHz +101 MHz(—1x10)(35— 20)


(2ni18.7x10°) (125x107)
-. The maximum signal is 140.6 my. IR = ,/1+(60)' (0.41) = 24.62 «The inductance of the tuner is
f, = 100,998485 MHz
where: 14.4 nH.
2 The operating frequency is
tt) |e] f 100,998485 MHz.
fi 1 PROBLEM 61

et
An FM broadcast receiver with high side
injection is tuned to a station at 93.5 va tt7A 95.7 _ O44 A PLL circuit uses a crystal oscillator a] 8d
MHz. What is the image frequency? POST ATA accurate within 0.001%: What is the
f =f, +2f output frequency if tLoperates at 35 Varactor modulators directly change the

Vlatations: f, = 95.7 +2(10.7) = 147.1 Miz MHz? frequency of @ carrier signal proportional
to the amplitude of the modulating
signal, If the sensitivity of the modulator
flo = local oscillator frequency (Hz) for two uncoupled tuned circuit Flotation: k= 3 kH2/V and the maximum deviation
fae = Signal frequency (Hz) is 75 kHz. and Is followed by 2 tripler and
fa = image frequency (Hz) IR; =1R? f= output frequency (Hz) 1 doubler, what audio voltage is needed
f = operating frequency {Hz) at the modulator to obtain full deviation?
Solution:
[Ry = 20log(24.62)" = 55.7 dB
k= accuracy
Af= change in frequency (Hz) Hlotaticns:
From “The image frequency rejection ratio
fim = fae 2h is 55,7 dB. Solution:
fag = Output of multiplier circuits (Hz)
for high-side injection Gee =Olltput of oscillator circuits (Hz)
PROBLEM 60 f, =f + Af
Vi = peak audio voltage (V)
fu = fae + 2f AT=kf = audio voltage RMS .(V)
An FM broadcast receiver tunes from 88
fiq = 99.5 MHz+ 2(10.7 MHz) =114.9 MHz to 108 MHz, The IF is 10.7 MHz. The 0.001
f= 35 MHz +(35x10 5 Hz)[ panied
nee | Solution:
local oscillator uses a variable capacitor
The image frequency is 114.9 MHz. with a minimum value of 125 pF. f=35.00035: MHz i
Suppose the mixer uses high side
Injection, what is the value of the +, The operating frequency Is
aA) inductor so that the local oscillator will 35.00035 MHz.
A superheterodyne receiver uses two tune the receiver to the highest
uncoupled tuned circuits, each with a Q frequency on the band when the
of 60, What is the image frequency capacitor is minimum? PROBLEM 62
rejection ratio of the receiver if the signal An FM radio transmitter transmits at
frequency is 95.7 MHz and the IF is 10,7 Hatations: exactly 101 MHz at 20°C. The
MHz? Assume high side Injection is transmitter Uses crystal oscillator with
used in the mixer. flo = local oscillator frequency (Hz) temperature coefficient of -1 pprmw°C.
Ir = signal frequency {Hz} Find the transmitting frequency when
{ig = image frequency (Hz) the tamperature is 35°C.

6-20
The Question Bank Series — Communications Engineering (Volume 2 - Problems) Question Bank 6 - Frequency Modulation (FM)

v, = Se Fotartiaes: fix = lock range (Hz}


{re = capture range (Hz) fo =af fra ook— 5
kK i, = operating frequency (Hz) {= input frequency lock ts first-achleved (Hz)
Y= 4167 Hz 1 a V L= inductance (H) fo = 14 Mite — SEE 12 tie
3000 HziV C = capacitance (F) Solution:
V=—bVv, _==
1.389 0.982 V = 982 mv V = bias voltage (V) fe hz
a. ire tSAR
v2 V2 C, = capacitance at zero bias voltage (F)

“The mms audio voltage is 982 mV. Solution: The frequency at which lock is lost on the
way Up.
4
a 6}

f=
dik]
4 ees 2nJ/LC te AB
A varactor is used to tune the receiver al 1 f, =13 MHz
= WL & MHz
178 MHz, with 150 WH. What is the
Note; The capture range is approsimately twice the
f.=14 MHz+ =18 MHz
maximum capacitance if no tuning
voltage is applied? Since frequency is increased by 75% difference between the free-running frequency and Note: The Jock range ts appromenately twice the
f, = 14.35 x 10° the frequency at which lock is First achieved. difference between the frequency where lock is
lost and the free-running frequency.
Flojaticnn: Cz ruil suey A aludis Bile .. The free running frequency is 13 The capture range is approximately twice the
4x3 (14.35 x 10°) (75x10) MHz. difference between the free-running frequency and
fs = operating frequency (Hz) the frequency at whieh lock ts first achieved.
L = inductance (H) C=1.64 pF
C = capacitance (F) a) Tate “The frequency at which lock Is
| Suppose that the free running frequency achleved Is 14 MHz and is lost at 18
Solution: MHz.
of the VCO is: 14 MHz, capture range4
The maximum capacitance of the varactor MHz, and lock range 8 MHZ. What
would be the frequencies at which lack
occurs for zero bias voltage. PROBLEM 68
is achieved arid lost provided the
reference input frequency begins at Arvamplifier is operating at class C' with
jal zero? an efficiancy of 80%. What power will it
f 2n/LC deliver to the load ifthe power drawn
c
1
Plotiateinas:
from the source is 125 W?
~ (2x,JL
{a = free-running frequency (Hz) ratios
1 1,
53.3 pF fin = lock range (Hz}
[2n(1.7@x10°) | (150x10°) : ~. The bias voltage at the varactor {in = capture range (Hz) Pay = output power (W)
is 4.44.V, f= input frequency lock is lost (Hz) P,, = input pawer (W)
” The maximum capacitance ts 53.3 pF. de = input trequency lack ts first achieved (Hz) 1 = efficiency
THT eT)
Solution: Solution:
a8) 381 Mt) A PLL has a VCO with a capture range
of6 MHz and a lock range of 14 MHz. fen =2(fra + fic) Peo =T1F
A tuned circuit resonates at a frequency
of 8.2 MHz. 4 varactor diode is used
that has a maximum capacitance of 5
Determine the free running frequency if
the input is gradually raised from zero fn =2(fe th) Pay = 0.8(128)
then the loop locks at 10 MHz. P., , = 100W.
pF with a 75. nH inductor-when tuning fe = tea +e m
Voltage Is zero, What is the tuning
voltage if the input frequency is Votatioes:
¢, The output power is 100 W.
increased by 75%7 Frequency at which lock is first achieved
{ra = free-running frequency (Hz) from zero frequency

6-22 6-23
The Question Bank Series — Communications Engineering (Volume 2 - Problems) Question Bank 6.- Frequency Modulation (FM)
c,
Fotations:
eR) Flotattows:
i+ 2
A BUT transistor has a power dissipation 6 = frequency deviation (Hz)
rating of 15 W. Determine the power an fg = output Irequency of the multiplier (Hz) G, a Cy, (Ji+2v) Vin = peak amplitude of the modulating signal (V)
f. = output frequencyof the oscillator (Hz) k= sensitivity (Hz/V) ‘
amplifier could deliver to the load
operating at class Awith an efficiency of
C, = 45x10? /1+2(10))= 20.6 pF
30%. Suppose that the transistor is the Solution: Solution;
only element that dissipates power in ‘The capacitance
at zero bias is
the circuit, 20.6 pF. B=kV,,
foun
f,.=
(fone )(3)(3(2)(2)(2) = 854 MHz
0% = SSA MP2 6 Getz 8=(2kHz/V) x 10 V=20 kHz
Plodealsenar 7277 ay 0 “The frequency deviation is’ 20 kHz.
P= input power (WW)
Pee = output power (\W)
[a Ete ne dm bf xe one Geen A PLL has a free running frequency of
12 MHz with a capture range of2 MHz
Pos = power dissipated (WW)
Gi He O72 ie (TS ee 0G ee SOT bate and lock range of4 MHz. What is the PROBLEM 74
lowest input frequency of the PLL that
1) = efficiency A PLL has‘a free running frequency of
+The output frequency of the oscillator will cause the unlocked loop to lock?
10 MHz with a capture range of 1 MHz
Solution: is 9.08 MHz.
andlock range of2 MHz: What is the
PLatirt inate: highest input frequency of the PLL that
can be tracked by locked loop?
Pa ae Pout +P at) 14 er
Pe SB Re Eq.(t) fig = free-running frequency (Hz)
The resonant frequency of a varactor- fin = lock range (Hz)
tuned oscillator is 15 MHz when the Hotitinta:
Fea a Eq: (2) fre = capture range (He)
tuning voltage of the varactor reaches {| = input frequency lock is first achleved (Hz)
10:-V. What is the maximum capacitance {in = treestunning frequency (Hz)
Eq (2) in Eq (1) lin = lock range (Hz}
of the varactor given the situation above Solution:
Pri = — Py ifthe inductance of 25 wH is used? foe = capture range (Hz)
q {, = inputfrequency lock is last (Hz}
fsa
fi. = tap = 2
Pose = p(t i Madatraise Solution:
in
Lowest input frequency lock is first
Py {, = operating Irequency (Hz) achieved fi
ai ) L = inductance (H) fi = te 2
(3 C = capacitance, (F) Bete
15 V = bias voltage (\/) = he Highest input frequency lock is lost.
ae

Pas (al = Sipe Cy = capacitance at zero bias voltage (F} 2 MHz


f, =12 MHz-
f
Solution: fe Steet a
.. The power that can be delivered to f, =11MHz
the load is 6.43 W. 1
”. The lowest input frequency is 11 f, =10 MHzy 2 te
— 2eike
PROBLEM 70
MHz. f, =11 MHz
. . tov
je aed z <5
4 local oscillator uses a frequency (2nf)L PROBLEM 73 “The highest input frequency is 17
multiplication circuit consisting of twa 4 MHz.
triplers and three doublers to geta C= A frequency-madulation system has a
frequency of 654 MHz from a crystal I 2a(15x10") | (25x10) maximum allowable modulating voltage
oscillator. What frequency should the 4 r of 10 V. If the sensitivity of the
crystal oscillator operates? Cipy = 4:5 P modulator is 2 KHz/V, what is the
system frequency deviation?

6-24
The Question Bank Series — Communications Engineering (Volume 2 - Problems) Question Bank 6 - Frequency Modulation (FM)

ehh ee ait
NS ed CS= 50 kHz ae Ta aes
An FM system has a rated system An FM modulator has a sensitivity equal
fo 2 kHz/V¥ with a rated system deviation
rh Sa
jal
An FM signal has a maximum
modulating frequency of 3 KHz and a
deviation of 15 kHz and a maximum
allowable modulating voltage of 5 V. of 3 KHz. Calculate the carrier swing if a modulation index of 0.5. The signal-to-
25 noise ratio at the input of the detector is
Calculate the sensitivity of the 0.5 ¥ signal is applied to the modulator. m,=—=
5 30 dB, Calculate the approximate
modulator.
“. The modulation index is 5. signal-to-noise ratio at the detectors
output.
Hoalations: Hotatiens:
Mgt):
83 ee) I)
& = frequency deviation (Hz) 6 = frequency deviation (H2} An FM linear modulator has‘a sensitivity
Hlotirteons:

Vn = peak amplitude of the modulating signal (¥) Vn = peak amplitude of the modufating signal (\V) equal to 2 KH2/¥ and a maximum
k = sensithaty (HV) Oy = phase deviation due to noise (rad)
k= sensitivity (Hz) allowable modulating signal voltage of 5
CS = carrier swing (Hz) V. Ifthe instantaneous camer frequency bs = phase deviation due to signal (rad)
Solution: varies between 98.95 and 99.05 MHz, Vu = noise voltage (V)
Solution: calculate the percent modulation Ve = signal voltage (V)
S/N = signal-to-noise power ratio
B=kV,, Vlatuctaona: f, = modulating lrequency (Hz)
§=(2kHz/V)x 0.5 V=1kHz Solution:
Gad = actual frequency deviation (Hz)
CS= 2x8 Sax = maximum frequency deviation (Hz)
CS= 2x1kHz= 2kHz Vin = peak amplitude: of the modulating signal (V) Vea SLD
M.
k = sensitivity (HV)
-. The sensitivity of the modulator is 3 ” The carrier swing of the signal Is CS = carrier swing (Hz) Ve i
2 KHz,
kHz/V, Yeo
—= |
2) et90 31.62

Solution:
a1 3 tc = ein te sgt
gt
18 Baa = kVp s Ss

An FM modulator has sensitivity equal An FM linear modulator operates at 92 Bye =(2 KHZIVx SV 1
to 2 KHz/V and a rated system deviation MHz (carrier frequency), It is modulated Prax = 39 62
ofS KHz. What is the maximum by a 5 kHz sinusoidal signal. Calculate By = 10 kHz
allowable signal voltage? the modulation index if the Sy = ob, = fy
instantaneous carrier frequency varies
_ GS.
Fa a by = Smaak okHzZ = 95 HZ
beiween.91.975 and 92.025 MHz.
Flotutients: Bean = | 99.05 msma 4Me 6, =m,x 3KHz = 1500 Hz
(s) = 2010928i
(3)
6°= frequency deviation (Hz) Petationa:
Bmax = 0.05 MHz ini 7s
Vey = peak amplitude af the modulating signal (V)
k= sensitivity (Hz/V) 6 = frequency deviation (Hz)
ny = FM modulation index
4M = 82 41 00% S|gs) ~ 20109 °8 208
ea.
1500 — 24 up
95
Ora
Solution: fn = modulating frequency (Hz)
CS = carrier seang (Hz) ag = 12 KNZ 409 “The output signal-to-noise ratio of the
50 kHz detector is 24 dB.
Solution: iM
= 20 %

CS = (92.025 - 91.975) MHz -. The percent modulation is 20 %. bit)


1 43
Vv, gate sy CS= 0.05 MHz Initially, FM signal bandwidth is 100 kHz
2kHz/V¥
. GS
§.=——=
S50 kHz =275 kHz
when the modulation index is 5, What is
2 2 the new modulation index if the signal
“The maximum allowable signal level is increased by 8 dB?
voltage is 1.5 V.

6-26
The Question Bank Series — Communications Engineering (Volume 2 - Problems) Question Bank 6 - Frequency Modulation (FM)

Votatraes:
PROBLEM 84
Thus

(
An FM receiver tuned at-91.9 MHz uses
VW = signal voltage (Vv) two uncoupled tuned circuits before the
ty = FM médulation index 2 mixer, each with a @ of 65. Ifthe
‘k= sensitivity (HzIV) fn (4 DR) receiver's IF is 10.7 MHz and tha local
i, = carrier frequency (Hz) _ oscillator uses low side injection, what is
i, = modulating frequency (Hz) (eae) the image rejection ratio of the receiver
ia ah (10)
eT 0 7 78.2 kHz in dB?
Solution:
Hoatattans:
ae “ The maximum modutating frequency
Vas Rees
is 8.2. KHz. = image frequency rejection ratio
i = log"! Moa f, =signal frequency (Hz)
an 20 ea DTA fy =Image frequency (Hz)
{ =intermediate frequency (Hz)
el
= 25 An FM system has a rated bandwidth of Q = quality factor
180 kHz and an output signal-to-noise
KVR ratio of 30 dB; when the maximum Solution:
m= modulating frequency is 18 KHz and the
v7 frequency deViation-is 75 kHz. What will
since m, is directly proportional to V,_ sa, be the new quiput signal-to-noise ratio if IR= 1+ O%p?
the deviation ratio. is increased to 8?
TO fran) =2,
= 5m toa | IR = /14(85) (0.54
Mecnom)
= 2-515) =12.5 Hidatione:
IR =35.1
where:
OR =deviation ratio f=
The new modulation index is 12.5. fe ueof,
S/N = signal-to-noise ratio
mk
si fy
6 = frequency deviation (Hz)
PROBLEM 82 (70.5: 91:9
finiean) = Maximum modulating frequency (Hz)
“i919 70.5]
An FM broadcast transmitter has a
Solution;
bandwidth of 180 KHz and deviation f, =f, -2f

ratio of 10. Suppose the carrier f, =91.9-2(10.7)


frequency is 98.2 MHz, what is the = 70:5 MHz
maximum allowable modulating
frequency?
For two-uncoupled tuned circuit

Hotties:
IR, = (IR)
2in
ee

BW = Bandwidth (Hz) IRijg = 20log(IR)


a
=

‘\
6 = frequency deviation (Hz) | =30dB+ 20I09| = |
wlio

finan) = Maximum modulating frequency (Hz)


IRygp = 20l0g(35.1)°
Det
=

DR = deviation ratio [Rigg = 61.8 dB


=30:dB+4.10dB
2luo
a

Solution: “, The image frequency rejection ratio


i

| =34.10B Is 61.8 dB.


Zzlo

BW =2(8+f,..) * tat

6
OR Ss ~. The new signal-to-noise ratio is 34.1
dB,

6-28 6-293
The Question Bank Series - Communications Engineering (Volume 2 - Problems)

Votes

Question Bank 7
TELEVISION AND FACSIMILE.
Question Bank 7 — Television & Facsimile

PROBLEM 1 PROBLEM 3
In TY, the vertical resolution ts the Past ECE Board Problem
number of horizontal lines that can be In facsimile transmission, if the drum
distinguished. If 485 lines are shown on diameter is 60 mm and the scanning
the screen, determine the vertical pitch is 0.1 mm, calculate the index of
resolution. cooperation using IEEE
recommendation.
Hotatians;
Hotations:
Ry = vertical resolution
NL = actual lines shown |GC = index of cooperation
D = drum diameter (mm)
Solution: P= scanning pitch (mm)
a

Ry = O.7N, Solution:
Ry = 0.70485)
Ry =340 lines For IEEE,
ioetO 2(60 mm)
“The vertical resolution is 340 lines. Pp O.4mm
lOc =1885
a1 “The Index of cooperation using IEEE
Past ECE Board Problem recommendation is 1885.
Calculate the video-frequency response
corresponding to the horizontal
resolution of 200 lines of a TV receiver. PROBLEM 4
Past ECE Board Problem
A signal in a cable television system has
Flotation: an amplitude of 4 mV ina 75-ohm
resistive line. Calculate its level in dBm.
P= video-frequency response
T= horizontal line synchronization time
= 63.5 ps (NTSC standard) Hoatations:
=.

J, = line suppression period


—_

= 10 us. (NTSC standard) dBm = dB referred to 1 rnillwatt

Solution: Solution:

1=Me/ ] dBm =10log} ——


(Fs
2 Nae

perio oe dBm = tolog( 2213 2)


1x10
2 \6a5xto* a dBm = -36.7 dBm
f=1.87 MHz
where:
2
-. The video-frequency response Is
1.87 MHz. P= (0) oats pW

”. The signal level is -36.7 dBm.


7-3
The Question Bank Series — Communications Engineering (Volume 2 - Problems)

10):
48 3 Oe) VMotutions:

Past ECE Board Problem


Ta = horizontal line synchronization time
Calculate the video-frequency response
= 63.5 jis (NTSC standard)
corresponding to the horizontal
resolution of 320 lines of a TV receiver.
Solution:

Vota tions: 10% (Ty) = 10 %6( 63.5 ps) = 6.4 ps

Ty = horizontal line synchrontcation time “The horizontal flyback time is 6.4 pus,
= 63.5 us (NTSC standard)
T, = line suppression perkod
= 10 jas (NTSC standard)
aU ed
1 = highest video frequency For analog TV transmission, typically
a= aspect ratio the bandwidth of video signal is up to 4
N = horizontal resakition MHz. If the vertical resolution is incraase
N, = suppressed horizontal fines to 500 lines, determine the maximum
bandwidth of the signal that the TV can
Solution: handle.

In practice, the highest video frequency


Solution:
is standardized at

{ i 0:369{ Ru
AIRE Ng ay
= (aes MHz Using 32 lines for vertical trace

320-40 herizontal line = (S00/0.7) + 32 = 747 lines


f= 0.35(1.93)( 0-0
A total of 30 frames scan in one second,
f= 2.4 MHz therefore
where:
a= aspect ratio = 1.33. # of horizontal line = frame time / line time
Ty = 63:5 ps { ine eaetime = ot
4/30
Tay AA Be BS
“Te: =10 ps
If 10 ws is used for horizontal blanking
Nz =40 lines pulse, therefore
Note:
Since the lowast frequency in the Trace time = 44,62 — 10 = 34.62 ps
video signal is nearly 0, therefore the Assuming the horizontal resolution stay at
highest video frequency also gives the 428 lines, the new bandwidth is Question Bank 8
video bandwidth required.
Trace time x BWx 2=428
MICROWAVE COMMUNICATIONS
°. The video-frequency response is
BW = 428/ (trace time x 2) = 6.2 MHz
2.4 Miz.
.. The maximum bandwidth of the
bd C0):
7 1) signal that the TV can handle is
6.2 MHz.
Past ECE Board Problem
Fora TV system, the horizontal flyback
is 10 percent, this time equals ;

7-4
The Question Bank Series - Communications Engineering (Volume 2 - Problems)

PROBLEM 5 Totutinns:

Past ECE Board Problem


Ty = horizontal line synchronization tee
Calculate the video-frequency response
corresponding to the horizontal = 63,5 jus (NTSC standard)
resolution of 320 lines-of a TV receiver.
Solution:

Potations: 10% (Tw) = 10 %( 63.5 jis) = 6.4 pas

Ty = horizontal line synchronization time “The horizontal flyback time is 6.4 js.
= 63.5 ps: (NTSC standard)
T, = line suppression period
= 10 jis (NTSC standard) at] 1h ay
f = highest video frequency For analog TY transmission, typically
a = aspect ratio the bandwidth of video signal is up to 4
W = horizental resolution MHz. if the vertical resolution is increase
HW, = Suppressed horizontal lines to 500 lines, determine the maximum
bandwidth of the signal that the TV.can
Solution: handle.

In practice, the highest video frequency


Solution:
is standardized at
NeN Using 32 lines for vertical trace
t=0:36{ | MHz
Ty (48)= Ts (us)
320 = honzontal line = (S00/0.7) + 32'= 747 lines
f=0.95(1 39)( 32-2
A total of 30 frames scan in one second,
f=2.4 MHz therefore
where:
a=aspect ratio = 1.33 # of horizontal line = frame time/ line time
Tj =63:5 ps : r
line time = oe
1/30
var = Ad G2 is
“Ts =10 us
If 10 us is used for horizontal blanking
N, = 40 lines pulse, therefore
Note:
Since the lowest frequency in the Trace time = 44.62— 10 = 34,62 1s
video signal is nearly 0, therefore the Assuming the horizontal resolution stay at
highest video frequency also gives the 428 jines, the new bandwidth is Question Bank 8
video bandwidth required.
Trace time x BW x 2 = 426
MICROWAVE COMMUNICATIONS
+. The video-frequency response is.
2.4 MHz. BW = 428) (trace time x 2) = 6.2 MHz

-. The maximum bandwidth of the


PROBLEM 6 signal that the TV can handle is
6.2 MHz.
Past ECE Board Problem
Fora TV system, the horizontal flyback
is 10 percent, this time equals .

7-4
Question Bank 8 — Microwave Communications

ad dik)
3] 03 |}
Whatis the path attenuation between In microwave communications, what |s
transmitter and receiver ata frequency the minimum recelved carrier power in
_| of 1.2.GHz and @ distance of 11,000 mi? dBm if the minimum carrier-to-noise
requirement for a receiver witha 10
MHz noise bandwidth is 24 dB?
4 Pioteitaies

L,= free space path loss or attenuation (dB) Motateyas:


d= path distance (km)
pi= frequency (GHz) Gon = minimum received carrier power (dBm)
: = carrier-to-noise ratio (dB)
Solution:
N= thermal noise (dm)
L, =96.6 + 20logd,,)
+ 20 log fay.
L, =98.6 +20log(11,000)
+ 201091 2 Solution:

iE =179 dB
on -< (dB) +N (dBm)
“. The path attenuation is 179 dB. Coy = 24 (dB)+ (-104 dBm)
Cia= -80.dBm
PROBLEM 2 where:
Calculate the beamwidth of a fully N=-174 dBm + 10 log (10%10°)
illuminated microwave passive repeater N=-174 dBm + 70 dB =-104 dBm
whose effective linear dimension is 25
feet with an operating frequencyof 6 .. The minimum received carrier power
GHz. is -80-dBm.

Flotutions
PROBLEM 4
0 = beamwidth (deqrees) Past ECE Board Problem
A= signal wavelength (Ht) What is the effective earth's radius when
L=linear dimension (It) N,= 3007

Solution:
Potations:

Ԥ= et (degrees) Fu = effective earth's radius (km)


Ry = true earth's radius = 6370 kin
p= (58.7 )(0.164) N, = surface retractivity = 300
i 25
06=0.385 degrees Solution:
where:
5-0 9x10"mis = 005m =0.164 ht R, =R,|1-0.046658%"="™ |!
=2h A
f 6x 10" Hz R = 6370)1-0,04565e° 97710]
“The beamwidth of a fully luminated R, = 8477 km= 8500 km
microwave passive repeater is 0.385°.

B-3
The Question Bank Series —- Communications Engineering (Volume 2 - Problems) Question Bank 8 -— Microwave Communications

~The effective earth's radius is Plotations: ea) A) PROBLEM 10


approximately 8500 km. Ina certain microwave links, the Past ECE Board Problem
k= correction factor for relatively flat earth following parameters are given: Path In a microwave communications system,
N,= surface refractivity= 250 distance = 40 km, average terrain and if the minimum camer to noise (C/N)
PROBLEM 5 requirement for a receiver with a 10
climate operating frequency = 8 GHz,
In space diversity, what is the antenna Solution reliability desired = 99.99 %. Determine MHz bandwidth is 22 dB, the minimum
separation required for optimum the fade margin to be assigned to this receive camier power is
operation if the operating frequency ls 8 k =(1-0.04665e"%"™.) link.
GHz and the path length is 40 miles?
ke [1 df 0.048¢5e°97702%0) Hotations:
Hotutions:

Fiotatines: k= 1.23 Ae= cammierto-noise ratio (dB)


d= path distance (km) N
5 = vertical separation distance between two . The correction factor is 1.23. a = terrain characteristics Cry= minimum receiver carrier power (dBm)
antennas (im) a= 1 for average terrain N = thermal noise power (dBm)
A= signal wavelength (m) b = chimate characteristics
R. = effective earth's radius (km)
PROBLEM 7 b = 0.25 for average climate Solution:
L= path lengthy or distance (hin) A microwave radio-link cperated at f= frequency (GHz)
R, = true earth's radius = 6371 km frequency of 4 GHz has a separation A = system reliability (08) = Cyan (8) -N(GBM)
k= comection factor lor relatrvely flat earth distance of 50 km between antennas.
k= 4/3 (typical value} By how much must the beam clear the Solution: Cina (118M) = (88) pnee
obstacle?
Cheq = 22 dB (-103.98 dBm)
Solution: FM = 30iogd,,,, + 1DlogBabf — 1Dlog(1- R)-70
G,,, =-82 dim
FM = 30log 40 + 10lng(6)(1)(0.25)(8)
Ss. 3AR,
Hotations:
—1Dleg(1—0.9999)- 70
where: ;
L
Leth = dearance from an obstacle (m) FM = 28.65 dB
_ U37-Sx10"° (8495.52x10") A = radius or height of first Fresnel Zone (m)
N=KTBW = (1.38x107"}(290)(10x10°)
6436x108 n= number of Fresnel Zones (FZ) .. The fade margin to be assigned to N=4.002ni0*W
n= 1 for first
FZ = 2 for 2" FZ,
ete. the link is 28.85 dB.
$=14.85m
Ben = 1Oig{ $029" *)- 103.98
where: Solution:
ed AS]

f
= eS
Bxi07Hz
oe stem h=0.6(R,,) For a relatively flat earth, what is the
PROBLEM 11
h = 0.6(30.58)
correction factor used when the surface
L=40 miles= 64.36 km refractivity is 300? Past ECE Board Problem
h= 18.35 m
R, =kR, In microwave communications systems,
where: Hetatiows: fora camer frequency of 6 GHz anda
4
R, = [Fear
Ry =17.3Peni qbade
distance of 40 km, determine the free-
k = correction for relatively dat earth space path loss in dB.
R, = 8495.52 km ‘GH: am Ne = surface refractivly
=i7g 2)
©. The antenna separation is 14.65 m.
iy (60) Solution: Flotutions:

Ra) = 30.58 m TNs i |, = free space path loss (di)


Ks [1 — 0.046650 0057
at] AA) d=path distance (km)
f = frequency (GHz)
What is the correction factor for ”, The clearance from the obstacle is k-=[1- 0.046652 9877000" 7"
relatively flat earth if the surface 18.35 m.
refractivity is:2507 k=1.33 Solution:

“. The correction factor is 1.33. = 92.44 20logd,,,, + 200g fo,


The Question Hank Series — Communications Engineering (Volume 2 - Problems)

Lp, =92.4+20log40 + 201096


L»=140 cB
“The free space path loss is 140 dB.

PROBLEM 12
In space diversity, whatis the anienna
“separation required for optimum
performance assuming a relatively flat
earth with a path length of 30 km at 10
GHz?

fi (eens:

Ae = effective earth's radius (km)


k = correction factor for relatively fat earth = 4/3
A, = true earth's radius = 3960 miles = 6371 km
L = path length or distance = 30 km = 30x10! m

Solution:

5
_ BR,

{ ax10° |(esosrat?)
_- toxto?
30x10"
§=25.48 m
where:
fiz $6731)
R, =8494.67 km
Question Bank 9
©. The antenna separation is 26.48 m.
SATELLITE COMMUNICATIONS
The Question Bank Series - Communications Engineering (Volume 2 - Problems)

Lp =92.4420log40 +20log6
Lp =140 dB
+. The free space path loss is 140 dB.

PROBLEM 12
in space diversity, what is the antenna
Separation required for optimum
performance assuming a relatively flat
earth with a path langth of 30 km at 10
GHz?

Vatitinia:

Ry = effective earth’s radius (kun)


k = correction factor for relatively flat earth = 4/3
A; = true earth’s radius = 3960 miles = 6371 jon
L = path length or distance = 30 km = 30% 107m

Solution:

| 3x10" |(esoene 0%)


40x10"
30x10"
$=25.48 m

where:
Ree $(6731)
R,, = 8494.67 km
*. The antenna separation is 25.48 m.
Question Bank 9 - Satellite Communications

PROBLEM 1 Hutatroes:

What is the uplink receiver frequency if


the downlink transmitter is on channel 4 EIRP = eHective isotropic radiated power (dBW)
(3.84 GHz)? Py = radiated power to the antenna from the
transmitter (qBW)
Gy = transmit antenna gain (dB)
Vlotutisna:
Solution:
{= uplink frequency (GHz)
fi = downlink frequency (GHz)
{, = transponder oscillator frequency (GH) EIRF) caw) = Payanwy + Gryap)
= 2.225 GHz (actual frequency) With no losses,
BW
Solution: Feige) = Ptapye) = toi0g{ = | =49
i Fa

i
fa 7 fy f fy EIRP,yy) =9 +40 = 49 dBW
f, =3.64 GHz + 2.225 GHz
f, = 6.065 GHz

i_%-4-),. 4S
-. The EIRP is 49 dBW.

2. The uplink receiver frequency is


6.065 GHz. PROBLEM 4
in satellite communications, what is the

ea
height of a satellite from the earth's
sa 121
surface if the sidereal period is 20
Calculate the required via net loss fora hours?
satellite link with a one-way delay of 250

a
ms.
How fren:

Vlotaticnes: q = gravitational acceleration = 9.81 x107 km/s?


T= sidereal period = 20 hours = 72,0005
VAL = via net boss (7B) R= earth's radius = 6371 km = 3960 miles
t= time for one-way delay (ms)
Solution:
Solution:

hata
VNL =(0.2t+0.4) dB lareR?
WNL = 0.2(250)+0.4
VNL= 50.4 dB ate enum ae neers
4n*
©. The required via net loss is 60.4 dB. h=31,022.5 km

PROBLEM 3 -. The height of the satellite is


31,022.5 km.
A satellite transmitter operates at 4 GHz
with an antenna gain of 40 dBi, The
receiver 40,000 km away has an ted eS
antenna gain of 50 dBi. If the transmitter:
has a power of 8 W, find {ignering Find the orbital period of a satellite ina
feediine lasses and mismatch) the EIRP
ercular orbit 600 km above the surface
of the earth.
in dBW.
The Question Bank Series —Communications Engineering (Volume 2 - Problems) Question Bank 9 — Satellite Communications

Plotations: Solution: PROBLEM 8 81:18 Be)


Past ECE Board Problem Past ECE Board Problem
v = satellite velocity in orbit {mys} CG—(dB)=EIRP ‘any + M-L, +228.6 An-uplink Tx of 2 satellite system A. satellite transmitter with 6 watts power
R= earth's average radius (km) = 6371 km operates at 6 GHz with a Tx power of 12 operates at 4 GHz with an antenna gain
h = satellite height (km) = 600 km kW and an antenna gain of 50 dBi. The of 40 dBi. The receiver, 40,000 km away
=57+20-205.6 + 228.6 has an antenna gain of 50 dBi. If the

2/0
d= distance = circumference of the orbit (rm) receiver on the geostationary satellite
T= satellite period (hr) has an antenna gain of 40 dBi, The overall loss Is 9 dB, find the ERP.
=100 dB elevation angle to the satellite from the

z2/o
Solution: ground is 45 deg., calculate the received
signal strength at the satellite. (Height of Hoteteains:
where: saiellite above the equator is 36,000
We 4x10"!
km., earth radius is 6400 km.) ERP= effective radiated power (dBW)
(R+h) EIRP yy) = Pr caswy + Sriaay
P; = satellite transmitter power (dBi)
ye eset EIRP soy ‘log (50
a W }40
Plotations; Solution:
(63774600) EIRP, ay, = 57 dBW
v= 7574.997 mis Pe= received signal strength (dBm) ERP ine) = Prjcaw) 7 Od LOSS
joy + Sally
M=20 dB Py = transmit power (dBm)
_d
oo2e(R+h) L, =92,4+ 20log38000 + 2010912 L, = tree space path loss (dB) ERP y= 2 dBW - 9 dB+ 40 dB
v
L,= 205.6 dB d = satellite range (kr) ERP, ja) = 40 dBW
Te
2n(63714.600)x10%m , where:
” 7574,997 mis .. The carrier-to-noise ratio at the
Solution:
T=5762)19 5: receiver is 100 dB .
P= rol0g( ‘aw
2) =9:03 dBW
Priam) = Prjccny * Grea) Soran) + Geer)
T =1.6 hours Fixed loss (transmitter) = 9 dB
Poin = 70.79 dBm +50 -199.56 + 40
~The orbital period ofa satellite is PROBLEM 7 i Aldi) =-39.8. dBm
.. The ERP is 40 dBW.
1.6 hours. Past ECE Board Problem
Calculate the free space loss between a where:
satellite transmitter and a satellite 42x10" ad
add) receiver at a distance of 25,000 km Pe aem) = 109 12402
Past ECE Board Problem
Past ECE Board Problem using 6 GHz.
P eon)= 70.79 dBm In satellite communications system, for
A satellite earth receiving installation a total transmit power of 5000 watts,
with a figure of merit of 20 dB is used as L, =92.4+ 20logf,,, + 20logd,,,
determine the energy per bit fara
a ground terminal to receive-a signal Hota tiene: L, =92.4 + 20log 6 + 20log 39115.34 transmission rate of 50 Mbps expressed
from a satellite ata distance of 38,000 L, = 199.58 dB in dBW.
km. The satellite has a Tx power of 50 La = free space path loss (dB)
W and an antenna gain of 40 dB. f= frequency (GHz) d= /(R+h) -R'cos’p —Rsingt
Assuming negligible losses between d= path distance (km)
satellite Tx and its antenna, calculate d,,,=((6400 + 36000)" — (6400) cos? 45 ~ FTotatiemts:

the C/N at the Rx fora BW of 1 MHz Solution: 6400sin45 = energy per bit (J/bit or J or dBW when referred
using 12 GHz frequency, (Boltzmann's
constant is equal to -225.6 dBW)
d= 38115.34 km to 1'W)
L, = 92.4+20logf,.. + 20logd,_
P; = total transmit power (W)
L,= 92.4 + 20log G + 20log 25,000 .. The received signal strength i, = bil rate (bps)
Hlotations: L,= 195,92 dB is -38.8 dBm.
Solution:
.. The free space loss is 195.92 dB,
z = camier-to-nolse ratio (dB)
= Fr _ 5000"
EIRP = effective isotopic radiated power (dBW) “a 50x10"
M = receiver figureof ment (dB) E,
= 100 j.l'bit
Ly = freespace palh loss (dB)
3-4
The Question Bank Series - Communications Engineering (Volume 2 - Problems)

IndBw,
&, =10log)
100x10'*
——————
lo 1W
E, =-40 dBW
”. The energy per bit is -40 dBW.

PROBLEM 11
At what height is the satellite from the
earth's surface if its sidereal period is 18
hours?

Holations:

h = satelite height (km)


T= satedite period = 18 hours = 648005
R= earth's average radius = 6371 km
Q = acceleration due to gravity = 9.81 x 10% km/s*

Solution:

h=
gl R? R
4x7 3

aS (oat x 10*)(64800)" (6374) tee >


ax
h= 28,486 km
-. The height of the satellite is
28,486 km.

Question Bank 10
MULTIPLEXING
The Question Bank Series - Communications Engineering (Volume 2- Problems)

in dBW,
100x10°%
E= so
iw
E, =-40 dBW
. The energy per bit is 40 dBW.

PROBLEM 11
At what height is the satellite from the
earth's surface if its sidereal period is 1B
hours?

Vlotutions:

h = satelite height (km)


7 = satellite period = 18 hours = 648005
R= earth's average radius= 6371 km
g = acceleration
due to gravity = 9.81 x 104 kmjs*

Solution;

payer -r

U (9.84 x 10*) 64800)" (6371) *


h { me - 6374
he 28,486 km

-. The height
of the satellite is
28,486 km.

3-5
Question Bank LO —- Multiplexing

Boe .. The output frequency to which the


baseband frequency will be
‘The basic European TDM telephone
translated is 79.6 KHz.
signal combines 30 Voice channels with
2 signaling channels that have the same
data rate as the voice channels. PROBLEM 3
The sampling rate is 8 KHz and there
are8 bits per sample for each voice For an FOM system with 75% speech
channel. Calculate the total bit rate for loading and 25% data loading,
this signal. determine the rms power level required
as recommended by most
manufacturers for 600 voice band
Plotiatienia: channels.

f, = bit rate (bps) ;


= total number of channels = (30-2) Viotiateces:
-b = number
of bis per sample = 8
{, = sampling rate (Hz) = 8 kHz Pons= mS power level (dB)
N= number of voice band channel
Solution:
Solution:
f, = Cf, =[(32)(8) (8000)
f, = 2.048 Mbps Pog =—11+ 10logN
Pg =—11+ 1009600
«. The total bit rate Is 2.048 Mbps.
Pine = 16-78 dB
ake)
1 ey -, The rms power level is 16.78 dB.
Find the frequency to which a 400 Hz
baseband frequency will be translated, tt) Se
for channe! 8 of a group.
Using CCIR recommendation, what is
the noise load ratio for an FOM
Mlatationa: configuration with 600 voice band
channels?
f= carrier frequency (Hz}
f, = ideal (original spectrum) (Hz)
foe = output Irequency (Hz) Fictatians:
f= channel number
NLR = noise load ratio (dB)
Solution: N= number of voice band ‘channels

Solution:
For channel 8,
i, =112-4n =112-4(8) NLR (dB) =-154 10logN:
f, =80 kHz NLR (dB) =-15410log600
fay =f - 400 Hz NLR (dB) = 12.78 dB
fy = (80 kHz - 400 Hz)
~. The total noise load ratio is 12.78 dB.
four = 79.6 KHz

10-3
The Question Bank Series - Communications Engineering (Volume 2 - Problems) Question Bank 10 — Multiplexing:

gts 5 Platitteane BW e193)


Determine the carrier frequency of ame . frsho” Past ECE Board Problem
channel five in frequency division f, = camer frequency (Hz) 1ZSkHz Assume a frequency range of 401 000
multiplexing { = (dea! (orginal spectrum) (He)
fn = outpul frequency (Hz)
Sn P00 Mae mori Miz to 401.050 MHz, if you apply 25
fy == 401.00625
401, MHz kHz channeling pian, determine the
n='channel number center frequency of the first channel
fram the lower limit.
a

Hae Solution: .. The center frequency of its first


channel is 401.00625 MHz.
{c= carrier frequency (Hz)
ep

n= channel number For channel 7, Hotations:


i

f, =(112—4n) kHz
Solution: Ui fe; = center frequency
of its first channel (Hz)
£=112-4(7) BW = bandwidth (Hz)
Past ECE Board Problem
£=(112-4n} kHz f= 84 KHz Let us consider a frequency range of f, = tower: frequency limit' (Hz)
405.0125 MHz to 405.0875 MHz and a
f, =[112-4(8)] fa = # - | = 84 kKH2z—400 Hz
25:kHz channeling plan, determine the Solution.
L=92kH2 faz 83.6 kHz center frequency of the second channel fs
from the lower limit. 25 kHE
ee |. ee ee

oo The carrier frequency is 92 kHz. « The output frequency to which the


——— ee

baseband frequency will be Hotitinns: :


transiated is 83.6 kHz.
Ta) {es = center frequency of its second channel (Hz) 401.000 MHz 401.0125 MHz
BW = bandwidth (Hz) \ ) 401.050 MHz
Determine the carrier frequency of ae At ey {i = lower frequency limit (Hz) Channel
roup five in frequency division
Paibleaie i : Past ECE Board Problem
lf frequency range of 407,000 MHz to: Solution: BW
401,050 MHz has to be channelized at
ee

fy f+
Hoturiane 12.5 kHz between channels, what is the

tr center of its first channel from the lower 25 okHz 25kHz


2
f, = 401.000
:
MHz + a kHz
: limit?
{; = carrier frequency (He)
n= channel number
i f., = 401.0125 MHz
Solution: Flotiations: 405.0125 MHz = 405.0875 MHe +. The center frequency of the first
channel is 401.0125 MHz.
: {p) = center frequency of its first channel (Hz)
rae

f, = (872 + 48n). kHz BW = bandwidth (Hz) fe =t, + BW + =


f, =[3724+48(5)| {= lower frequency limit (Hz) 25 kHz PROBLEM 11

fc = Ble Rie Solution:


ge SOS. F2o Mie Se RZ to Past ECE Board Problem
A Assume a frequency range of 405.0125
.. The carrier frequency is 612 kHz. ' Be Oe MHz to 405,0875 MHz and 25 KHz
ct channeling plan, determine the center
. The center frequency of the second frequency of the second channel from
12.5 kHz
channel is 405.05 MHz. the lower limit. ;
PROBLEM 7 :
Past ECE Board Problem
Find the frequency
to. which a 400 Hz 407.000 MHz 401.00625 MHz Plotatama
baseband frequency will be translated dgi.G80 Me
for channel 7 of a group of FOM system, Channel 4 : feo = center frequency of its second channel (Hz)
BW = bandwidth (Hz)
f, = lower frequency limit (Hz)

70-4 10-5
The Question Bank Series —Communications Engineering (Volume 2 - Problems)

Solution: PROBLEM 13
Following CCITT’s recommendation
fs fz (now ITU-T) for loading multichannel
FDM systems, whatis the average
25 kHz 25kHz power level required when the number
t : of voice band channels is 2407

405.0125 MHz 405.0875 MHz Flotations:

BW Poe = average power level (dB)


fo =f + BW + on N = number of voice band channels.

fog = 05,0125 MHz + 25 ka + “> Solution:

fie EOE Me For N = 240 voice band channels


.. The center frequency of the second P= 2484 40iogN
channel |s. 405.05 MHz. ave Pog
Piya = 15 +10log240
P,.
= 8.8 dB
rate) 10d i]
Past ECE Board Problem .'. The average power level is 8.8 dB.
Assuming a UHF frequency range of
405.0125 MHz to 405.0875 MHz at 25.
kHz channeling plan, how many
channels can you produce?

Flotations:

N = total number of channels


fy = upper requency limit (Hz)
{Slower frequency limit (Hz)

Solution:

fit
het
BW Question Bank 11
N= (405.0875 - 405.0125)MHz
iL, 25 kHz
N=3

+. The total number of channels. .


is 3.

10-6
The Question Bank Series — Communications Engineering (Volume 2- Problems)

Solution: PROBLEM 13
Following CCITT’s recommendation
fy fa (now ITU-T) for loading multichannel
FOM systems, what is the average
25 kHz. 25kHz power level required when the number
: E of voice band channels is 2407

405.0125 MHz 405.0875 MHz Hovations:

BW Pun = average power level (dB)


eae N = number of voice band channels
leg = 405.0125 MHz + 25 KHz+ oe Solution:
fg = 405.05 MHz For N = 240 voice band channels
.. The center frequency of the second Pave = —15+ 10l0gN
channel is 405.05 MHz. P, = 15+ 1010g240

Pig = 8-608
PROBLEM 12
Past ECE Board Problem -. The average power level is 8.8 dB.
Assuming a UHF frequency range of
405.0125 Mrz to 405.0875 MHz at 25
kHz channeling plan, how many

7
channels can you produce?

Foto pce!

N = total numberof channels


fy = upper frequency limit (He)
f, Slower frequency firnat (Hz)

Solution:

N= fu-f
Bw
N= (405.0875 - 405.0125)MHz
Question Bank 11
25 kHz
N=3

-. The-total number of channels


is 3.

10-6
Question Bank 11 — Radar

PROBLEM 1 PROBLEM 3
Past ECE Board Problem Past ECE Board Problem
Given the pulse width of 8 microseconds What is the peak power of a radar pulse
and a duty cycle of 8%, determine the if the pulse width is 1.0 ys, PRR is $00
pulse repetition time of radar system, and the average plate power inputis 45
watts?

Hlotatroms:
Flotationta:
PRT = pulse repetition time ()15)
PW = pulse width (p15) PRR = pulse repetition rate
PW =oulse width (yrs)
Solution: PRT = pulse repetition time (ms)

Pulse Width_ Sus Solution:


PRT = eee
“Duty Cycle
Cycle =o 08
PRT= 100 ps Duty Cycle= Average Power
Peak Power
k Po Average
Eee Power
“The pulse repetition time is 100 ys.
Peek Power = Duty Cycle
Peak Power= 72" _ 50 nw
baste)
=) 49 ie 9x10"
where:
Past ECE Board Problem
Find the distance in yards to an object if PW
the display of a radar signal measures Ruy CGE PRT
4.4 microseconds. 10x 10° a
he Sang = OI
Buty 1A4x 10° s

Hlotiricice: PRI=—
PRR

d= distance (yards) PRT = aa =1.44x10%s


‘y= ¢ = signal velocity=3 410% m/s
t= time (5}
-. The peak power of the radar is 50 kW.
Solution:
ag
d=vt=(3x10°)(2.2x10*) What is the Doppler shift in frequency
d=660m (2.28 0m)( 122ca caused by a vehicle moving towards a
radar at 96 kph if the radar is operated
g=721.6 yards at 10 GHz?
where-
v=3x10°mis Vlotmticers:

l= “ = = 2.2 4s (time to reach the abject) fp = Doppler shitt'in frequency (Hz)


¥, = velocity of the car (m/s)
{| = operating Frequency (Hz)
.. The distance is 721.6 yards.
= Wave velocity (m/s)

11-3
The Question Bank Series - Communications Engineering (Volume 2 - Problems) Question Bank 1] — Radar

solution; BW = Bandwidth (Hz) “. The peak pulse power of the radar is


62.4 MW.
2 = signal-to-noise ratio (dB)
2Viii F =noise factor
i zai k= Boltzmann's constant = 1.38 x 1027 1°K

2(26.7)(10%10") Solution:
a 184s
Ge 3x10° Calculate the radial velocity of the
S=-174 dBm +NF *10logat +=
i =1,78 KHz target, if the Doppler frequency seen by
Pain = RT{BW (F -1) a stationary radar is. 1 KHz and the radar
where: S=-174 410 +10leg(2x10°)+0
P.,, =(1.38x107 })(290)(2.0x10" )(100 ~ 1) transmit a CW ata frequency of9 GHz.
eee

V, = 96 kph $=-101dBm
kr hry 1000.m Pine eeax to Ww
ee

= $5 ——x =26.7 mis Hotapeava: 2. The minimum signal received for


. Sr 3600 km where|
satisfactory reception is -107 dBm.
-. The Doppler shift in frequency is F=k9"| |= 100 4 = radial velocity (m/s)
1.78 kHz.
f;=Dopoler trequency (H2}
f= operating frequency of the radar {Hz} aL
-. The minimum signal received is 4. = signal wavelength (m) A deep space radar has a noise figure
edit) 7.923 x10 W. of 10 dB, IF bandwidth of 2.0 MHz, a
Solution: +7 dBm third order intercept and 0.dB
A deeo-space radar of Air
SIN, and the sensitivity level is— 100
Transportation Office has a duty cycle of PROBLEM 7 dBm, Calculate the dynamic range af
25%. What is the PW ifthe PRT is 6
the receiver.
ims? A space radar is used to detect
unidentified flying object 500:km from
the earths’ surface. What is the peak Motations
Tatratinna: pulse power of the radar if the-minimum
recelvable power is 10°, the capture DR = dynamic range (dB)
PW = pulse-width (ms) area of the antenna is. 10m’, and the
PAT = pulse repetition time (ms) target's cross-sectional area is 30 mat Solution:
10 GHz operating frequency?
Solution:
DR = = (input intercept - noise floor)

(7 -(-100)]
Mosteatione:
PW 2
Duty cycle ~ BRT DR=—|7 -{-100
fra = Maxkrium radar range (rr)
“. The radial velocity of the target is OR=71.33'c8
PW = (Duly cycle}(PRT) P;= peak pulse power of the radar (WV)
16.67 mis.
PW =(0.25)(6 ms) =1.5ms Ay = capture area of the receiving antenna (m*)
$= radar cross section or effective area (m*} “. The dynamic range of the receiver is
Poy = mintmum-receivable power-of the recenver {W)) 74.33 dB.
«. The pulse width is 1.5 ms. dit]
Solution: A deep space radar has a noise figure
of 10:08, IF bandwidth of 2.0 MHz, a 011
dey he) PAZS iia +7 dBm third order intercept and 0. dB NASA's deep space radar has the
A deep spate radar has a noise figure man, 4n-P,,, S/N. What.is the minimum signal following parameters:
of 20 dB and IF bandwidth of 2.0 MHz. ir received for satisfactory reception? Operating frequency — 3 GHz
What is: the minimum signal received for ad (4n3°P.,,) Peak pulse power 20 MW
satisfactory reception? R= A2S Fotatioaces
Antenna diameter —— 32m.
Target cross section—— 1.5 m?
aU is (500,000) 4 Cees) 3 (1x10)gh § = minimum signal received for satisfactory Noise factor cm 15
Receiver bandwidth —— & KHz.
reception (dBm)
est ah
Pirie = minimum power received (\W') -
(30) (15) NF = noise liqure (cB)
What is the maxirmum range of the
fadar?
T= room temperature {"K) Py= 92.4 MW At = IF bandwidth (MHz)

11-4 11-5
The Question Bank Series - Communications Engineering (Volume 2 - Problems)

Notations:

= peak value of transmitted pulse power (W)


D = antenna diameter (rm)
$= radar cross section or effective area (i)
BW = Banchidth (Hz)
#= wavelength (m)}
NF = noise factor

Solution:

es

r= 48 _ D'S
sai BW? (NF —1)
——

— aa) 20810" 22") 0.8) be


Powe | (Bxi08)(0.
4) (1 5-9
SSF

fax = 45,202km
—_—a

Question Bank 12

WAVEGUIDES
=

11-6
The Question Bank Series — Communications Engineering (Volume 2 - Problems)

Flutations

P, = peak valueo! transmitted pulse power (WV)


f= antenha diameter (m)
$= radar cross section or effective area (m")
BW.= Bandwidth (Hz)
A, = wavelength. (Mm)
NE = noise factor
SS

Solution;
——

‘ ae pps aia

on BW." (NF 1)
20x10°(32")(1 5) ‘i
(Bx10?}(0.4(1.5 -1)
ma Tas Tn Bg cen nak

Ix = 45,202 km
ee
i

Question Bank 12
WAVEGUIDES
<a

11-6
Question Bank 12 —Waveguice
a 1a as Aa e)
What is the cut-off frequency ofa The cutoff frequency of a TEs mode is
regular waveguide with a width of 0.65 3.75 GHz In an alr-dielectric waveguide
inch and a height of 0.38 inch? with an inside cross-section of 2.cm by 4
em. What is the characteristic
impedance of this waveguide at the
Notations operating frequency of 10 GHz?

k= cut-off frequency (MHz) Flotationa:


a= width (m)

‘Solution: 2, = characteristic impedance (Q)


ie = cutoll frequency (GHz)
[= operating frequéncy (GHz)
300
Ee Solution’
300
k=
2) (0.65)(2,64x10°7) Zs 277
{, = 9,085 MHz
“ The cut-off frequency of a regular
le
waveguide is 9,085 MHz. Z,= ar7

a0 ed
2, = 406,682
Determine the cut-off wavelength of a
cifcular waveguide having an intemal
fadius of 10 em if the solution ofa
“The characteristic Impedance of this
Bessel function equation (kr) is 3.83. waveguide is 406.68 0.

Notaticrs:
a]
Ae = Cut-off weavelengith (mn) A rectangular waveguide has a width of
f= circular waveguide
internal radius (m) 0,65 in, and a heightof 0.38 in, What is
ir =salution of a Bessel function equation the typical operating frequency of this
waveguide?
Solution:

Zar Hinations:
1G i)
f= operating frequency (GHz)
2n(10x10-*) a= width (Mm)
to 393 fe =:cutofl frequency (GHz)
2, =164.05 mm Solution:

~. The cut-off wavelength of a circular The typical operating frequency is given by


waveguide is 164.05 mm. oo ie OO
f oF D7 12,98 GHz

12-3
The Question Bank Series - Communications Engineering (Volume 2 - Problems) Question Bank 12 - Waveguide

where: dd ed
Flojatinnn:
“Ty ko
oa. SOD
a gost 1, = qutofl wavelength (m)
A cassegrain feed sectoral horn antenna
“= 20.65 nx O84 4 {= carol frequency
{GHz}
uses @ waveguide with an inner cross.
section af 10 by 20 mm. Calculate the
ax10°
{, = 9.085 GHz Vp = propagation velocity (m/s) cutoff frequency for TE. mode. AG =

Solution saxt08 [ It-{ 7 (78 iI


.. The typical operating frequency is
Potatnons:
* \ (42
12.98 GHz, Assuming dominant mode, ig = 0.082 m
(= autott frequency(GHz)
‘A. za. m=i
1]
0 m a= waveguide sadth (m) . The length of the antenna slot is
2(1.2) c= wave vekcity = 2x10 mis 16 mim.
Past ECE Board Problem
A rectangular waveguide used for
anaes Solution:
i, 22.4 in =0.061 m
microwave transmission has a width of aint)
1.4 inches and a height of 0.8 inches. All
signals above GHz will be al A wave is propagated in a rectangular
passed by the waveguide. 28 waveguide with/plane separation of 6
+» 3x 10° om by 3 cm (inside measurements),
=— =7.5 GH
The cutoff frequency
Is 4.9 GHz. f= 3(0.020) . Calculate the cutoff frequency of the
TE. mode
Hlatatinas

~The cutoff frequency for TEis mode is. Pahitiona


‘Ag = cutoff wavelength (rn) ata ey 7.6 GHz.
{. = cutolf frequency (GHz)
Past ECE Board Probiem Tinn = wave Quide mode
a = width (in)
A rectangular waveguide has a width of m=landn=1
Vp = propagation velocity (m/s) ad
0.6 inch. Calculate the waveguide cut- {, = cutoll frequency (Hz)
Solution: off frequency A slot antenna is used in a waveguide to 1 = widest sepviration belween walls (1m)
radiate EM wave. Suppose the b= separation between wali (m)
Assuming dominant mode, J latatnont Operating frequency of the signal is 12
GHz and the cutoff frequency of the Solution;
2a waveguide is 7.5 GHz, how long should
ia m= Ae = Cutoll wavelength (im)
4 m the antenna slot be installed ina Rectangular weveguita TE,
L.= colt frequency (itt)
waveguide?
hi 24.26 in=0.071.m = propagation velocty {m/s}
Ca (ey fay
ts Vo Solution: Vlatatrins
= ON a \b)

rs oe Assuming dominant mode, (m= 1) L = lenght of the antenna-stot (mm)


= ee
0.071
422 Gite 2a 2(08)
1.s2eosS— Ae = guide wavelength (m)
m 1 4 = signal wavelength (m)
- The cutoff frequency is 4.22 GHz. te aa i= 0.030450 f= cutoff frequency (GHz)
1 = signal frequency (GH?) «The cutoff frequency of the TE;;
© = wave velocity {m/s} mode is 5.8 GHz.
Prt)
:14 9 08
Past ECE Board Problem t= 0.03048 = 9.64 GHz
ad) 90
A rectangular waveguide hasa width of
1,2.in, and a height of 0,7 in. The <The wavequide cut-off frequency A rectangular waveguide with inside
waveguide will pass all signals above 9.84 GHz. measurement of 5 cm x 2 comand the
GHz. operating frequency is 6 GHz
Determine the guide wavelength for the
dominant mode

124 12-5
‘The Question Bank Series— Communications Engineering (Volume 2-Problems) Question
Bank 12— Waveguide

Viotatinens: if ow | a1 | 2a 2(25)
V.=c) h-|2 | |=3xt0* (0.868) —seSo 5

Ag = quide wavelength (rm) SPW aed A rectangular waveguide has an inside


measurement of 5.0 cm x 2.5 cm.
m 4

de = cutoll wavelength (1m) Calculate the cutoff wavelength, ina


V, =26x 10" mis standard rectangular waveguide, for the
A. = signal wavelength (1m) TMi; mode.
f= sighal frequency (GHz) ». The group velocity is 2.6 x 10° mis.
c= wave welocity (m/s)
Hotutiess:
Solution:
aa) 4 -0.78
A circular waveguide is used for conical Tien = wave guide mode hy
For dominant mode, m=1
horn antenna. If the internal diameter of m=landn=1 4 =0.78(),) =0.78(8)=3.89om
Jo= cutott wavelength (cm) 1
f== ©223x10! 27.7 GHz
a =28 28) <0 on the waveguide is 5 cm fora 12 GHz
operating frequency, what is the a = widest separation
between walls (cm)
b = separation
between walls (cm)
ah 0.0388
. acne characteristic impedance of the guide in
aa ieane ae a dominant mode?
Solution:
“The frequency of the signal in the
dominant mode Is 7,7 GHz.
Hoedutwns 2
‘ Jevay +(niby
yi 7 Ke (3) cteri
2, = charaimpedance stic
of the quide ((2) a
cteri
aa = charaimpedance stic
of free space (02)
Te) h
A = cotoll wavelengt(m) he 7 2
(V5) + (1/2)
a
A wave is propagated in the dominant
mode using rectangular waveguide
A. = signal wareelenygtth (m) having 4 = 2.5 cm and b =1,25 cm and
“The guide wavelength for the
1 = ctoular wave guid
internal radiuse(m)
i, =3.71em 2.5 GHz operating frequency, What is
dominant mode ta 16.8 cm. the voltage attenuation provided by a
Wy =solution ol a Bessel function equation
.. The cutoff wavelength Is 3.71 cm, 150 mm length of waveguide?
ai) 0d Solution’

A rectangular waveguide with inside a ne Flrstatiiven:

measurement of cm x 2 cm andthe A rectangular waveguide with a wall


operating frequency is 6 GHz, Calcutate separation of 2.5 cm, propagate a signal
Ag = cutolt wavelength (cm)
the corresponding group velocity. with unknown frequency, If the a= width (cm)
characteristic impedance of the fay = vollage altenuation (dB)
waveguide is 600 ©, what is the {= Waveguide Jengih (cm)
Flotation: frequency of the signal in the dominant
made? Solution:
Vg = group velocity (m{s}
& = cutotl wavelength (m)
4. = signal wavelength (m) _ 2nf _ 2n(25) _
Notations:

{ = signal frequency (GH2)


c= wave wekocily (m/s} “(ke)” (4) 2. = characteristic
impedance of the quide (92)
kr = 1.84 for TE,, mode @ = characteristic
impedance of free space (£2)
Solution: 2, = cutoff wavelength(m)
o .. The characteristic impedance of the A= signal wavelength (m)
hes =$on guide in a dominant mode fs 394 (2 4 = widest separation between walls (cm)
he =23=2(5)=10'on
1 = signal frequency (Hz)
Solution: a, = 16.35 dB
For dominant mode m= 1 .. The voltage attenuation is 16.35 dB.

12-6 12-7
The Question Bank Series — Communications Engineering (Volume 2 - Probicms)

tt)
‘Suppose the two waveguide circular and
rectangularis to have the same cutoff
wavelength for its dominant mode. What
is the width of the rectangular
waveguide ifthe radius of the circular
waveguide is 3 om?
“tl

VDioteatitien

Aye) = cutott wavelength of circular quide (mn)


Ager} = Guitot wavelength of rectangular quide (tm)
4, = signal wavelength (m)
a= width (om)
P= cipcdar waveguide internal radius (m)
kr =solution of & Besse! function equation

esae
Solution:

For the dominant TE, mode

a
oS
ina ciroular waveguide,
2m) 2nr
eta “(ey see

Area of a cirde
=A, rr?
In the rectangular waveguide,
for TE mode
2a
hye = oa 2a 2a
Preomey = Megiay

2a
¥

ar)
=f Tiel
(kr) 1.64
a=5.120om
Question Bank 13
The width of the rectangular
waveguide is 5.12 cm. F) DIGITAL & DATA COMMUNICTIONS

12s
*
The Question Bank Series— Communications Engineering (Volume 2 - Problems)

Pa ae
Suppose the two waveguide circular and
rectangular isto have the same cutoff
wavelength for its dominant mode. What
is the width of the rectangular
waveguide if the radius of the circular
waveguide is 3.cm?

Wotations

Aecee) = Cutol wavelength of circular guide: (m)


Aefrec) = culot wavelength of rectangular quide (m}
2. = signal wavelength (m)
a= width (cm)
r= clecular waveguide internal radnus (Mm)
kr =solulidn ofa Bessel function equatian

Solution:

For the dominant TE,, mode


ina circular waveguide,
2a
Mnan =aox SeaQn eet,
(kr) 1.84
Area of a circle
= A, =ar?
In the rectangular waveguide,

Question Bank 13
. The width of the rectangular
waveguide is §.12 cm. DIGITAL & DATA COMMUNICTIONS

12-8
Question Bank 13 — Digital and Data Communications

e110 PROBLEM 3
In data communications, how many Ona 10 Mbps Ethemet packet format,
errors can be expected in the how fast can a 1500-byte block of data
transmission of eight sequential 512- be transmitted?
byte packets if the system BER is 2 x
10°?
Solution:

FTiuatione:
time for 1 bit =10x
—\—10 =100 ns

time for1 byte =8 x 100= 800 ns


BER = Bit Error Rate
time for 1526 byte = 1526 x B00 ns
Solution: time for 1526 byte = 1.2208 ms

8 packets x 512 bytes = 4/096 bytes Note: For Ethernet (original)


4,096 bytes x 6 bits = 32,768 bits
1526 byte =-1500 bytes (data) + 8 bytes
32,768 (preamble) +6 bytes (destination address)
= 9.2768 sets of 10,000 bits
10,000 +6 bytes (source address) + 2 bytes
Error = 2(3,2768) (type) + 4 bytes (frame check sequence)
Error = 6.5536 bits
~ The time required for a 1500 byte of
.. The errors in the transmission Is data to be transmitted Is 1.2208 ms.
7 bits.
a)A a
PROBLEM 2 Past ECE Board Problem
In an office with 20.PGs, how many Calculate the transmitted data stream
interconnecting wires or links are for the following data and CRC
required so that any computer can generating polynomials:
communicate with any other computer? Data G (x) = x raised to the power of7
+X raised to the power5 + x raised to
the power4+ x raised to the 2nd
Phatathnee
power +x + one
P (x) =x ratsed to power5 + x raised to
power4 +x + one
N= number of links
f= number of terminats
Votutianes:
Solution:
CRC = Cyclic Redundancy Check
_hin=1) BCS = Biock Check Sequence
Nie
n= 220-2 _ 190 Solution:

Multiply G(x) by the number of bits in


>. The number of wires or links needed
the GRE code.
is 190.
xi(xi axa bP xls x)
SRT
RE Be gt
= 1011011100000

13-3
The Question Bank Series — Communications Engineering (Volume 2 - Problems) Question Bank 13 - Digital and Data Communications

Divide the data message G(x) by Tse i Solution: Motertinits:


a generator polynomial function P(x), Past ECE Board Problem
For an Ethernet bus that is 500 meters a Vina OR = dynamic range (absolute value}
Discard the qoutient and the n= number of PCM bits exchading sign bit
in length, using a cable witha velocity Resolution
femainder is truncated to 76 bits. factor of 0.66, and a communication rate 1 Vv.
Add to the message as the block of 10 Mb/s, calculate the total number of Resolution =—@2*. Solution:
DR
check sequence (BCS) bits that would be sent by each station
before it detects a collision, if both mens: 2.55 V Pon DR=2"-1
The result is the transmitted data stream: Resolution = 197 O0.02V¥
stations begin to transmit at the same DR=27-1=511
Transmitted data time. where: where:
= 1011071101001 DR'= 2"=1; n=8 -1 (excluding sign bit) n=no, of bits (excluding sign bit)
.. The transmitted data stream Is
Viatatites:
OR = 2! ~1=127 n=10-1<3

10011071707001,
Vp = propagation velocity {m/s} a The resolution of an 8-bit linear sign -. The dynamic range is 614. .
c= wave veloaty (m/s) magnitude PCM is 0.02 V.
PROBLEM 5 \ = velocity factar
K = total number of bits (bits) PROBLEM 10
Past ECE Board Problem |. = cable length (m) PROBLEM 8
4 telephone line has a bandwidth of 3.2 Past ECE Board Problem
kHz anda signal-to-noise ratio of 36':dB- Past ECE Board Problem For a dynamic range of 120, how many
Solution:
A signal ls transmitted dawn the line Fora sample rate of 40 kHz, determine bits are required in the POM code
using a three level code. Calculate the the maximum analog input frequency. (including the sign bit)?
maximum theoretical data rate taking Vp =o.)
into account the presence of noise.
V, = (300 x 10”}(0.66) Potattents: Flatatsrt

V,=198 x 10°mis £; = minimum Nyquist sample rate (Hz)


Hotations: OR = dynamic range {absolute value)
K=6 Nf
f; = highest frequency to be sampled (Hz) n= number of PCM bits excluding sign bat
|
C= channel capacity (bit/sec)
BW = banctwidth (Hz) mh 1 a
Solution: Solution:
S/N =signal-to-nolse ratio (absolute value} K=| 10—— || ———__~—_|(500
| sec |\198x 707 m Fp 9 O9(DR +1)
f, = 2f, = 40 kHz
Solution: K= 25 bits >
where: log 2

c=Es 3.328Wlog| | 3}= “The total number of bits sent before it


detects a collision is 25 bits.
_tog(120
- Sie +1) _2 6.9

© = 3.32(3.2 kHz)log(t + 3981.07) _ 40. kHz n= 7 bits (excluding sign bit)


C= 36-25 kbps. PROBLEM 7 eee Total numberof bits
i, = 20 kHz =n+ 4 (with (sign bit}
where : Past ECE Board Problem
S23 ( 98:\_ Determine the resolution for an 8-bit = 6 bits
we ($2 |- 3061.07 2 The maximum input frequency ts
linear sign-magnitude PCM for a 20 kHz.
maximum decoded voltage of 2.55 Vp. ” The total number of bits including
+. The maximum theoretical data rate is
sign bitis 8.
38.25 kbps. aia) ae
Votutions:
Past ECE Board Problem id 161-30
DR = dynamic range (absolute value) Determine the dynamic range for a 10-
bit sign magnitude code.
Past ECE Board Problem
Veor = Maximum decoded valtage (V) How many Hamming bits would have to
n= number of POM bits excluding sign bit be added toa data block containing 128
bits?

13-4 13-5
The Question Bank Series - Communications Engineering (Volume 2 - Problems) Question Bank 13 — Digital and Data Communications

Tlitations: ed)
1 a Solution: Vas = output voltage (¥)
For digital communications, determine
n= number of Hamming bits Int +h Mn_| Solution:
the signal to noise ratio in dB which
‘m = data bits would be required for an ideal channel Ve ‘oul ~~= aVena J
"m, In(t + nw)

with a bandwidth of 2500 Hz. {eine


Solution:
In| 1+ cass 228)| out ea
2) >men+4 Hota tignter
Vout = in( + 255) 1+ 1n(67.8) 22)
Try n=,
C= channel capacity (bps) Vou = 0.75 V
21284744 BH = bandwidth (Hz) You ine7.e)
ya
128= 136 (not true) N= number of coding Mvels “The output voltage of a p-law Vou = 0196 ¥
Try n=, compander is 0.76 V.
Solution: . The output voltage is 0.96 V.
Ssi28+a+H
256 241238 (true) For an idea! channel, PROBLEM 15
C = 2BWlog, N For binary phase shift keying (BPSK) at) ee
. The number of Hamming bits added modulation with a camer fraquency of
with N = 2 (binary ceding) What is the Shannon limit for
is 8. 80 MHz and an input bit rate of 10 information capacity for a-standard voice
C= 2BW = 2(2500) Mbps, determine the minimum Nyquist band communications channel witha
O=5000 bps bandwidth signal-to-noise ratio of 1000 (30 dB) and
a0) Ba Bg a bandwidth of 2.7 KHz7
For a practical channel
Past ECE Board Problem Hoatatipns:
(with some noise present),
An-asynchronous communication ,
systam uses ASCII at 9600 bits/sec with C= 3.32BWiog(1+S/N) {y= minimum Nyquist bandwidth (Hz) Vlotationa:
eight bits, one start bit, one stop bit and S000 = $,.32(2500) log (1+S/N) iy = Wwansmission rate (bps)
no party biL Express the data rate in C = charinel capacity (bps)
S/N=3
words per minute, (Assume a word has Solution: BW = bandwidth (Hz)
five characters or letters and one space) In dB, S/N = signal-to-noise ratio (abSotute value)
SIN = 10i0g'3 For BPSK,
Solution: SIN = 4.77 dB fy =f, =10 MHz Solution:

Bit rate = $600 bits/sec -. The signal-to-noise ratio Is 4,77 dB. .. The minimum Nyquist bandwidth is S
C= BY 1+ —
= 576,000 bits/imin
10 MHz. ot 3)
10 bits per character a) G=2700legs(1+ 1000)
PROBLEM 16 C=26.9 kbps
(8 bits + 1 start bit+ 1 stop bit) Determine the output vollage of a ptlaw
One. word = 5 characters + one space compander with a maximum voltage In A-law companding, determine the
. The reflector coupling factor is 0.971.
range of 1 V and an input of 0.25 V. output voltage if the input voltage to the
compander is 0.8 Vand the maximum
The rate in words per minute (wom)
possible input voltage is 1 V.
a2 ia
: (576,000 oF sae IS) Hotations:
In PCM, assuming unifonn quantization,
min) 6 characters J 10 bits Mota tions:
= 9600 wom Vow = output voltage (V} what is the signal-to-noise ratio in dB for
Vij = amplitude of the signal ata particular instant al, Vin = amplitude of the signal at a partiqular instant of an B-bit code word?
“The data rate in words per minute is time (V/) tine
Vox = Maxinum uncompressed analog input Vier = Maximunt uncompressed analog input Hotatians:
9600 wpm.
amplitude amplitude
A= parameter used to define the amount of S/N = signal-to-noise ratio (dB)
LH = parameter used to define the amount of
compression = 255 (lypical) compression = &7,6. (typical) n= number bits

13-6 13-7
The Question Bank Series - Communications Engineering (Volume 2 - Problems) Question Bank 13 — Digital and Data Communications

Solution: Solution: Solution: Plolatrener:

f and
and eent
f, a $) c- 3.328Wiog| ' = C = channel capacity (bps)
=6n+1.8
Sl

BW = bandwidth (Ha)
i=
2, 2(5 MHz) C=3.32(4 kHz}log(1+ 10) N= number of coding fevels
= 6(8)+1.6
Slo

Q | G=13.83 kbps
f, = 10 Mbps Solution:
where:
= 49.8 dB
=|

5 § Without noise,
-. The effective radiated power is —(dB)
nt ) =10log—
oy
1577.61 W. €=26Wlog, N
“, The signal-to-noise ratio is 49.8 dB, 5 4f 10 dB
== a | G = 2(36 MHz)log,2
7 10
PROBLEM 19 bd 119 a C= 72 Mbps

Past ECE Board Problem » The channel capacity is 13.83 kbps. where:
What is the signal-to-noise ratio ina
A-receiver has a sensitivity of 0.6 N=2 (binary coding)
PCM system with 512 quantizing levels
microV and a blocking dynamic range of BW = 36 MHz typical bandwidth
using uniform quantization?
60 dB. Whatis the strongest signal that a1 ea)
can be present alongwith a 0.6 microV per transponder}
Past ECE Board Problem
signal without blocking taking place’? if the transmission rate of a digital
Pata tiinitn:
communication system of 10 Mbps
’, The data rate of the transponder is
Potatoes: modulation scheme used is 16 QAM,
72 Mbps.
S/N = signal-to-noise ratio (dB)
M = number of output conditions possible determine the bandwidth efficiency
} = Strongest signal (V)
Vo = weakest signal (V) edit) 13 de
Solution: Past ECE Board Problem
Vlotirterrin:
Solution: For digital communications, determine
=4 Blog| © | i, = transmission rate (bps) the signal to noise ratio in dB which
| on
ae

Blocking dynamic range = 20log-! would be required for an idea! channel


{y= minimum bancwidth required (Hz)
l |
Zz
with a bandwidth of 2500 Hz.
=10log) 2 (5127
Zhi

60 dB =20lo) 4 Solution!
706 pv
L2 Jj
in.

=== 59/95 do 60 fed Veta teint:


Ve ,= =0:0.6 pViog saree
35 BW efficiency = t = ;
=

Hl =
4 € = channei capacity (bps)
-. The signal-to-noise ratio is 55.95 dB. VW, =0.6
mV =600 jv
BW efficiency = 4 bits/cycle BW = bandwidth (Hz)
N= number of coding levels
«The strongest signal is 600 pV. where for 16 QAM,
S/N =signal-to-noise ratio (dB)
PROBLEM 20
=a f
Past ECE Board Problem Solution:
In digital modulation system, what is the
Paid)] ead
input rate to MSK modulator, if the mark Past ECE Board Problem + The bandwidth efficiency is For an ideal channel,
frequency is 5 MHz. Assume nis equal Determine the channel capacity of a4 4 bitsicycle. C=28Wlog, N
fo. unity kHz channel with S/N = 10 0B:
G = 2BW = 2(2500)
PMoatations: et C= 5000 bps
Votiitings: Past ECE Board Problem with N= 2 (binary coding}
{, = mark frequency (Hz) Assuming there is negligible noise, what Fora practical channel
{; = space frequency (Hz] C= channel capacity (bps) would be the data rate of one satellite
{, = bit rate (bps) WW = bandwidlh (Hz) transponder used for binary data (with some noise present),
n= any odd integer SIN = signal-to-noise ratio (absolute value} transmission? CG =3.328Wiog(1+S/N)

13-8 13-3
The Question Bank Series -Communications Engineering (Volume 2- Problems) Question Bank 13 = Digital and Data Communications

5000 = 3.32(2500) log (1+5/N) Solution:


Plidations: Plitatiinra:

S/N =3 C= BWiog f ea
ee N Pras =peak power (W) fh, = transmission.rate (bps)
In dB, A = signal level (V) fy = mingnum bandwidth required (Hz)
S/N =10log 3: =4,77 dB C= 10x10? lag.(1+ 31.6)
7 = impedance (Q)
C=603 kbps Solution
~. The signal-to-noise ratio is 4.77 dB, Solution:
where: f
BW efficiency = zi
a Ae) =S =log"!|(83) | =316 hi
nN? lio For QPSK, fy = i
Past ECE Board Problem
A telephone line has a bandwadth of 3.2 “The channel capacity is 50.3 kbps,
kHz and a signal-to-noise ratio of 34 dB . The peak power is 24 yW. BW efficiency = 0 = 2 bitsieycle
ib
A-signal is transmitted down this line 2
using a four-level code, What is the a) es
maximum theoretical data rate? ate)
1A edt)
An A/D converter uses 14-bit numbers- .. The bandwidth efficiency Is
Past ECE Board Problem 2 bits/cycle.
and has a voltage range of -6 to +6 V.
Hotatenes: What is the bandwidth required to
What is the resolution of digitization
transmit at a rate of 10 Mbits/sec in the
expressed as the smallest voltage
C= channel capaoty (bps) presence of a 28 dB S/N ratio? a]4 MCPs
increment?
BW = banchwicth (Hz)
a/N =signal-to-noise ratio (absolute value) Past ECE Board Problem
Flotationa:
Given the pulse-spreading constant
‘Solution:
Hotties: equal ta 10 nsim andthe cable length
C= channel capacity (bps)
équal to. 100 meters, determine the
EW = bandwidth (Hz)
OR = dynamic range (absolute value) maximum bit rate in Mbps for UPNRZ
C=3,328Wiog(1+S/N) V = total votlage range {¥)
_ S/N =signal-to-noise ratio (absolute value}
transmission.
C = 3.32(3.2x10" jlog(1+ 2511.87) n= number of PCM bits exclading sign bit Solution:
C= 36.12 kbps Solution: = Pietanticn:
where: C=3)32BW1 0al “nl
{+=
SiN = 34 dB = 2511.87 The total voltage range is -6 V to +6 V f, =transmission rate {hps)
=12V
pwe— 1G + 3) 10 Mbps At= pulse spreading (sec/m)
>, The maximum data rate is 36.12 kbps, Vv
Re solution =—_ = ———
12V
3.3291 3. 32log(1+ 630.96) L= cable Jengih (mn)
DR 24 i

12V BW = 1.075 MHz Solution:


bat)
CS aed Resolution = Bay
where: For UPNRZ transmission,
4 radio channel has a bandwidth of 10 Resolution = 732.47 pV
kHz and a signal-to-noise ratio ef 15. dB 3 4/28
mee => |= 630.96 1
What is the maximum data rate thal can ’. The resolution of digitization N (3) f=
be transmitted using any system? 2 2AP KL
expressed as the smallest voltage
range is 732.47 wV. -. The bandwidth required Is 1.075 MHz. ‘
ee
Noiatons: " 2(toxt0-*\(100) F
ia ee ee
C= channel capacity (bps)
Past ECE Board Preblem Past ECE Board Problem >. The maximum bit rate is 0.5 Mbps.
BW = banebwidth (Hz)
Calculate the peak power ina 120 Determine the bandwidth efficiency for
‘S/N signal-to-noise ratio (absolute value] OPSK modulation scheme ata
millivolts NRZ square wave on a 600-
ohm channel of a digital system. transmission rate of 10 Mbps.

13-10 13-11
The Question Bank Series — Gommunications Engineering (Volume 2- Problems) Question Bank 14 — Digital and Data Communications

PROBLEM 33 bad ee) ¢, The absolute minimum bandwidth PROBLEM 39


required is & kHz. Past ECE Board Problem
Past ECE Board Problem Past ECE Board Problem
Whatis the channel! capacity for signal What is the dynamic range in dB of an Determine the dynamic range for a 10
power of 200 W) noise!power of 10 W 8-bit linear sign-magnitude PCM bit sign-magnitude PCM code.
and a bandwidth of 2 KHz of a digital spectrum whose maximum decoded
bah ees
system? voltage at the receiver is 1.27V,? Past ECE Board Problem
Pita pene
For binary phase shift keying (BPSK)
modulation with a carrier frequency of
Thta tinea: Vliotutéita: 80 MHzand an input bit rate of 10 DR = dynamic range (dB)
Mbps, determine the minimum Nyquist n= number of PCM bits -exchading sign bat
C= channel capacity. (bps) DR =‘dynamic range (dB) bandwidth.
Solution;
BW:= bandwidth (He) n= number of PCM bits exchutling siqry bit
S/N =signal-to-noise ratio (absolute value) Hotuticns:
Solution: 2"-1=DR
Solution: {, = transmission rate (bps]
For min, OR,
OR=2"-1 fy = minimum bandwidth required (Hz)
DR=2"-4
GC=3 S28Wlog| 1+ 5) bps DR=2"-12127 Solution:
1 \ N .
DR=29-1=514
Rag = 20logDR = 20log127 = 42/1 0B
In dB,
G=3,32(2x 10° Jog 1 pee where: For BPSK,
DR = 20 log'511 = 54 dB
n=8-1 (excluding sign bit) fy =) <10 MHz where:
G=8.779 kbps
n=7 bits f= 10-1 =9 (excluding sign bit)
“. The power gain of the antenna is
“The channel capacity is 8.779 kbps. 4.61.
.. The dynamic range is 127 or 42.1 dB,
“The dynamic range is 611 or 64 dB,

a dE) bd)
Past ECE Board Problem
ditt)
1A Ee)
Past ECE Board Problem Past ECE Board Problem
What is the bandwidth needed to Fora sample rate of 30 KHz ina PCM Past ECE Board Problem
A binary digital signal is to be
Support a capacity of 20,000 bits/s
transmitted at 10 kbits’s, what absolute system, determine the maximum analog A coherent binary phase shift keyed
(using Shannon's theary), when the ratio minimum bandwidth is required to pass input frequency. BPSK transmitter operates at a bit rale
of power to noise is 2007 of 20 Mbps with a carrier to noise ratio
the fastest information change
CIN of 8.4-cB. Find E,/No.
undistorted?
Phistatsona:
Fatatioive
C= channel capacity (bps) Ploshustirara Fleatiatiiea:
{= minimum Nyquist sample rate. (Hz)
BW = bandwidth (Hz) {, = highest frequency to be sampled (Hz)
S/N =siqnal-to-nolse ratio (absolute value) C= channel capacity (bps) BN, = energy per bit-lo-nalse power density ratio
BW = bandwidth (Hz) Solution: (dB)
Solution: N = number of coding levets C/N = carrier-to-noise ratio {dB}
f, = 24, Bi = bandwidth (Hz)
Solution: 4, = transmission rate (bps)
C =3.328Wiog| 1+
210

ae fy 30 KH 15 kHz
2 2
aie St 2g.900 From Hartley Law, Solution:
$.32log| A =| - -4.32l0g(1+ 200) C=268Wiog, N
binky For binary coding,
.. The maximum analog Input
frequency is 15 kHz. EsEy (4B)ey -10 logeaye
c +10 log Bw
te
BW = 2615.54 Hz capil fy
Es<8. 8.8.d8 +10 log| 4
— |= 8.808
- The bandwidth needed is 2615.64 Hz. BW = g 10 Kops N,, 8 eal 35 /
BW =5 kHz

13-12 13-13
The Question Bank Series - Communications Engineering (Volume 2 - Problems) Question Bank 13 - Digital and Data Communications

where: .. The signal-to-noise ratio is 67.8 dB, where; Todiatinns

CG ‘
10 log ==
‘ §.808
5 == =log* We |= ix to! {, = transmission rate (bps)
ite) A é
fy = minimum bandwidth required (He)
=20 Mbps
Assuming uniform quantizing in PCM ~The bandwidth needed js 3.85 kHz.
BW = 20 MHz (Note: BW =f, for BPSK) system, whatis the S/N if there are 128 Solution:

ra
levels of quantization?
., The energy per bit-to noise power at] JS Be} For QPSK,
density ratio is 8.8 dB.
Flotation:
Past ECE Board Problem f=3fe =_ 10
5 | = MHz
A broadcast TV channel has a
ae) ie bandwidth of G6 MHz. Ignoring noise,
S/N = signal-to-noise ratio (dB) “The minimum Nyquist bandwidth is
calculate the maximum data rate that
Past ECE Board Problem M = number of output conditions possible & MHz.
could be carried ina TV channel using 4
What is the transmission rate of a 16-level code and determine the:
system for transmitting the output of a Solution:
minimum possible signal-to-noise ratio
microcomputer to a line printer operating in dB for the calculated data rate.
at hare
al a speed of 30 lines/minute? Assume =10log) (30 | Past ECE Board Problem
zie

that the line printer has 8 bits of data per 12 voice channels are sampled at 6,000
character and prints out 40 Heilatiniag Sampling rate and-encoded inte 8-bit
characters/ine. = 10109) 3(20) | PCM word, Determine the rate of the
ani

C= channel capacity (bps) data stream,


=43.9 dB N= number of coding level
zit

Flatutions:
Bi! = bandwidth (H2}
S/N = signal-to-noise ratio (dB) Hlotatigns:
fh = transmission rate (bps) *. The signal-to-noise ratio is 43.9 dB.
Solution; 4, = transmission rate (bps)
Solution:

_fa, linesif .char\/.. bits.\/ min


ba Ae} Solution:
G = 2BW log, N
b (30 min jie line ] edie | tan) What bandwidth is needed to support a
f, =320 bps capacity of 128 kbps when the signal C= 2(6x10° }log, 16
power to noise power ratio in decibels is fs =| 855 \(12chann)|goo SSrPiS
C=48 Mbps
1007
. The transmission rate is 320 bps. C= 232BW
log (t+ S/N) f, = 768 kbps
48 Mbps = 3,32(6x10°)log(14-S/N)
Flucarticnuss «The rate of the data stream is
ei aa oe S/N = 255.83 768 kbps.
In an 74-bit code word with uniform C = channel capacity (bps) S/N=24 08
quantizing, what is the S/N? BW = banctwidth (Hz)
S/N =signal-to-noise ratio (absolute value} ©. The channel capacity is 48 Mbps and a etd
the signal-to-noise ratio is 24 dB,
Analog broadcast television has a total
Hotrtinns: Solution: bandwidth of6 MHz. What is the
eT) maximum data rate of the video signal,
S/N = signal-to-noise ratio (dB) a ignoring noise if the video signal uses 4
5
n= number of bits Cat328Wog| i | Past ECE Board Problem MHz of the 6 MHz bandwidth? Assume
C For quatemary phase shift keying that the video signal uses a G2-level
Solution: BW =——— (QPSK) modulation, data with a carrier code.
$.32log| 4 S| frequency of 70 MHz, and an input rate
5 7 ont 18 =6(11)+1.6 nN of 10 Mbps, determine the minimum
: 4128x102 Nyquist bandwidth, Vlataeiins:
s ~ 232iog(1+1 x 10"
nN = 67.6 dB ea : S/N = signal-to-noise rato (dB)
BW =3.85 kHz }
13-14 13-15
The Question Bank Series
— Communications Engineering (Volume 2 - Problems)

M= number of output conditions possible Vleitutinta:

Solution: S/N = signal-to-noise ratio (dB)


M = number of output conditions possible
C= 28log,M=2(4%10")(log,
32) C= channel capacty {bps}

C=40 Mbps Solution

.. The maximum data rate is Ideal channel


40 Mbps. C = 2Blog,M = 2{20x10" )log, 8
C=120 kbps
4 be
4, u-law compressor has a maximum With noise.
possible input signal of 25 V. lithe input C= 3.328Wiog| 1 eal bps
signalis 7 V, what is the output-signal/ of
the compressor?
Gs 3.32(20 x 40" log(1+ 4000)
VTatintinaia: G = 200 kbps
where:
Vou = output voltage S_. 4/30
Vin = amplitude of the signal at a particular instant of —=log°:) — |=1000
se K\a 5
time
Veas = MaxITIUM uncompressed analog input
“The maximum theoretical bit rate is
amplitude
200 kbps
(= parameter used to define the amount of
compression
= 255 (typlcal)

Solution:
eee in(1 +
i #3)
ou ae
in(t+u);

' 7
in| 14 255 25
Vil =19.3.V
TT TL
-. The output signal of the compressor
is 19.3 V. Question

PROBLEM 50
FIBER OPTICS
A. certain analog cellular systam uses
FM with channel bandwidth of 20 kHz. If
the channel were used for digital
communication, what is the maximum
theoretical bit rate using 8-level code if
the available signal-to-noise ratio is 30
dB?

13-16
The Question Bank Series — Communications Engineering (Volume 2 - Problems)

M = umber of output conditions possible Hooter tiviness

Solution: S/N = signal-to-noise ratié (dB)


M= number of output conditions possible
G = 2Blog, M= 2(4xt0*)(iog, 32) C= channel capacity (bps)

C= 40 Mbps Solution:

-. The maximum data rate is Ideal channel


40 Mbps. C = 2Blog, M = 2(20x107 flog, 6
C= 120 kbps
dT Ate)
With noise
A L-law compressor has a maximum
oF
possible input signal of 25 V. IFthe input C=3.308Wioa!1+=), b
signalis 7 ¥, what is the output signal-of oa 1+ 5} bP
the compressor? C= 3,32(20x 10° )log(1+ 1000)
Vietutinaa: C=200 kbps
where,
Vea = output vollage $_—=log 4/30"
Vn = aniplitude of the signal at a particular instant of | = |=1000
N tog \io)
time
Vous = Maximum uncompressed analog input
.. The maximum theoretical bit rate is
amolitude
200 kbps
ji = parameter used to define the amount of
compression
= 255 (typical)
Solution:
icin
Vue a mae alt Hines
IAT + ju)

n 1+ 265 z )
Vig 25 et
In(1 4-255)

~The output signal of the compressor


Is 19.3'V, °
Question Bank 14
FIBER OPTICS
a) ey)
A certain analog cellular system uses
FM with channel bandwidth of 20 KHz. If
the channel were used for digital
communication, what is the maximum
theoretical bit rate using B-level code if
the available signal-to-noise ratio is 30
dB?

13-16
Question Bank 14—- Fiber Optics

a8) ig]
38
For @ mult+mode step-index fiber with What is the number of modes that-an
glass core (ni=1.53) and a fused quartz odtical fiber can support if the
cladding (n2=1.44), determine the cntical normalized frequency is 10?
angle.
Hutu tions:
Hotareans:
M = number
of modes
O,; = critical angle (degrees) fy = normalized frequency (unitless)
ni = relractive index of the core (unitless)
ne = refractive index of the cladding (unitless) Solution:

Solution:
Mh12
0, = sin 122
3! }
4 , 2
M==(10
ui)
6. =sin
‘ =+ 1.44
—— M=50 modes
is 1.53
0: =70.25" “The number of nodes that the optical
fiber can support is 50 modes.
-. The critical angle of a multi-mode
step index fiber is 70.25°. PROBLEM 4
What minimum bandwidth is required to
PROBLEM 2 pass a pulse train without distortion if its
nse time 6 Bns?
Fora glass (n= 1.55) to ethy| alcohol
(n= 1.36) interface. and an angle of
incidence of 33 degrees, detennine the fictattonss
angle of refraction,
Von = minimum bandwidth (MHz)
FLoterteoman: t= rise time
{ 1.5)
= 6s ='0.006 ps
Oy = angle of incidence (degrees)
Selution:
Oz = angleof refraction (degrees)
Ty = refractive index af:the glass (unites)
fa = refractive index of the ethyl alcohol (unitless) BW, = 0.35
t
Solution:
0.35
n, sing, =n, sinGs BHM = 0.006
fn) BW, = 58.33 MHz
i, = sn sl sno,
Vp} p
The minimum bandwidth required
0 =sin (3 jana iF 38.37 is 65.33 MHz.

’ The angle of refraction is 38.37°.

14-3
The Question Bank Series —Communications Engineering (Volume 2 - Problems) Question Bank 14 —Fiber Optics
PROBLEM 5 . The fastest rise time of a pulse Solution:
Is. 1.76 ys. ve
In-a single-mode fiber cable, determine 5(43\(0.367)
the maximum allowable diameter of the
core for light frequency of 300 THz and
d=0.0127 psikm NA sain = ye
numerical aperture of 0.35. PROBLEM 7 d= 12.7 nsfkm NA = J(Q.5ty ~ (1.354)
The input square wave to an
/ The dispersion factor of the fiber NA = 0.67 i
oscillascope has a rise time of 15 ns.
Piatra What ts the rise time of the displayed optic cable is 12.7 nsikm,
” The numerical aperture of the cable
square wave if the bandwidth of the is 0.76. r
Gro = oaamUM allowable diameter of the core (pum) oscilloscope is 60 MHz?
Fras = Maximum allywable radius of the core {1irn) PROBLEM 9
NA = numerical aperture What is the maximum permissible value PROBLEM 11
Pitotpors:
for the pulse-spreading constant of 65
Solution: km length fiber that must not lengthen Past ECE Board Problem
1) = composite rise time of ampllier output (ns) An optical fiber and its cladding have fi |
the pulses by more than 200 ns?
tf: = rise time of input square wave (ns) relractive indexes of 1.55 and 1,32
0.383% ty = rise time of ampiier (ns) respectively. Determine the acceptance
Tran = NA Hlotiet iis angle
0.383(1 x 10°) Solution:
At= maximum permissibve pulse spreading (ns/km)
lua =a DD eAED
0.35 AT = maximum pulse spreading (ns) ti hal featas

fax = 109. um att? et L = length of the fiber optic cable (km)


0, = acceptance angle (degrees)
Ghiox = <iinae
pet (ry + (5.833) Solution: hi = refractive index of the: fiber
Gee = 2(1.09x10°*) b= TT. ns nz = retractive index-of the cladding
hae = 2.2 um
Solution
where
. The maximum allowable diameter of 0.35° 0,35
the core is 2.2 um. = —— 2 —— = 0.005833
"a BW. 60 ia i= sin” ni ¥ ns
t5=5.633ins
At=3,08-ns/km 0, = sin! (7.55) (1.42)°
ed 81 0, = 54.34
~The composite rise time of the
What is the fastest rise time that a circuit “The maximum permissible pulse
amplifier output is 17.7 ns. spreading Is 3.08 ns/km.
will pass a pulse train faithfully ifthe “The acceptance angle of the fiber is
bandwidth is 200 kHz? 64.34 degrees.
bide]
Sd sy PROBLEM 10
HTolytimn: The length of the fiber-optic cable is Past ECE Board Problem ae Na bs
0.367 km. lts:maximum data rate is 43
An optical fiber and its cladding have
BW, = minimum barchwielth (MHz) Mbps. VVhat is its dispersion factor? refractive indexes of 151 and 1.454
Past ECE Board Problem
t, = fastest rise dime (1s) = 6s = 0.006 ps Medium 1 is made ofsilican and
respectively, Determine the numencal
medium 2 is made of glass. Their
aperture.
Solution: Hletietione
refractive indices are-3.3 and 1.3
respectively, Foran angle of refraction
a5 d= dispersion factor (j5/km) Hotectiies:
of 30.5 degrees, determine the angle of
BW = a R= maximum data rate (Mbps) incidence,
D = length of the fiber optic cable (km)
0:38
BY Solution:
NA = numerical aperture
Ty = refractive index of the core Volutinis
Ne = relractive index of the clackding
0.36 |
t =—— = 1.75 =——
r= "02 om SRD 6) = angle of inédence (degrees)
Q2 = angle of refraction (degrees)
14-4 14-5
Question Bank 14 —-Fiber Optics
The Question Bank Series— Communications Engineering (Volume 2 - Problems)

fy) = refractive index of medium. 1 FT sta Hatta: Votutioma PROBLEM 18


np = refractive index of medium 2 Past ECE Board Problem :
H.= acceplance angle (deqrees) 6) = angle of incidence (degrees) For a multi-mode step-index fiber with a
Solution: NA = numerical aperture G2 = angle uf refraction (Gegrees) glass core (ny = 1.55) and a fused
fy = refractive index of medium + quartz cladding (ny = 1.46), determine
n,sinf, =n, sing, Solution: ng = refractive: index of medium 2 ihe critical angle.

, = sin (2sing, | NA =sin 6, Solution:


PToduti¢ne:
ny #
NA = sin20
PY a ect
0, = sin ie one! NA = 0.94
n,sing, =n sinh,
4, = critical-angle (degrees)
By =i ino fi) = refractive index of the glass core
6, =41.53 “. The: numerical aperture of the fiber is Hy n; = relractive index of the cladding
0.34,
‘ es
+, The angle of incidence is 11.53°. ,=sin-'| sin =sin32
[si Solution:
aT 6,=10417
el 1909 Past ECE Board Problem
Past ECE Board Problem An eptical fiberand its cladding have + The angle of incidence is 10.17°.
refractive indexes of 1.33 and 1.23
For a multi-mode step-index fiber with
glass core (n1=1.53) and a fused quartz respectively. Determine the numerical
cladding (n2=1,44), determine the aperture. PROBLEM 17 t, = 70-38
critical angle. Past ECE Board Problem
Vlatirtaina: An-optic fiber is made of glass with a .. The critical angle is 70.38°.
surface index of 1.55 and is clad with
Flatationsy another glass with a refractive Index of
NA= numerical aperture
fy = refractive index of the optical fiber 1.51. Launching takes place on air. ee
0, = critical angle (degrees) termine the numerical aperture of the
Ne = refractive index of the cladding Past ECE Board Problem
ny = refractive index of the glass core fiber. Mediuirn 1 is made of glass, while
fe = refractive index of the cladding
Solution: medium 2 is made of ethyl alcohol, Their
refractive indexes are 1.3 and 1.28
Solution; Vlatations: respectively, For an angle of incidence
NA = Jn? —n37 of 34 degrees, determine the angle of
NA = numerical. aperture refraction,
NA = ,/(1.43)° - (1.23)
fi
8, = sin)
C n, ni = rélractive index of the glass. core
NA = 0.506 fy = refractive index of Ihe glass cladding
ay 444
4. = sin ——— THatations:
= 1.53 Solution:
* The numerical aperture of the fiber is
0, = 70.25 0.506, 8) = angle of incidence (degrees)
NA = Jn? —n? 6; = angle of refraction (degrees)
-. The critical angle is-70.25°. ny = refractive index of tnedivm 1
PROBLEM 16 NA = (1.55) ~(1.51)7 fi = refractive indexoot medium 2
Past ECE Board Problem NA =0:35
a | Medium 7 is made of silicon and Solution:
medium 2 is made of glass. Their
Past ECE Board Probiem «The numerical aperture of the fiber is
Tha numencal aperture of a fiber, if the refractive indexes are 3 and 1 0.35. n,sin6, =n, Sint,
angle of acceptance is 20 degrees is respectively. For an angle of refraction ‘
ni r]
of 32 degrees, determine the angle of 5 -= sin | Bsina,
incidence, Na

14-6
The Question Bank Series - Communications Engineering (Volume 2 - Problems) Question Bank 14 — Fiber Optics

k , , Vlatatioas Plinteetisivan:
NA=Jnp —n27
8, =sin" Fe biting 8), = acceptance angle (degrees) NA= numerical aperture
1.28 ny = ne ~NA*
fy = refractive index of the liber = refractive index of the optical fiber
0, =34.61 = relractive index.of the cladding nz = refractive index.of the cladding nz = (1.55) -(0.092)° = 1,547
.. The angle of refraction is 34.61°. Solution: Solution: .. The refractive index of the cladding is

NA= jh? -ne


1.547.
te)
1 ed Ga= singin ny
Past ECE Board Problem Oj, = sin 4/(4,38)° - (1.21° NA =, /(1-381)' (1.29) PROBLEM 25
Mediurn 1 is made of silicon and
0, = 36.77 NA = 0.493 Past ECE Roard Problem
medium 2 is made of glass. Their Medium 1 is made of silicon and
refractive indexes are 3.1 and 1.1 .. The numerical aperture of the optical medium 2 is made of glass. Their
respectively, For an:anole of refraction -. The acceptance angle is 36.77°. fiber is 0.493. rafractive indexes are 3.2 and 1.2
of 31.5 degrees, determine the angle of raspectively. For an angle of refraction .
incidence. of 31 degrees, detenmine the angle of
di 3 edt) 1 ee) incidence.
Past ECE Board Problem Past ECE Board Problem
Flatatioma: A fiber optic. cable has a loss of 16 A single-mode fiberis made with a core
dB/km. What would be its attenuation if HToartiesnan:
O) = angle of incidence (degrees) diameter of 10 micrometers and is
the cable is 2000 ft. long? coupled to a light source with a
0. = angle of pelraction (deqrees)
wavelength of 1.2 micrometers. Its core 6) = angle of incidence (degrees)
= refractive index of medi 4 glass has a refractive index of 1.55. 02= angle of refraction (degrees)
= relractwe index af medium 2 Votctines:
Determine the cladding index required ny = refractive index of medium 4
for producing single-mode propagation. n= refractive index of medium 2
Solution Ly = tolal loss (08) (WV cut off = 2.405)
a= attenuation constant (dB/km) Solution:
n, sing; =n, sin, = lénath (kin)
Votutinsa= nsing, =n sind,
0, =sin Pesin, Solution:
ny NA = numerical aperture 6,= sin" [2 jn
14 L;=aD ny = refractive index of the core
8, = sin? Ea) sina. n= relractive index of the cladding
dB im 1 km
Lr=|
16 — || 2000-ftx. -———
es) 3.28 ft asco
@, = sin it 55 mst
é, = 10.68" Solution:
L,=9.76dB
a,=11.14 |
-. The angle of incidence is 10.68°. V number= k.a.NA
.. The total loss of the fiber optic The angle of incidence is 11.14°.
cable is 9.76 dB.
_V number
bg UT | ka

Past ECE Board Problem PROBLEM 23 PROBLEM 26: i


@ 2nf “n= an
An-optical fiber and its cladding have Past ECE Board Problem
refractive indexes of 1.35 and 1:21 Past ECE Board Problem c h 1.2 pm
YWedium 4 is made of silicon and |
respectively. Determine the acceptance An optical fiber and its cladding have
medium 2 is made of glass. Their
angle. refractive indexes of 7.387 and 1.29 = Sim
refractive indexes are 3.4 and 1.4
respectively. Determine the numerical
2.405 respectively. For an angle of refraction
aperture: = 0.092
if
of 30: degrees, determine the angle of
erzs um) incidence.

14-8
The Question Bank Series — Communications Engineering (Volume 2 - Problems) Question Rank 14 — Fiber Optics
Hotertitine: PROBLEM 28 “The anergy?of the photon is Sclution:
Past ECE Board Problem 1.28% 10°
6); = angle of incidence (degrees) An optic fiber made of glass has a SYS (ns) 1.15? +DeEGs
= angle of refraction (degrees) refractive index of 1.56 and Is clad with
ni = felractive index of medium 1 another glass whose refractive index is ba)
DN ed) SY5 (ns}=16 ns
nz = refractive index of mediuen 2 1.51. Determine the acceptance angle
when launch In air,
Past ECE Board Problem
Ifa fiber optic system has a rise time of
2. (SYS).
Solution: }
38.55 ns, the source rise time is 12.ns
and the detector rise time |s 12 ns, what
hy sing, =n, sing, Moleationtss is the cable rise time?

6, - sin’ Ffng)
[ens | Oi, = acceptance angle (degrees) c= (8) “4 5) - (2)
tf = refractive Index of the glass core Flotatrenes:
hire: ne =relractive index of the qiass cladding
=14 ns
6, =sin flee
14 )sinao

0
5. = source rise time (ns)
Solution: 0 = detector mse time (ing) .. The cable rise time is 14 ns.
6, =14.88 C= cable rise time (ns)
Gi, shasine
h salinlaed
in? aes<n; re S¥S= system rise time (ns)
“The angle of incidence is 11.88. PROBLEM 32
6, =sin-’ \(1.66)° - (1.51) Solution:
Past ECE Board Problem
a OT a By, = 23,077 An optic fiber has a core diameter of 50
SYS (ns)=1,1VS? +D? + C? um and is used ata median light
Past ECE Board Problem wavelength of 0.28 jum. Its numerical
“The acceptance angle is 23.07°. SYS (ns)= 38:55 ns
For a glass (n= 1.55) to ethyl aleohol aperture is 0.343. Determine the V
(nf = 1.36) interface, and an angle of or = (S18) @2. p? number,
incidence of 33 degrees, determine the
angle of refraction PROBLEM 29
Past ECE Board Problem
Calculate the energy of the photan of
C= oy
1.1
_§?_p? Vlotiticana:

Vototponsa: infrared light energy al 1.55 jm. NA = numerical aperture


0) = angle of incidence (degrees) o= Lee 52) (2 (02)
(sess) _ eee
Fars ny = relractive index of the tore
fa = refractive index of the cladding
O; = angle of refraction (degrees) Vlotatinny:
C =30.66
ns
fy = relractive index of the glass Solution:
nz = refractive index of the ethy? alcohal E = energy (1) <The cable rise time is 30.66 ns.
h = Planck's constant, 6.6256 x 10° |/Hz Vnumber=k.a.NA
Solution: {= frequency (Hz}
c= speed af Bght (m/s) edit)
2] Nt DH ke nu ===
here: 2nf 2a
— Gino aen
n, sino, =n, sino, A= wavelength (m)
Past ECE Board Problem
Ifa fiber optic system has a rise time of ee 7.66 x 10°
0 =sin | EJs Solution: 16 ns, the source rise time is 1.5 ns and
~ 082% 10

|
BS
the detector rise time is 2 ns, what is the a= core radius
1.55 E=hf, Joule (4) cable rise time?
a
ndaSraSOs 25 um
6, =sin lee
c_3x10°m/s =1.9395%
foe=s 10"H
0, = 38.37 a 4.55 um : ; Motartenns:
V number= (7,66 x 10°}(25 x 10°)(0.343)
= (6.6256 x 10°*)(1.935 x 10)
“. The angle of refraction is 38.37°.
§ = source rise time [ns] V number= 65.7
E=1.28x 107%) D = detector rise time (ns)
C= rable rise time (ns) » The V number Is 65.7.
SYS= system rise time: (ns)
14-10 14-11
The Question Bank Series —- Communications Engineering (Volume 2 - Problems) Question Hank 14 — Fiber Optics

dN Be | a a) where: aN eT)
Find the refractive index of the fiber Calculate the external acceptance angle cosd, = + In? <n?
: 2 Given thatthe acceptance cone half
made of glass ifthe core glass refractive of glass clad fiber with 0.35 numerical angle of the fiber is 25°. What should
index is 1.55 and the cladding is doped apertures, ny
the refractive index of ihe cladding if the
to give a fractional difference of 0.035, ni? cos, = jn? —ne fiber optic glass core refractive index is”
1.627
Flatatinns;
Fla ntiivats 2 The numerical aperture is 0.4962,
O, = extemal acceptance angle (degrees) [intl ticiitad
A = fractional difference NA = nurnercal aperture
fy = refractive index of the glass care
a)8 ed By = acceptance cone half angle (degrees|
fy = refractive index of the cladding Solution: A fiber optic is made of glass core with Ny = refractive index of the core
refractive index of 1.62. What is the ip = refractive Index of the cladding
Solution: acceptance cone half angle ifthe
sing, (max) =NA
internal critical angle is 82°? Solution: i
Q,(max) = sin 'NA= sin '(0.35)
ee fa (max) = 20.49" Piotatrows assuming the light is launch in the air
My
2 Fy
ny = An, +n, =n,(1+ A) O; = critical internal reflection angle (degrees) sinf,(max)
ny =f
= ie
- The external acceptance angle of the
ny =1,5(14+ 0.035) glass clad is 20.49°, Oo = acceptance cone half-angle (degrees} Ng

Ny = 1.5525 ny = relractive indexof the core sin fl, (max) = nent


nz =relractive index.of the cladding
8) TAT) NA = furnencal aperture nz =n} —(sind,(max))°
~. The fractional difference is 1.5525.
What is the numerical aperture of the
Solution:
nz =1.62" —(sin25) = 2.446
fiber made of glass with 1.5 core
PROBLEM 34 refractive Index and the critical internal n, = 2.446 =1.564
reflection angle of the light is 85°? * een .
An optical fiberis designed to give a . = + + = c08(82) * The refractive index of the cladding is
numerical aperture of 0.25. What is the nm
1.564.
fractional differance between the core cost. = 0.1392
and the cladding if the fiber is made of Flotatieitas
glass with refractive index of 1.517 iny —ny =m, (0.1392)
bate
Oc = critical imernal reflection angle (degrees)
1) = refractive index of the core
gn? <n} = (1.62)' (0.1392)
= 0.2652 A fiber glass has a core diameter of 60
Hotatrons:
ne = refractive index-of the cladding Assuming the light is faunch in the air
pm and numerical aperture of 0.235.
NA = numerical aperture What is its V-number if the fiber is

(iisns
NA= numerical aperture operates with light source wavelength of
A= fractional difference 0.735 pm?
ny = refractive index of the core SOMMOH, sind, (max) =
zat nm,
Solution: NA vine sing), (max) = Jn? ~n2 Hotiat jist:

No
NA =n.J2a Se: light is launch in the air
A, (max) = sin!(0.3652) d= core diameter (jm)
O.(max)=21.423' HA = numencal aperture
NA =,/n; —n;
x (NA) 2802 iN
Asal ite kg = baht source wavelength (1m)
al 161) NRE cote “ The acceptance cone half angle is
Solution:
A=0.0137 NA =1,5° (0.0872) 21.423°.
NA =0.1962
Vo number = pe NA
' s The fractional difference is 0.0137.
ho

14-12 14-13
The Question Bank Series —-Gommunications Engineering (Volume 2 - Problems)
Question Bank 14 - Fiber Optics
Flatationa: Solution:
( ae 1A |
fie: ¢

V number =
{ *
*| a or750 235) d= core diameter (jum) A= a FER What is the maximum bit rate allowed of
NA = numerical aperture the fiber if the total dispersion of the
= relractive index of the core
1.55(1000)(0.065) fiber is 20 ns? Assume dispersion
Ve number =60.3
= fight source wavelength (jum) ~~x10°(1—0.065) limiting
“. The V-number is 60.3. A= fractional difference Ay = 359.2 ns/km
where: Pouitions:
Solution:
A=n—n, =1:55-1.485
a nae le = maximum bit rate [Mbps}
A=0.065 = rise time (M5)
A fiber glass.has a core diameter of 25
(Nay Total dispersion = A,L=359.2(20) he intermodal disperskon
um and numerical aperture of 0.135
How many modes will it support if the \ md Total dispersion = 7.18 us
fiber operates with light source NA = /A2(n,) Solution:
Wavelength of 0.835 jam?
NA = ,/(0.0007)(2) (1.55) “The total dispersion Is 7.78 jis.
ran
meu
— Bt

Vioteat nis NA =0.058 dispersion limiting t, = At


a a
d = core diameter [uum)
NA (cutoff) = vont A fiber with core index 1.5 and cladding 3... mae =——
1
BAt

NA = numerical aperture
ie |
is doped to -have a fractional difference {
Ag = light source wavelength (4m) of 0.0035, Calculate the intermodal Bags = = 10 Mbps
J Vicuton) fa) dispersion considering multimode um (8)(20x10"*) ‘
Solution: graded-index fiber.
| NA(cuteff) | x J
-| 2.ae (44.5) Ue “The maximum bit rate Is 10: Mbps.
Hatatiena:
V number= «LNA 0068 || x |
hy
d=19.8 pm = retractive index of the core Aa
V number c=! {oa
sass \ 0.435 ) A = refractive index fractional diflerence A germanium doped single-mode fiber is:
«The maximum diameter of the core is Ay = intermodal dispersion designed to have a intermodal
M number =12.7 19.8 pm. c= speed of light dispersion of 22 ns, material dispersion
7= 1000 of 3.25 ns, and wavequide dispersion of
Nimodes) = 3 number} 0.525 ns. What is the maximum bit rate
PROBLEM 42 Solution: of the fiber if the transmitted pulse-width
N(modes)
= (12,7) =81 Find the total intermodal dispersion ofa is 0.6557
step index multimode fiber 20 km length, E n,za®
>. There are 81 modes that this fiber given that the core index is 1.55,
optic cable can support. Cladding index is: 1.485. usar Viintaat eines

4.5(1000)(0.0035)" A; = resultant dispersion


PROBLEM 41 Va tatiniia: i 8(3x10") A, {ima} = intermodal dispersion
A, = 7.66 ps/km Ay (md) = material dispersion
A fiber made of glass is made to operate
ny = relractive index of the core A; (Wad} = waveguide dispersion
as single mode, The core has a
A= relractive index {rectional difference t= rise time. (ns)
refractive index of 1.55, and the “The intermodal dispersion is
fractional difierence-of the fiber is Ay = Intermodal dispersion 7.66 ps/km. t. = pulse widih (ins)
0.0004. What is the maximum diameter L=length of the-fiber: optic
of the core fits operating ata 2= 1000 Solution:
wavelength of 1.5 um? For single mode
Vicutoff} = 2.405 For single-mode, zero intermodal
dispersion

14-14 14-15
The Question Bank Series - Communications Engineering (Volume 2 - Problems) Question Bank 14 —- Fiber Optics

A, = \/A? (ima)+ 4? (md) +4? (wad) Maximum bit rate PROBLEM 48 Hoviutinines

A, = 0? 43.25" 40.525" = 3.29 ns Ba - ae Past ECE Board Probiem A. = Angstrom is a practical unit of wavelength for
Received pulse
St, 20 angstroms is equal to how many higher frequencies like light frequencies
microns?
Lata, Be me ee
(5)(22.895 x 10°) Solution:
| =0:65+3.29
t, = 3.94 ns
Byse = 8.74 Mbits/s Fiotittina:
a
Cc

A, = Angstrom is a practical unit of wavelength for f


Maximum bit rate “The maximum bit rate is 8.74 Mbps,
higher frequencies like light frequencies Poe ax 10"
1
B yr 322 x 107"
5t,
fare ead es
Solution: A= 0.9317 pm
1 a)
hae =
(5)(3.94 x 10°) A fiber glass with a total dispersion of 15 :
41 A=0,0001 micron
1 A=0.0001 th
ns/km, and the pulse width of the
B,.., = 50.8 Mbits/s transmitted signal is 0.5 ns. Whatis the eet
1A =O317 A
maximurn length of the fiber to handle & [20 A) seotmaron
micron | = 0.002 micron cee | sa = a8
.. The maximum bit rate is 50.8 Mbps. Mbits/sec without repeaters? 1A
2. 20 Angstroms is 0.002 micron. ~. The wavelength in Angstroms
is 9347.
aaa eT) Hotties;

A multi-mode silica fiber is designed to Baa, = maximum bit rate (Mbps) PROBLEM 49
have a intermodal dispersion of 22 ns,
material dispersion of 3.25 ns, and A, = resultant dispersion (ns/km) Past ECE Board Problem
waveguide dispersion of 0.525 ns. What
t= rise time (ms) 400 Angstroms is equal to how many
is the maximum bit rate of the fiber if the te = pulse width (ns) microns?
transmitted pulse width is.0.65 ns? L = maximum length of the fiber (km)

Solution: Pitations:
FRatatiams:
A. = Angstrom is a practical unit of wavelength for
1 higher frequencies like light requencies
A, = resultant dispersion Brnax * 5p
A, (imd) = intermadal dispersion 1 Solution:
Av (md) = material dispersion tr
Ay (wgd) = waveguide dispersion | SBrnax
t= rise time (ns) 1 + A =0.0001 micron
{: =——__=25 ns
ty = pulse width (ns) i (5)(8x10°}
{100 a} 0.0001 micron | = 0.01 micron
Solution: Received pulse 1A
t= ty tA
A, = [Ai imd) + aF (md) +33 (wad) pein
At
«400 Angstroms is 0.01 microns.

A, = \222 44.25" 40.525? L=


25-0.5
15 = 1.63 km ae ae
A, = 22.245 ns
Past ECE Board Problem
Received pulse Determine the wavelength in angstroms
“The maximum length of the fiber is
batprd, 1.63 km, for the light frequency 322 THz,
{, = 0.654 22.245
|, = 22.895 ns

14-16 14-17
The Question Bank Series - Communications Engineering (Volume 2 - Problems)

Votes

uestion Bank 15
TELEPHONY

14-18
Question Bank 15 — ‘Telephony
ba) PROBLEM 3
A province in the Philippines has an A callphone used by a woman Inside'a
area of 2000 sq. kms. It has to be traveling car is operated at 825 MHz. IF
covered by cellular mobile telephone the car is traveling at 90 kph then find
service using calls with a radius of 2 tha time between fades,
kms. Assuming hexagonal cells, find the
number of call sites needed.
Flatationa:

Votes tiniest t= hme between fades (sec)


¢ =speed of light (m/s)
N = number of cell sites f= frequency (Hz)
f= cell site radius v= speed of the car (m/s)

Solution: Solution:
A
Ne 3464?
_2000
3.464(2)" ie 3x10°m/s
N= 144.34 celis
N= 145 cells
where: be t2ems
A= 2000 km?
~The time between fades is 7.27 ms.
r=2km

.. The number of cell sites needed is


approximately 145.
a TH
Calculate the via net loss (VNL) ofa
telephone signal that takes 3.ms to
PROBLEM 2 reach its destination for an acceptable
amount of echo,
Mr, Cruz uses tone-dialing phone to dial
the number 784-3742. If the digits
pushbuttons are held down for 0.25.5 Hlotutivais
and the interdigit time is also 0.25's5,
how long will it take him to dial the said VNL = via net loss (8)
number? t= one-way delay time (ms)

Solution:
Solution:
VAL= 0.2t+ 0.4
Time for the digits =7 x0.25= 1.755 VNL=0.2(3)+0.4
Time for interdigit = 6% 0.25 = 1.505 VNL=1dB
Total time.= 1.75 + 1.50
Total time = 3.25 seconds ~The via net loss (VNLj is 1 dB.

.. The time required for him to dial is


3.25 seconds.

15-3
The Question Bank Series — Communications Engineering (Volume 2 - Problems) Question Bank 15 —Telephony
PROBLEM 5 PROBLEM 7 Patativiies PROBLEM 11
In acellular system design, whalis the In cellular system, what antenna Past ECE Board Problem
frequency pause factor if the co-channel separation |s required if the antenna C = channel capacity (bits/s) lfa switching machine ofa telephone
interference reduction factor is 4.67 height at the base station is 19m? BW = channel bandwidth (Hz) sysiem is sel fo accept pulses ata rate
BW = 4.1 kH? for @ standard telephone channel of 10 pulses persec,, with 60% break,
S/N = signal-to-noise ratio (absolute value} find the make interval of the switch,
toty tpueie?

Heatatteeriee: Solution:
k = frequency reuse factor Solution:
q) = co-channel interference reduction factor $= antenna separation (m) By Shannon-Harlley Theorem
h =antenna height (m} Make duration
Solution: 5
Cc = 9 928g +—=| = Pulsing period - Break duration
Solution:
2.
= 100 ms - 60 ms
Ieee C =3.32(3.1 kHz)log(1+ 1024)
9

“I = 40ms
reas, C =30.99 kbps
k= (4.6) 11 11
where:
a °. The capacity of the telephone Pulsing periad = a =100.ms
5=1.436m
k=7 channel is 30.99 kbps.
-, The antenna separation required Break duration = eens = 60 ms
The frequency reuse factor is 7. is 1.36 m.
a TA)
.. The make interval of the switch
PROBLEM 6 Past ECE Board Problem
ditt] By Atelephone signal takes 2 quarters of a
is 40 ms.
In cellular system cell-splitting, what is 4 certain cellular telephone company second to reach its destination.
the numberof cells required fora Calculate the via nel ioss necessary for
100,000 m7 given area assuming
has 1000 cells with 1,000,000 PROBLEM 12
subscribers. If each subscriber uses the an acceptable amount of echo,
hexagonal cells? The radius of a circle phone on the average of 45 minutes per Past ECE Board Problem
inserbed in the hexagon is 100m. day then determine the average traffic in Telephone communication takes place
Enangs per cell, Histatinics: between two earth stations via satellite
36,000 km apart, Suppose Jane calls
Hotationes; Susan at 6 O'clock am, Susan
VNL = via net loss (dB)
PTotiitwaies: t= one-way delay time (ms) immediately answers back. How much
A= total area to be covered = 100,000 m* time has. elapsed before Jane hears
a= area of one cell (hexagonal area) A= average trafic per cell (Eriangs) Solution: back ihe beginning of Susan's reply?
a= 3.46r = 3.461100)? = 34600 me
Solution:
Solution VNL =(0.2t+0.4) dB
Petitions:

4,000,000)( 45m /d)/ fh — acelina VNL =.0.2(500)+0.4


nee a x arta) VNL = 100.4 dB t= time elapsed (sec) |
a 1000 calls d = distance (m)
where: t= (1) = 0:5 5=500'ms v= speed of light (m/s)
N= 100,000 A = 31.25 Enlangs
24,600 Solution:
N=2.89 cells “The average per cell is 31.25 Eriangs. “The via net loss mecessary for an
N= 3 cells acceptable amount of echo is

{oa
100.4 dB.
eA a v
“, The number of cells required is
Past ECE Board Problem t= 72,000.x 10°
approximately 3. =O0.24'5
A telephone channel has an S/N of axi0°
1024. Determine the capacity of the The total time delay is 2(0.24s)='0.48 s.
telephone channel,

15-4 15-5
The Question Bank Series — Communications Engineering (Volume 2 - Problems) Question Bank 15 — Telephony

where : *. The antenna separation required ina where: A=2000km* Ar = total resistance: (02)
cellular system is 1.82 m.
d =2(36,000 km)= 72,000 km r=2km
Solution:
v=3x 10'm/s «The number of cell sites needed is
a) 9}
approximately 145 cells.
.. The time that has elapsed before Past ECE Board Problem
Jane hears back the beginning of How often will hand-offs occur when
Susan's reply is 0.468 5. vehicle travels through a CMTS at 100 PROBLEM 17
km per hour speed if the distance
The voltage across the telephone line
between cellsites is 10.kms?
bed 0) TT decreases by 90% when the telephane
goes off-hook. The internal resistance of
Past ECE Board Problem the instrument when in use is 220 ohms
A telephone signal takes 11.8 ms ito Plotutions:
and open circuit when not in use. If the Rr aeVa
reach its destination. Calculate the via source voltage is 48 V, find the current
net loss required for an acceptable t=time elapsed (sac) that flows when the phone is off hook.
d = diameter (mj R= 120 = Ga7 10
amount of echo 31 mA
Vv = speed of light (m/s)
I Tootart itn
Ri =R, +R,
Solution:
Flotaiieas: Ra =Ry-R,
V= source volage (V/)
Ry, =387 1-320
VL = via net loss (dB) R = internal resistance (£2)
Rp = 35510
t= one-way delay time (ms) vo 27.8mis |= current (A)
t=719.42s
solution: Solution: * The ringer internal resistance
where:
is 3551 ohms.
d=diameter=2(10)=20km
VAL = 0.2 + 0.4 0B Off-heok condition
v= 100 knvhr= 27.6 mis
VNL = 0.2(11/8) 40.4 dB V=48(1-0.9)-48V ate tS)
VNL = 2:76 dB «. The hand-offs occur every 719.42 s. “Vv A.PGS uses 1.9 GHz carrier frequency
If the callphone is in the car and is
”, The via net loss (VNL) is 2.76 dB. moving at BO km/hr how rapidly would
8) 84] j= 4:8
220 94 52ma the signal fade?
4 province in the Philippines has an
baa) 39 | area of 2000 sq. kms. It has to be “. The current that flows when the [starters
Past ECE Board Problem covered by cellular mobile telephone phone is off-hook is 21.82 mA.
What antenna separation is required in service using cells with a radius of 2
{= time between fades (sec)
a callular system if the antenna height at kms, Assuming hexagonal cells, find the
¢= speed of fight (m/s)
the base station is 20 m? nurnber of callsites needed a AE
{= frequency (Hz)
A telephone system generates 120 V ¥ = speed ‘ofthe car (m/s)
Flatitions
Pinta tims: ringing voltage on the line. If the local
loop resistance is 320 ohms, calculate Solution:
d=antenna'separation (m)
N= humber ol cellsites the ringer intemal resistarice if the line
h = antenna height (m) r= cellste radius (m) current. is 31 mA.
A= coverage area (m*)
Solution: ,
Solution:
Tt shad tone
a: axio! fs
h h A Va = ringing waltage (V) 2(4, 8x10" Wao}(ix1O" y(3600)
==1l)d=— = 5
d VW 3.464r° Ay = loon resistance (£24 T=3.55 ms
|) = bine. current (A)
d= 04.82m
20m
= —2000 = 44434 calls = 145 cells “The signal fades in 3.65 ms.
| 3.464 (2) Ra = ringer internal resistance (£2)

15-6 15-7
The Question Bank Series — Communications Engineering (Volume 2 - Problems)

bed 027 edt)


An ECE is asked to determine the
maximum number of hexagonal calls
with 20 km radius needed to covera
certain area with 300 km radius.

Hatations:

N= number of cellsites
F = ceusite radius (m)
A= coverage area (m*)

‘Solution:

Reese
3.464P
a
AT
See
3.464(1}
__x_(300 }
~ 3.464\ 20
N= 204 calls
“The maximum number of hexagonal
cells is 204,

Question Bank 16
ACOUSTICS -

15-8
The Question Bank Series -Communications Enginesring (Volume 2 - Problems)

ates] Need]
An ECE (s asked to determine the
maximum number of hexagonal cells
with 20 km radius needed to cover a
certain area with 300 km radius:

Vladations:

N = number of cellsites
r= celisite radius 4m)
A= coverage area (m‘)

Solution:
A
N= Faear
a
3.46412)

en (202)
3.464) 20
N= 204 calls

“The maximum number of hexagonal


cells js 204.

Question Bank 16
ACOUSTICS ~

15-8
Question Bank 18 — Acoustics

PROBLEM 1
Wy
Determine the sound power froma ** Bnr®
motor car whose SPL ata distance of __ 10
7.6m is 87 dB, Assume that it radiates = = 3.98x10 Wim?
sound uniformly. 2n(20)
I; =l +l, =10.35x10 5 Wim?"
Victorians: ! 10.35
x 107
SPL, =10log= = 10log— > *
SPL = sound pressure level (dB) iE cai Me {4x 1077
PL = sound power level (dB) SPL,
= 100.15 dB

Solution: -. The SPL of the sound heard by the


listener is 100.15 dB.
SPL=PWL -20logr—8
87 =PWL—20log7.5-8 a OTN IR
PWL = 112.50 dB The basic voice band has approximately
PWL =10logW + 120 how many octaves?
W=0.18 W
Flotations:
“The sound power from a motor car is
0.18 W.
{, = the lower voice frequency = 300 Hz
f= the upper voice frequency = 3000 Hz
bhi)
1 Solution:
Two sounds of 4\W and 10 W powers
are produced at ground level at a No. of oclaves = 22h a)
distance of 10 m and 20 m respectively i
from a listener. If the ground is level,
|
unobstructed and non-absorbing, what No, of octaves = 2.320
will be the SPL of the sound heard by 300
the listener? No. of octaves = 3.32 octaves

©, The number of octaves is 3,32,


Notations:

|) & lp = sound intensities at 10 mand 20 m PROBLEM 4


respectively (Wim=)
hy = total sound intensity (W/m?) Ifthe loudness level is 100 phons, what
SPL = sound pressure level (dB) is this in sone unit?

Solution: Hotations:

12 Ww Phons = dB unit of loudness level


"2a? Sone = absolute unit of loudness (nat in dB)
4 '
= 2x(10)° Solution:
7

_j,--1('Phon=40) _
= 6.9710" Wim? Sone =log (Pra) 64.15

16-3
The Question Bank Series — Communications Engineering (Volume 2 ~ Problems) Question Bank 16 - Acoustics

“The sone value is 64.15. ate)


1 13 ed Wa) W=(4zr)I= | 4x(30.49) |(@.2x10* wim?)
For the safety and comfort of factory The sound intensity received from the
workers, the sound-intensity level ina nearby jet plane is 12.6 uW/m? and that W=3,74x107W
bd 00:18 3) certain factory must remain below BS from another jet plane some distance
How much louder is 80 dB over 60: dB? dB, What Is the maximum sound away 15 0.45 pVWim*. Find the relative @ 1 mile= 1609 m,
intensity allowed in this factory? loudness of the two.
W 3.74x10*W
HHotatiaics: dnt? 42(1608)°
Flotation: Motationss
Phons = dB unit of loudness level f= 1.15x10° Wim?
Sone = absolute unit of loudness (notin dB) \. = sound intensily bevel (dB) |, = sound intensity level (dB) 1.15x10°' Wim?
| = sound intensity (Wim*) | = sound intensity (W/m?) |, = 10log
07? Wim?
Solution: |, = 10.6 dB
Solution: Solution:
Phon—40\
Sone= og "|
33.2 } \ :. The intensity is 10.6 dB.
|. =10log— I =10log!2
80 dB =80 Phons=16 Sones
i, I
60 dB=60 Phons=4 Sones I 12.6 bad) 310
65 dB == 406910log Ot
——— | =10log——
+ Itls 4 times louder.
Sabine What ls the velocity of sound in dry air
| =14.5 dB when the temperature change is 15
1=3.2x107 Wim?
degree Celsius?
Oy «. The maximum sound intensity
” The relative loudness of the two is
14.5 dB.
What is the frequency of a 2-am sound allowed in the factory is Hatatiins
wave in sea water at 25°C? 3.2 x 107 Wim’.
ae) aa ea Te= temperature change (°C)
y =sound velocity (ms)
Flotiatinsis: PROBLEM 3 The noise from an airplane engine 100 ft
from.an observer is found to have an Solution:
What sound intensity comesponds to a
f= sound frequency (Hz) intensity of 45 dB. What will be the
sound-intensity level of 50 dB? intensity when the plane flles overhead
Ve = sound velocity (m/s)
For temperature change < 20°C
i= sound wavelength (m) atan altitude of 1 mi?
¥=331.45+0.607Tc
Hatadions:
Solution: V¥=331.4540.607(15)
Hotattons:
|, = sound intensity level (dB) ¥=340.5 mis
For sea water @ 25°C, | = sound intensity (Wim?) |, = sound intensity level (dB)
V, = 1520 mis ~. The velocity of sound in dry air
| =sound intensity (W/m)
Solution! is 340.6 mis.
W =sound: power (Wf)
i=

f=
| =40log—
a

, 1520 mis Solution: aa eg


I

~ 2x10?m 50 dB = 10log——— For a temperature change of 25 degree


f= 76 kHz So? | =10log! Celsius, what is the velocity of sound in
=1xi0" Wim? ' I dry air?
+ The frequency is 76 kHz. |
45 dB = 10log pao7
~The sound intensity that corresponds Flotaticns;
to 50 dB is 1x 107 Wim’. [=3.2x10 "Wim?
Tk = temperature change (°K)
im wait =100 ft=30.49 m 1c = temperature change (°C)
Bad
¥ =sound velocity (m/s)

16-4 16-5
The Question Bank Series- Communications Engineering (Volume 2 ~ Problems) Question Bank 16 — Acoustics
Solution; Solution f= frequency at which the absorption coelficientis 3.416 x10" wim
measured (Hz) = 10lo9——_——
For temperature change = 20°C NFO wene
Wik (ar , =25:0B
Solution:
LT 208 ‘2a VV
=991,4 He. <991145, 228 .. The intensity when the plane flies
ak S374 273 _ 330 |2(6x10 *) overhead at an altitude of 1 km is
V¥=346.3 mis an 15 f= i254"
f =13.56 Hz
Vv 25 dB.

where: 180
1000 = 12531180
Ty =273+ GC ». The resonant frequency of a V PROBLEM 18
Ty =2fd+25 Helmholtz resonator is 13.66 Hz, V=0.35.m" Past ECE Board Problem
T, #298 K A sound pressure (RMS) of 125
“The volume of the chamber required microbars is equivalent to what sound
PROBLEM 15 is 0.35 m’, pressure level?
«The velocity of sound is 346.3 mis.
A double-glazed window with intemal
glass of mass-7 kg/m" has an airgap of
PROBLEM 13 200 mm and ts lined with an-acoustic PROBLEM 17 Vleitiatavata:
absorbent. Find the approximate The noise from an airplane engine 100
Determine the number of levels ina expected resonant frequency of this SPL = sound pressure level (dB)
m from an observer is found to have an
compact dise audio system if the panel absorber? intensity of 45 dB. What-will be the P= Sound pressure (jibar)
number of bits per sample is 16.
intensity when the plane flies overhead P, = reference sound pressure = 0.0002 jabar
at an altitude of 1 km?
Wiest tecak
Hotations: Solution:
m= mass of the panel = 7 kgm? Holationie:
(4 = number af bevels
n= number of bits per sample
d = depth of the air space = 200 107 m SPL = 200g,
{, = resonant frequency of the panel (Hz) | = sound Intensity level (dB)
|= sound intensity (Wim?) 425, phar
Solution: Solution: }- 115,92 dB
M=2"
W = sound power (W)
eo 20109| ee
M=2" Solution:
-, 125 bars is equivalent to 115.92 dB.
M= 65,536 ed es | = 10g)
.. The number of levels is 65,536,
J7(200x10°) | PROBLEM 19
{, = 50,71Hz 45 dB
= 10log——._
ot wim! Past ECE Board Problem
2 2c ao Ifthe RMS sound pressure is 5 [b/sq_ft..
PROBLEM 14 “The approximate expected resonant what is the sound pressure level?
frequency of this pana! absorber ts
In'acoustics, what ls the resonant
60.74 Hz. 1=3.16% 10°Wiime?
frequency of a Helmholiz resonator
whose cavity volume is 15m anda j- Fist trem:
radius of 5 cm? Ant?
PROBLEM 16 W = (I)(4a1*) SPL = sound pressure level (dB)
In reverberation chamber method, whal P= sound pressure (Ib/it*)
Matatiaies
volume of the chamber is required if the W = (3.18 x 10° W/m") (42)(100)° | Pi = reference sound pressure = 2.084 (byt?
absorption coefficient is to be measured
v= velocity of sound in air = 330 m/s at 1000 Hz? W =3.97 x 10° watt = 3.97 mw Solution:
V = volume of cavity= 1.5 m? atr=1km=1000m
r=radius of peck =Sem=52 10%m Potential: _ 3.97 %10* watt
F916x 10 Wim"
4n(1000)
Y= volume of the chamber (rm) SPL = 7.6108
16-6 16-7
The Question Bank Series - Communications Engineering (Volume 2 - Problems) Question Bank 16 — Acoustics

“The sound pressure level is 7.61 dB. where: a 0.161 Solution:


| =500 candle
tea
at) ame)
d=12 feet =3.66 m
0.161(184,2) Ww
|= —,;} then la —
1
b= 4nr° o oe
Past ECE Board Problem
What frequency is 10 octaves above 30 @= 35,305 m* or metric sabines (eee Sat (Ne
“600 candle power provides 37.32 lux. 4n(ar)" Ol ane’,
Hz?
’. The absorption effect of the material
PROBLEM 22 is 35,305 m’ or metric sabines. .. The intensity Is reduced to 1/9 of Its
Hotutionts: original value.
Past ECE Board Problem
n= number of octaves What ts the increase in sound pressure aie) TAS re)
fy = the lowest frequency in the octave band (Hz) level in dB, if the pressure is increased
Past ECE Board Problem PROBLEM 26
r= the common ratio between octave bands four times?
Assume the speed of sound is 1000 fi's, Past ECE Board Problem
what frequency has a wavelength of 24 The sound power level ofa jet plane
Solution: inches? flying ai-a height of 1 km is 160°dB (re
Victattans:
40°"), What is the maximum sound
10th octave = 2fr"* SPL = sound pressure level (dB)
pressure level on the ground directly
Tlatectionss: below the plane assuming that the
10th octave =2 (30)(2""') P; =sound pressure (Pa) aircratt radiates sound equally in all
10th octave = 30,720 kHz Py = reference sound pressure (Paj A = sound wavelength (tt) directions?
Vp = sound velocity (ft/s)
. The frequency 10 octaves above 30 Solution: [= soured frequency (He)
Hz is 30.720 kHz. Flotation
SPL = 2009/2; P, =4P,
P, Solution:
SPL = sound power level (dB)
PROBLEM 21 SPL= 20log PWIL =-sound power level (dB)
P, M
Past ECE Board Problem
= at
r= distance from the sound source (m)
SPL=12 dB f
A 500 candle power that is located 12
feet from @ surface provides how many powb = 1000 Miss Solution:
-) The increase in sound pressure level 4 Ti aaantte)
lux?
is 12 dB. (241) |in)
24 in.)
SPL=PWL-= 20 log r(m)-10.9
f = 500 Hz SPL =160 20 log 1000-10.8
Votations:
PROBLEM 23 SPL = 89,1 dB
. The wavelength of 24 inches has. a
E = illumination (ax) The reverberation time of a 184.2 oubic frequency of 500 Hz.
+ «The maximum sound pressure level
|= intensity of fight source (candle power) meters broadcast studio Is 0.84 sec. on the ground directly below the
d= distance trom the ight source (m) Find the absorption effect of the plane is 89.1 dB.
materials used in metric sabines. PROBLEM 25
6 = the angle between the direction of the light and
anormal to the surface where the light strikes Past ECE, Board Problem
So Flotations:
If the distance between the listener and PROBLEM 27
fution: the source is tripled, the intensity is
Past ECE Board Problem
reduced to
te = reverberation time (5) One-hundred twenty bars of pressure
| V = volume of the room (m?} vanation is equal to
E=—, cosa ; s i 9
d? a = total absorption (m? ar metric sabines) Flatadigny:
= awe cos 0 fan
(3 66)" Solution: Vlotititin:
| = sound intensity (Wim*)
E = 37.32 lux W = sound power(W)
SPL = sound pressure level (dB)
_ O61 ¢ = distance trom the source (m)
P= sound pressure (\.bar)
ae a
P, = reference sound power = 0.0002 jibar

16-8 16-3
The Question Bank Series — Communications Engineering (Volume 2 - Problems) Question Bank 16-—- Acoustica

Solution: Solution: Notations: Flotatsnns:

f= frequency at which the absorption coefficient is fj, = resonant frequency


of the Helmbaltz resonator
SPL = 20log Far T. > 20°C,
measured (Hz) (Hz)
\ = chamber volume (m7) V= volume of the Helmholtz resonator (m*)
* f 120 pbars v= 391.45, [7g , ms
SPL = 20109\
5 9002 bar \273 r= neck radius (m)
Solution: } = length of the neck (m)
SPL =115.1608 = 331.45 a = 349.19 mis v= sound velocity = 330 m/s
where:
«120 bars is equal to 115.16 dB.
T= (+278 = 1254 '80 Solution:

=306+273 eo vr 25
PROBLEM 28 f= 1253 ——
ses
=
Ty =303 K 2x (2f+ar)V
Past ECE Board Problem f=99,96 Hz =100 Hz
A car hom outdoors. produces a sound with ¢ = 0.(no neck),
'. The velocity of sound in dry air is
intensity Javel of 90 dB at'10 ft away At .. The frequency is approximately fot ar
349.15 m/s for a 30°C temperature
that distance, what is the sound power 100 Hz: Ton VV
in watt? change.
ago 2(10 x 10)
PROBLEM 30 PROBLEM 32 toe: (aaa
Viotiettiijdse
A church has an internal volume of £=16.6 Hz
In reverberation chamber method, what
SPL =:sound pressure level (dB) volume of the chamber is required if ihe
2,550 m*. When it contains 186 metric
sabines of absorption, what will be its
PWL = sound power level (dB) absorption coefficient is to maasured at (, The resonant frequency |s 16.6 Hz.
reverberation time in seconds?
Wi=sound power (W) 500 Hz?

Solution: Voter tieiinn:


Totations:

5PL= PWL~ 20iog T=


f= frequency al which the absorption coefficient is tea = reverberation time {5}
PWL=SPL+20iog r+ a measured (Hz) Y= yolume of the room (m4)
= chamber volume (m?) a = ‘otal absorption (m* or metric sabines)
PW = = 20 8 » 206g (10)( 5 48
Solution:
PWL = 107.66 dB Solution; =
PWL =10 log WY _ 0,161V(m*)

We ='W, too" | = 0.06 watt


f=1254%80 a(r)
Ve G0 = 120,
_ 0,161(2,550)
a 488
“The sound power is 0.06 W, 125)
ge
125
tog
= 2.25

V.=2.81m .. The reverberation time is: 2.2 5.


eT ee)
. The volume of the chamber is
What |s the velocity of sound in dry air 2.81 m’. de) 1a
for a temperature change of 30°C?
What is the resonant frequency of a
aTA a ed Helmholtz resonator whose volume is
Tlatationa: 2m with neck radius of 10 em?
What frequency at which the absorption
v =sound velocity (ms) coefficient is measured if the chamber
Ta = temperature change ("K) volume required is 352 m°?

16-10 16-11

You might also like